UPSC CSE Prelims 2020 GS Paper Solution In Hindi

 


कार्बन नैनोट्यूब के संदर्भ में निम्नलिखित कथनों पर विचार कीजिएः

  1. उनका उपयोग मानव शरीर में दवाओं और एंटीजन के वाहक के रूप में किया जा सकता है।
  2. मानव शरीर के एक घायल हिस्से के लिए उन्हें कृत्रिम रक्त केशिकाओं में बनाया जा सकता है।
  3. उनका उपयोग जैव रासायनिक सेंसर में किया जा सकता है।
  4. कार्बन नैनोट्यूब बायोडिग्रेडेबल होते हैं।

ऊपर दिए गए कथनों में से कौन सा सही है?

व्याख्या
  • कार्बन नैनोट्यूब (CNTs) बेलनाकार अणु होते हैं जिनमें संकरित कार्बन परमाणुओं की एक षट्कोणीय व्यवस्था होती है, जो ग्राफीन की शीट (शीटों) को रोल करके बनाई जा सकती है।
  • हाल के वर्षों में, कार्बन नैनोट्यूब ने कई शोधकर्ताओं को दवा वितरण वाहक के रूप में आकर्षित किया है। चूंकि कार्बन नैनोट्यूब का सतह क्षेत्र उच्च होता है, वे लाखों की संख्या में सेल में जाते हैं, और एक विशिष्ट सेल को डिलीवरी की बहुत उच्च दक्षता हो सकती है। अत: कथन 1 सही है।
  • मानव शरीर के एक घायल हिस्से के लिए कार्बन नैनोट्यूब को कृत्रिम रक्त केशिकाओं में बनाया जा सकता है क्योंकि वे दवा वितरण प्लेटफार्मों का वादा कर रहे हैं जिन्हें विभिन्न प्रकार के जैव-अणुओं, जैसे एंटीबॉडी, प्रोटीन या डीएनए के साथ क्रियाशील किया जा सकता है। अत: कथन 2 सही है।
  • हाल ही में, नासा ने कार्बन नैनोट्यूब एरेज़ का उपयोग करके जैव रासायनिक सेंसर का प्रदर्शन किया है। अत: कथन 3 सही है।
  • बैक्टीरिया और कवक सहित कई प्रकार के रोगाणुओं में कार्बन नैनोट्यूब (CNTs) को नीचा दिखाने की क्षमता होती है। अत: कथन 4 सही है।

इसलिए, विकल्प (डी) सही उत्तर है।

2

निम्नलिखित गतिविधियों पर विचार करें:

  1. फसल के खेत में कीटनाशकों का छिड़काव
  2. सक्रिय ज्वालामुखियों के क्रेटरों का निरीक्षण
  3. डीएनए विश्लेषण के लिए टोंटी व्हेल से सांस के नमूने एकत्र करना

प्रौद्योगिकी के वर्तमान स्तर पर, ड्रोन का उपयोग करके उपरोक्त में से कौन सी गतिविधियों को सफलतापूर्वक किया जा सकता है?

व्याख्या
  • मानव रहित हवाई वाहन (यूएवी) या ड्रोन ऐसे विमान हैं जिन्हें बिना मानव पायलट के बोर्ड पर नेविगेट किया जा सकता है। जीपीएस ट्रैकिंग सिस्टम का उपयोग करके ड्रोन को जमीन से नियंत्रण के माध्यम से नेविगेट किया जा सकता है।
  • प्रारंभ में, ड्रोन ज्यादातर सैन्य अनुप्रयोगों के लिए विकसित किए गए हैं। हालाँकि, इसका उपयोग वैज्ञानिक, मनोरंजक, वाणिज्यिक और शांति स्थापना और निगरानी, ​​उत्पाद वितरण, हवाई फोटोग्राफी, कृषि, आदि सहित अन्य अनुप्रयोगों में विस्तारित हुआ है।
  • खड़ी फसलों को कीटों से बचाने के लिए अब इनका उपयोग कृषि क्षेत्रों में कीटनाशकों के छिड़काव के लिए किया जा रहा है। अत: कथन 1 सही है।
  • वैज्ञानिक भी सक्रिय ज्वालामुखियों का अध्ययन करने के लिए ड्रोन का उपयोग करते रहे हैं। ड्रोन सांस के नमूने एकत्र कर सकता है और सामान्य स्वास्थ्य स्थितियों का आकलन करने के लिए हवा से व्हेल की उच्च-रिज़ॉल्यूशन तस्वीरें ले सकता है। अत: कथन 2 और 3 सही हैं।

इसलिए, विकल्प (डी) सही उत्तर है।

3

"प्रयोग एक समबाहु त्रिभुज के आकार में उड़ान भरने वाले अंतरिक्ष यान की तिकड़ी को नियोजित करेगा, जिसकी भुजाएँ दस लाख किलोमीटर लंबी हैं, जिसमें लेज़र शिल्प के बीच चमकते हैं।"

प्रश्न में प्रयोग को संदर्भित करता है

व्याख्या
  • विकसित लेजर इंटरफेरोमीटर स्पेस एंटीना (ईएलआईएसए) अंतरिक्ष में तीन अंतरिक्ष यान, एक मां और दो बेटी अंतरिक्ष यान स्थापित करने की एक शानदार योजना है, जो एक त्रिकोणीय गठन में उड़ान भरेगी, जो पृथ्वी को सूर्य के चारों ओर अपनी कक्षा में अधिक दूरी पर पीछे ले जाएगी।
  • 50 मिलियन किमी. काल्पनिक त्रिभुज की प्रत्येक भुजा, माँ से लेकर प्रत्येक बेटी अंतरिक्ष यान तक, लगभग एक मिलियन किमी की दूरी तय करेगी।
  • eLISA 0.1 मेगाहर्ट्ज से लगभग 100 मेगाहर्ट्ज तक आवृत्ति रेंज में गुरुत्वाकर्षण तरंगों को मापने का प्रयास करता है। इसे प्राप्त करने के लिए, इंटरफेरोमीटर के लिए एक लाख किलोमीटर की लंबाई होना आवश्यक है और यह पृथ्वी आधारित सेटअप के साथ हासिल करना असंभव है।

इसलिए, विकल्प (डी) सही उत्तर है।

4

निम्नलिखित कथनों पर विचार करें:

  1. संभावित माता-पिता के अंडे या शुक्राणु पैदा करने वाली कोशिकाओं में आनुवंशिक परिवर्तन पेश किए जा सकते हैं।
  2. प्रारंभिक भ्रूण अवस्था में किसी व्यक्ति के जीनोम को जन्म से पहले संपादित किया जा सकता है।
  3. मानव प्रेरित प्लुरिपोटेंट स्टेम सेल को सुअर के भ्रूण में इंजेक्ट किया जा सकता है।

ऊपर दिए गए कथनों में से कौन-सा/से सही है/हैं?

व्याख्या
  • जर्मलाइन जीन थेरेपी अंडे या शुक्राणु कोशिकाओं में जीन का प्रतिस्थापन है जिसके साथ एक संतान को एक नया गुण विरासत में मिलता है। यह बीमारी पैदा करने वाले जीन वेरिएंट के सुधार की अनुमति देता है जो पीढ़ी से पीढ़ी तक पारित होने के लिए निश्चित हैं। अत: कथन 1 सही है।
  • CRISPR (क्लस्टर्ड रेगुलर इंटरस्पेस्ड शॉर्ट पैलिंड्रोमिक रिपीट) तकनीक का उपयोग मानव भ्रूण को महिलाओं के गर्भाशय में स्थानांतरित करने से पहले संशोधित करने के लिए किया जाता है। हाल ही में, शोधकर्ताओं ने दुनिया के पहले आनुवंशिक रूप से संपादित बच्चे को सफलतापूर्वक बनाया था। CRISPR तकनीक का उपयोग करके, भ्रूण के जीनोम को एक जीन, CCR5 को निष्क्रिय करने के लिए संपादित किया गया, जो एचआईवी को कोशिकाओं को संक्रमित करने की अनुमति देता है। अत: कथन 2 सही है।
  • मनुष्यों के साथ साझा की गई कुछ शारीरिक और शारीरिक विशेषताओं के कारण, सुअर को मानव रोगों का एक महत्वपूर्ण पशु मॉडल माना जाता है, जिसमें सर्जरी और ज़ेनोट्रांसप्लांटेशन अध्ययन में अद्वितीय लाभ होते हैं। अत: कथन 3 सही है।

इसलिए, विकल्प (डी) सही उत्तर है।

5

भारत में न्यूमोकोकल कंजुगेट टीकों का उपयोग करने का क्या महत्व है?

  1. ये टीके निमोनिया के साथ-साथ मेनिन्जाइटिस और सेप्सिस के खिलाफ प्रभावी हैं।
  2. एंटीबायोटिक दवाओं पर निर्भरता जो दवा प्रतिरोधी बैक्टीरिया के खिलाफ प्रभावी नहीं हैं, उन्हें कम किया जा सकता है।
  3. इन टीकों का कोई साइड इफेक्ट नहीं होता है और न ही कोई एलर्जी होती है।

नीचे दिए गए कूट का प्रयोग कर सही उत्तर चुनिए:

व्याख्या
  • न्यूमोकोकल कंजुगेट टीके (पीसीवी) न्यूमोकोकल रोगों को रोकते हैं। न्यूमोकोकल रोग न्यूमोकोकल बैक्टीरिया के कारण होने वाली किसी भी बीमारी को संदर्भित करता है। स्ट्रेप्टोकोकस न्यूमोनिया (न्यूमोकोकस) बच्चों में बैक्टीरियल निमोनिया, मेनिन्जाइटिस और सेप्सिस का एक प्रमुख कारण है।
  • पीसीवी संभावित रूप से छोटे बच्चों में जीवाणु रोग, निमोनिया, मेनिन्जाइटिस, सेप्सिस और ओटिटिस मीडिया के प्रकरणों के पर्याप्त अनुपात को रोक सकता है। अत: कथन 1 सही है।
  • आमतौर पर इस्तेमाल किए जाने वाले एंटीबायोटिक दवाओं के लिए न्यूमोकोकल बैक्टीरिया का बढ़ता प्रतिरोध न्यूमोकोकल रोग को नियंत्रित करने के लिए टीकों के उपयोग की तत्काल आवश्यकता को रेखांकित करता है। पीसीवी एंटीबायोटिक प्रतिरोधी न्यूमोकोकल संक्रमण को रोकता है। अत: कथन 2 सही है।
  • लाली, सूजन, दर्द, या कोमलता जहां शॉट दिया जाता है, और बुखार, भूख न लगना, उधम मचाना (चिड़चिड़ापन), थकान महसूस करना, सिरदर्द और ठंड लगना हो सकता है। अत: कथन 3 सही नहीं है।

अतः विकल्प (ब) सही उत्तर है।

6

भारत में, "सार्वजनिक कुंजी अवसंरचना" शब्द का प्रयोग किस संदर्भ में किया जाता है?

व्याख्या

पब्लिक की इंफ्रास्ट्रक्चर (पीकेआई) डिजिटल दुनिया में उपयोगकर्ताओं और उपकरणों को प्रमाणित करने की एक तकनीक है। इस प्रणाली के तहत, एक या अधिक विश्वसनीय पक्ष यह प्रमाणित करते हुए दस्तावेजों पर डिजिटल रूप से हस्ताक्षर करते हैं कि एक विशेष क्रिप्टोग्राफिक कुंजी किसी विशेष उपयोगकर्ता या डिवाइस से संबंधित है। तब कुंजी का उपयोग डिजिटल नेटवर्क में उपयोगकर्ता के लिए एक पहचान के रूप में किया जा सकता है।

इसलिए, विकल्प (ए) सही उत्तर है।

7

पौधे और पशु कोशिकाओं के बीच सामान्य अंतर के संबंध में निम्नलिखित में से कौन सा कथन सही है?

  1. पादप कोशिकाओं में सेल्यूलोज कोशिका भित्ति होती है जबकि पशु कोशिकाओं में नहीं होती है।
  2. जंतु कोशिकाओं के विपरीत पादप कोशिकाओं में प्लाज्मा झिल्ली नहीं होती है।
  3. परिपक्व पादप कोशिका में एक बड़ी रिक्तिका होती है जबकि पशु कोशिका में कई छोटी रिक्तिकाएँ होती हैं।

नीचे दिए गए कूट का प्रयोग कर सही उत्तर चुनिए:

व्याख्या
  • पौधे और पशु कोशिकाएं भिन्न होती हैं क्योंकि पूर्व में कोशिका भित्ति, क्लोरोप्लास्ट, प्लास्टिड और एक बड़ा केंद्रीय रिक्तिका होती है जो पशु कोशिकाओं में अनुपस्थित होती है। अत: कथन 1 और 3 सही हैं।
  • दूसरी ओर, पशु कोशिकाओं में सेंट्रीओल होते हैं जो लगभग सभी पौधों की कोशिकाओं में अनुपस्थित होते हैं। पौधे और पशु कोशिकाओं दोनों में प्लाज्मा झिल्ली होती है। अत: कथन 2 सही नहीं है।

इसलिए, विकल्प (सी) सही उत्तर है।

8

बेंजीन प्रदूषण के संपर्क में आने के निम्नलिखित में से कौन से कारण/कारक हैं?

  1. ऑटोमोबाइल निकास
  2. तंबाकू का धुआं
  3. लकड़ी का जलना
  4. वार्निश लकड़ी के फर्नीचर का उपयोग करना
  5. पॉलीयुरेथेन से बने उत्पादों का उपयोग करना

नीचे दिए गए कूट का प्रयोग कर सही उत्तर चुनिए:

व्याख्या
  • बेंजीन (सी 6 एच 6 ) एक मीठी गंध के साथ एक रंगहीन, ज्वलनशील तरल है। हवा के संपर्क में आने पर यह जल्दी वाष्पित हो जाता है। बेंजीन प्राकृतिक प्रक्रियाओं, जैसे ज्वालामुखियों और जंगल की आग से बनता है, लेकिन बेंजीन के अधिकांश जोखिम मानवीय गतिविधियों के परिणामस्वरूप होते हैं।
  • पर्यावरण में बेंजीन के मुख्य स्रोतों में ऑटोमोबाइल निकास, औद्योगिक स्रोत और गैसोलीन फिलिंग स्टेशनों से ईंधन का वाष्पीकरण शामिल हैं। अत: कथन 1 सही है।
  • इनडोर वायु में उच्च स्तर पर बेंजीन का पता चला है। हालांकि इस जोखिम में से कुछ निर्माण सामग्री (पेंट, चिपकने वाले, आदि) से हो सकता है, अधिकांश सिगरेट के धुएं से होता है। अत: कथन 2 और 4 सही हैं।

इसलिए, विकल्प (डी) सही उत्तर है।

9

यदि निकट भविष्य में एक और वैश्विक वित्तीय संकट आता है, तो निम्नलिखित में से कौन सी कार्रवाई/नीति भारत को कुछ प्रतिरक्षा प्रदान करने की सबसे अधिक संभावना है?

  1. अल्पकालिक विदेशी उधार पर निर्भर नहीं
  2. अधिक विदेशी बैंकों को खोलना
  3. पूर्ण पूंजी खाता परिवर्तनीयता बनाए रखना

नीचे दिए गए कूट का प्रयोग कर सही उत्तर चुनिए:

व्याख्या
  • विदेशी ऋण किसी सरकार, निगम या निजी परिवार द्वारा किसी अन्य देश की सरकार या निजी ऋणदाताओं से उधार लिया गया धन है। कुल विदेशी ऋण अल्पकालिक और दीर्घकालिक देनदारियों का एक संयोजन हो सकता है।
  • लंबी अवधि के विदेशी ऋणों की तुलना में, अल्पकालिक विदेशी ऋण अधिक अस्थिर होते हैं और वैश्विक संकट के समय में तरलता की कमी पैदा कर सकते हैं। इसलिए, अल्पकालिक विदेशी उधार पर निर्भर न होना निश्चित रूप से वैश्विक संकट के समय में भारत को कुछ प्रतिरक्षा प्रदान कर सकता है। अत: कथन 1 सही है।
  • विदेशी बैंकों ने भारतीय ग्राहकों को एटीएम और क्रेडिट कार्ड से परिचित कराने में महत्वपूर्ण भूमिका निभाई है। इसके अलावा, भारतीय कंपनियां तेजी से विदेशों में निवेश की तलाश में हैं, विदेशी बैंक उनके लिए धन जुटाने, उन्हें वैश्विक ग्राहकों और उपभोक्ताओं से जोड़ने में महत्वपूर्ण भूमिका निभा सकते हैं।
  • हालांकि, विदेशी पोर्टफोलियो निवेशक (एफपीआई) आज कई विदेशी बैंकों के सबसे बड़े शेयरधारक हैं, जिसका अर्थ है कि वैश्विक संकट की स्थिति में, एफपीआई अपना पैसा वापस ले लेंगे और इसे कहीं और पार्क कर देंगे, इस प्रकार, बाजार में अस्थिरता और नकदी की कमी पैदा होगी। अत: कथन 2 सही नहीं है।
  • पूंजी खाता परिवर्तनीयता विदेशी निवेशकों की भारतीय संपत्ति (जैसे इक्विटी, बांड, संपत्ति) और घरेलू नागरिकों की विदेशी वित्तीय संपत्ति खरीदने की स्वतंत्रता है। पूर्ण पूंजी खाता परिवर्तनीयता के कुछ नुकसानों में उच्च अस्थिरता, विदेशी ऋण का बढ़ा हुआ बोझ और व्यापार और निर्यात के संतुलन पर प्रभाव शामिल हैं। इस प्रकार पूर्ण पूंजी खाता परिवर्तनीयता की अनुमति वैश्विक संकट के समय में अर्थव्यवस्था को नुकसान पहुंचाएगी। अत: कथन 3 सही नहीं है।

इसलिए, विकल्प (ए) सही उत्तर है।

10

अगर आप रुपये निकालते हैं। आपके बैंक में मांग जमा खाते से नकद में 1,00,000, अर्थव्यवस्था में कुल मुद्रा आपूर्ति पर तत्काल प्रभाव होगा

व्याख्या
  • किसी विशेष समय पर जनता के बीच प्रचलन में धन के कुल स्टॉक को मुद्रा आपूर्ति कहा जाता है। यह ध्यान देने की जरूरत है कि पैसे का कुल स्टॉक पैसे की कुल आपूर्ति से अलग है।
  • मुद्रा की आपूर्ति मुद्रा के कुल भंडार का केवल वह भाग है जो जनता के पास एक विशेष समय पर होता है।
  • परिसंचारी धन में मुद्रा, मुद्रित नोट, जमा खातों में धन और अन्य तरल संपत्ति के रूप में शामिल है।
  • आरबीआई मुद्रा आपूर्ति के चार वैकल्पिक उपायों के आंकड़े प्रकाशित करता है, अर्थात। एम1, एम2, एम3 और एम4।
    • एम1 = सीयू + डीडी
    • M2 = M1 + डाकघर बचत बैंकों में बचत जमा
    • M3 = M1 + वाणिज्यिक बैंकों की निवल सावधि जमा
    • M4 = M3 + डाकघर बचत संगठनों के पास कुल जमा (राष्ट्रीय बचत प्रमाणपत्र को छोड़कर)
  • सीयू जनता द्वारा धारित मुद्रा (नोट्स प्लस सिक्के) है और डीडी वाणिज्यिक बैंकों द्वारा धारित शुद्ध मांग जमा है।
  • M1 और M2 को नैरो मनी कहा जाता है। M3 और M4 को व्यापक मुद्रा के रूप में जाना जाता है। उनकी तरलता का क्रम है: M1 > M2 > M3 > M4।
  • M1 मुद्रा आपूर्ति का एक संकीर्ण माप है जिसमें भौतिक मुद्रा, मांग जमा, ट्रैवेलर्स चेक और अन्य चेक करने योग्य जमा शामिल हैं।
  • M1 लेनदेन के लिए सबसे अधिक तरल और आसान है। इसलिए, मांग जमा खाते से नकदी की निकासी पर, अर्थव्यवस्था में कुल मुद्रा आपूर्ति पर तत्काल कोई प्रभाव नहीं पड़ेगा।

इसलिए, विकल्प (डी) सही उत्तर है।

1 1

भारत के सांस्कृतिक इतिहास के संदर्भ में, निम्नलिखित में से कौन सा 'परमित' शब्द का सही वर्णन है?

व्याख्या
  • परमिता या परमी (क्रमशः संस्कृत और पाली में) एक बौद्ध शब्द है जिसका अनुवाद अक्सर "पूर्णता" के रूप में किया जाता है।
  • महायान बौद्ध धर्म में, बोधिसत्व छह पारमिताओं, या उत्कृष्ट सिद्धियों का अभ्यास करता है जो उदारता, अनुशासन, धैर्य, परिश्रम, ध्यान एकाग्रता और ज्ञान हैं।
  • बौद्ध टीकाओं में परमिटों का वर्णन सामान्य रूप से प्रबुद्ध प्राणियों से जुड़े महान चरित्र गुणों के रूप में किया गया है।

इसलिए, विकल्प (सी) सही उत्तर है।

12

भारतीय इतिहास के सन्दर्भ में 1884 का रहमाबाई कांड इर्द-गिर्द घूमता था

  1. महिलाओं को शिक्षा प्राप्त करने का अधिकार
  2. सहमति की उम्र
  3. वैवाहिक अधिकारों की बहाली

नीचे दिए गए कूट का प्रयोग कर सही उत्तर चुनिए:

व्याख्या
  • रहमाबाई (1864-1955) ने इतिहास में अपनी पहचान उस कानूनी मामले के कारण बनाई, जिसमें वह शामिल थीं, जिसने सहमति अधिनियम, 1891 की आयु के अधिनियमन में योगदान दिया।
  • 1885 में, शादी के 12 साल बाद, उनके पति ने "वैवाहिक अधिकारों की बहाली" की मांग की, रहमाबाई को अपने पति के साथ रहने या छह महीने जेल में बिताने का आदेश दिया गया। अत: कथन 3 सही है।
  • रहमाबाई ने उस आदमी के साथ रहने से इनकार कर दिया, जिससे उसकी शादी बचपन में हुई थी, क्योंकि शादी में उसका कोई अधिकार नहीं था। रखमाबाई ने महारानी विक्टोरिया को पत्र लिखा। रानी ने अदालत के फैसले को खारिज कर दिया और शादी को भंग कर दिया।
  • इस मामले ने जो लहरें पैदा कीं, उन्होंने एज ऑफ कंसेंट एक्ट, 1891 के पारित होने पर प्रभाव डाला, जिसने पूरे ब्रिटिश साम्राज्य में बाल विवाह को अवैध बना दिया। अत: कथन 2 सही है।
  • हालांकि रखमाबाई ब्रिटिश भारत में चिकित्सा का अभ्यास करने वाली पहली महिला डॉक्टर बनीं, लेकिन यह मामला महिलाओं के शिक्षा प्राप्त करने के अधिकार से संबंधित नहीं था। अतः कथन 1 सही नहीं है।

अतः विकल्प (ब) सही उत्तर है।

13

भारत में नील की खेती 20वीं शताब्दी के प्रारंभ में कम होने के कारण हुई?

व्याख्या
  • इंडिगो एक नीला रंग है जो नील के पौधे से प्राप्त होता है। ईस्ट इंडिया कंपनी के शासन के दौरान और बाद में ब्रिटिश राज के दौरान, यह भारत में व्यापक रूप से उगाया गया था। इसे अक्सर "ब्लू गोल्ड" कहा जाता था और यूरोपीय बाजार में व्यापक रूप से कारोबार किया जाता था।
  • बढ़ती मांग और उत्पादन के कारण 18वीं और 19वीं शताब्दी में बंगाल और बिहार में नील के उत्पादन में उल्लेखनीय वृद्धि हुई। हालांकि, 20 वीं शताब्दी की शुरुआत में, सिंथेटिक नील के आविष्कार के साथ, जो कृषि नील की तुलना में सस्ता था और उत्पादन के दौरान कम समय लेता था, भारत में नील की खेती की मांग में काफी गिरावट आई। सिंथेटिक इंडिगो ने प्राकृतिक नील को किसानों और व्यापारियों के लिए गैर-लाभकारी बना दिया।

अतः विकल्प (ब) सही उत्तर है।

14

वेलेस्ली ने कलकत्ता में फोर्ट विलियम कॉलेज की स्थापना की क्योंकि

व्याख्या
  • ब्रिटिश अधिकारियों को प्रशिक्षित करने के लिए, फोर्ट विलियम कॉलेज की स्थापना 1800 में लॉर्ड रिचर्ड वेलेस्ली, गवर्नर-जनरल ऑफ बंगाल (1798 से 1805) द्वारा की गई थी। कॉलेज का उद्देश्य सिविल सेवकों को बनाना था जो भारतीय भाषाओं, इतिहास, संस्कृति और स्थानीय कानूनों से परिचित होंगे।
  • साथ ही, वे पश्चिमी भाषाओं और प्रशासन की कला में भी प्रशिक्षण प्राप्त करेंगे।

इसलिए, विकल्प (डी) सही उत्तर है।

15

भारत के इतिहास के संदर्भ में, "उलगुलान" या महान कोलाहल निम्नलिखित में से किस घटना का वर्णन है?

व्याख्या
  • बिरसा मुंडा (1875-1900) का जन्म मुंडा जनजाति में हुआ था जो छोटानागपुर क्षेत्र बंगाल प्रेसीडेंसी (वर्तमान झारखंड) में रहती थी। उन्हें अक्सर 'धरती अब्बा' या पृथ्वी पिता के रूप में जाना जाता है।
  • बिरसा मुंडा ने विद्रोह का नेतृत्व किया जिसे उलगुलान (विद्रोह) या ब्रिटिश सरकार द्वारा लगाए गए सामंती राज्य व्यवस्था के खिलाफ मुंडा विद्रोह के रूप में जाना जाने लगा।
  • उन्होंने जनता को जगाया और उनमें जमींदारों और अंग्रेजों के खिलाफ विद्रोह के बीज बोए।
  • आदिवासियों के खिलाफ शोषण और भेदभाव के खिलाफ उनके संघर्ष ने 1908 में छोटानागपुर काश्तकारी अधिनियम पारित किया, जिसने आदिवासी लोगों से गैर-आदिवासियों के लिए भूमि के पारित होने को प्रतिबंधित कर दिया।

इसलिए, विकल्प (डी) सही उत्तर है।

16

प्राचीन भारत के विद्वानों/साहित्यकारों के संदर्भ में निम्नलिखित कथनों पर विचार कीजिएः

  1. पाणिनि का संबंध पुष्यमित्र शुंग से है
  2. अमरसिंह का संबंध हर्षवर्धन से है
  3. कालिदास का संबंध चंद्र गुप्त-द्वितीय से है

ऊपर दिए गए कथनों में से कौन-सा/से सही है/हैं?

व्याख्या
  • तक्षशिला अपने शिक्षकों की विशेषज्ञता के कारण सीखने के स्थान के रूप में प्रसिद्ध हो गया। इसके विख्यात शिष्यों में प्रसिद्ध भारतीय व्याकरणविद् पाणिनी थे। वह भाषा और व्याकरण के विशेषज्ञ थे और उन्होंने अष्टाध्यायी (500 ईसा पूर्व) नामक व्याकरण पर सबसे महान कार्यों में से एक लिखा।
  • शुंग साम्राज्य मगध का एक प्राचीन भारतीय राजवंश था जिसने लगभग 185 से 75 ईसा पूर्व तक मध्य और पूर्वी भारतीय उपमहाद्वीप के क्षेत्रों को नियंत्रित किया था। मौर्य साम्राज्य के पतन के बाद पुष्यमित्र शुंग द्वारा राजवंश की स्थापना की गई थी। अतः कथन 1 सही नहीं है।
  • कहा जाता है कि अमरसिंह, एक संस्कृत व्याकरणविद् और कवि थे, जो विक्रमादित्य (चंद्रगुप्त द्वितीय) के दरबार को सुशोभित करने वाले नौ रत्नों में से एक थे, जिनका शासनकाल लगभग 375 सीई था। हर्षवर्धन ने 606 से 647 ईस्वी तक उत्तर भारत पर शासन किया। अत: कथन 2 सही नहीं है।
  • कालिदास, शास्त्रीय संस्कृत लेखक, नाटककार और नाटककार, चंद्रगुप्त द्वितीय के शासनकाल के दौरान फले-फूले, और इसलिए चौथी-पांचवीं शताब्दी सीई के आसपास रहते थे। अत: कथन 3 सही है।

इसलिए, विकल्प (सी) सही उत्तर है।

17

मानव प्रजनन प्रौद्योगिकी में हालिया प्रगति के संदर्भ में, "प्रोन्यूक्लियर ट्रांसफर" का उपयोग किसके लिए किया जाता है

व्याख्या
  • प्रोन्यूक्लियर ट्रांसफर में एक ज़ीगोट से दूसरे ज़ीगोट में प्रोन्यूक्लि का स्थानांतरण शामिल है। इस तकनीक के लिए सबसे पहले स्वस्थ दान किए गए अंडे (माइटोकॉन्ड्रियल डोनर द्वारा प्रदान किए गए) को इच्छित पुरुष माता-पिता के शुक्राणु के साथ निषेचन की आवश्यकता होती है। साथ ही, इच्छुक मां के प्रभावित oocytes को इच्छुक पिता के शुक्राणु के साथ निषेचित किया जाता है।
  • 'मातृ स्पिंडल ट्रांसफर' नामक तकनीक का उपयोग करके, मातृ डीएनए को एक दाता महिला के अंडे में डाल दिया जाता है, जिसे बाद में पिता के शुक्राणु का उपयोग करके निषेचित किया जाता है।
  • इस प्रक्रिया को मौजूदा इन-विट्रो-फर्टिलाइजेशन (आईवीएफ) उपचारों में मदद करने के लिए विकसित किया गया था जिसमें माताओं को माइटोकॉन्ड्रियल बीमारियां होती हैं।
  • मातृ डीएनए में उत्परिवर्तन माइटोकॉन्ड्रियल बीमारी का एक कारण है, बीमारियों का एक विषम समूह जो समय से पहले मौत का कारण बन सकता है, कभी-कभी बचपन या बचपन में। अधिकांश माइटोकॉन्ड्रियल बीमारियों में विशिष्ट उपचार की कमी होती है, और जो महिलाएं कारक उत्परिवर्तन करती हैं, उनके संतानों को रोग प्रसारित करने का उच्च जोखिम होता है।

इसलिए, विकल्प (डी) सही उत्तर है।

18

विकास की वर्तमान स्थिति के साथ, आर्टिफिशियल इंटेलिजेंस निम्नलिखित में से क्या प्रभावी ढंग से कर सकता है?

  1. औद्योगिक इकाइयों में बिजली की खपत कम करें
  2. सार्थक लघु कथाएँ और गीत बनाएँ
  3. रोग निदान
  4. टेक्स्ट-टू-स्पीच रूपांतरण
  5. विद्युत ऊर्जा का वायरलेस संचरण

नीचे दिए गए कूट का प्रयोग कर सही उत्तर चुनिए:

व्याख्या
  • Google अपने डेटा केंद्रों में ऊर्जा की खपत को 30% तक कम करने के लिए अपने डीपमाइंड अधिग्रहण से इंटरनेट ऑफ थिंग्स (IoT) और आर्टिफिशियल इंटेलिजेंस (AI) का उपयोग कर रहा है। अत: विकल्प 1 सही है।
  • संगीत बनाने या संगीतकारों की सहायता करने के लिए एआई को एक उपकरण के रूप में उपयोग करना काफी समय से चलन में है। 1990 के दशक में, डेविड बॉवी ने वर्बासाइज़र को विकसित करने में मदद की, जिसने साहित्यिक स्रोत सामग्री ली और नए संयोजनों को बनाने के लिए शब्दों को बेतरतीब ढंग से फिर से व्यवस्थित किया जिन्हें गीत के रूप में इस्तेमाल किया जा सकता था। 2016 में, सोनी ने द बीटल्स की शैली में एक राग बनाने के लिए फ्लो मशीन्स नामक सॉफ्टवेयर का उपयोग किया। संगीत बनाने वाला AI सॉफ्टवेयर पिछले कुछ वर्षों में उन्नत हो गया है। यह एक व्यवहार्य उपकरण है जो रचनात्मक प्रक्रिया में मदद करने के लिए उत्पादकों द्वारा उपयोग किया जा सकता है और किया जा रहा है। अत: विकल्प 2 सही है।
  • रोबोटिक्स और इंटरनेट ऑफ मेडिकल थिंग्स (IoMT) के साथ संयुक्त रूप से AI स्वास्थ्य देखभाल के लिए संभावित रूप से नया तंत्रिका तंत्र हो सकता है, जो स्वास्थ्य संबंधी समस्याओं के समाधान के लिए समाधान पेश करता है। कैंसर देखभाल में एआई प्रौद्योगिकी के एकीकरण से निदान की सटीकता और गति में सुधार हो सकता है, नैदानिक ​​निर्णय लेने में सहायता मिल सकती है और बेहतर स्वास्थ्य परिणाम प्राप्त हो सकते हैं। अत: विकल्प 3 सही है।
  • वाक् संश्लेषण मानव भाषण का कृत्रिम उत्पादन है। यह भाषा को मानव आवाज (या भाषण) में बदलने का एक तरीका है। उदाहरण के लिए, Google का सहायक, Amazon का Echo, Apple का Siri, आदि। इसलिए, विकल्प 4 सही है।
  • ऊर्जा क्षेत्र में अल के उपयोग के संभावित मामलों में ऊर्जा प्रणाली मॉडलिंग और पूर्वानुमान शामिल हैं जो अप्रत्याशितता को कम करने और शक्ति संतुलन और उपयोग में दक्षता बढ़ाने के लिए हैं। विद्युत ऊर्जा का वायरलेस ट्रांसमिशन कोई नई तकनीक नहीं है। एक वायरलेस पावर ट्रांसमिशन सिस्टम में, एक ट्रांसमीटर डिवाइस, एक शक्ति स्रोत से विद्युत शक्ति द्वारा संचालित होता है, एक समय-भिन्न विद्युत चुम्बकीय क्षेत्र उत्पन्न करता है, जो एक रिसीवर डिवाइस को अंतरिक्ष में शक्ति पहुंचाता है, जो क्षेत्र से बिजली निकालता है और इसे विद्युत आपूर्ति करता है। भार। अत: विकल्प 5 सही है।

इसलिए, विकल्प (डी) सही उत्तर है।

19

विजिबल लाइट कम्युनिकेशन (वीएलसी) तकनीक के संदर्भ में, निम्नलिखित में से कौन सा कथन सही है?

  1. वीएलसी विद्युत चुम्बकीय स्पेक्ट्रम तरंग दैर्ध्य 375 से 780 एनएम का उपयोग करता है।
  2. वीएलसी को लंबी दूरी के ऑप्टिकल वायरलेस संचार के रूप में जाना जाता है।
  3. वीएलसी ब्लूटूथ की तुलना में बड़ी मात्रा में डेटा तेजी से प्रसारित कर सकता है।
  4. वीएलसी में कोई विद्युत चुम्बकीय हस्तक्षेप नहीं है।

नीचे दिए गए कूट का प्रयोग कर सही उत्तर चुनिए:

व्याख्या
  • विज़िबल लाइट कम्युनिकेशन (वीएलसी) सिस्टम संचार के लिए दृश्य प्रकाश को नियोजित करते हैं जो 375 एनएम से 780 एनएम तक विद्युत चुम्बकीय स्पेक्ट्रम पर कब्जा कर लेते हैं। अत: कथन 1 सही है।
  • वीएलसी को शॉर्ट-रेंज ऑप्टिकल वायरलेस कम्युनिकेशन के रूप में जाना जाता है। अत: कथन 2 सही नहीं है।
  • Li-Fi, एक प्रकार का VLC है, जिसकी सीमा लगभग 10 मीटर है और यह दीवारों या किसी ठोस वस्तु से नहीं गुजर सकता है।
  • वीएलसी ब्लूटूथ की तुलना में बड़ी मात्रा में डेटा तेजी से प्रसारित कर सकता है। वीएलसी संचार के लिए दृश्य प्रकाश का उपयोग करता है ताकि 10 जीबी/एस तक उच्च गति इंटरनेट प्रदान किया जा सके जबकि ब्लूटूथ 4.0 25 एमबी/एस तक की गति का वादा करता है। अत: कथन 3 सही है।
  • वीएलसी में कोई विद्युत चुम्बकीय हस्तक्षेप नहीं है। रेडियो फ्रीक्वेंसी (आरएफ) आधारित संकेतों में अन्य आरएफ संकेतों के साथ हस्तक्षेप की समस्या होती है जैसे कि विमान में पायलट नौवहन उपकरण संकेतों के साथ इसका हस्तक्षेप। इसलिए, विद्युत चुम्बकीय विकिरण (जैसे वायुयान) के प्रति संवेदनशील क्षेत्रों में वीएलसी एक बेहतर समाधान हो सकता है। अत: कथन 4 सही है।

इसलिए, विकल्प (सी) सही उत्तर है।

20

"ब्लॉकचैन टेक्नोलॉजी" के संदर्भ में, निम्नलिखित कथनों पर विचार करें:

  1. यह एक सार्वजनिक बहीखाता है जिसका निरीक्षण हर कोई कर सकता है, लेकिन जिसे कोई एकल उपयोगकर्ता नियंत्रित नहीं करता है।
  2. ब्लॉकचेन का स्ट्रक्चर और डिजाइन ऐसा है कि इसमें मौजूद सारा डेटा क्रिप्टोकरेंसी के बारे में ही होता है।
  3. ब्लॉकचेन की बुनियादी विशेषताओं पर निर्भर एप्लिकेशन बिना किसी की अनुमति के विकसित किए जा सकते हैं।

ऊपर दिए गए कथनों में से कौन-सा/से सही है/हैं?

व्याख्या
  • एक ब्लॉकचैन सार्वजनिक खाता बही का एक रूप है, जो ब्लॉकों की एक श्रृंखला (या श्रृंखला) है, जिस पर निर्दिष्ट नेटवर्क प्रतिभागियों द्वारा उपयुक्त प्रमाणीकरण और सत्यापन के बाद लेनदेन विवरण दर्ज किया जाता है और सार्वजनिक डेटाबेस पर संग्रहीत किया जाता है। एक सार्वजनिक बहीखाता देखा जा सकता है लेकिन किसी एक उपयोगकर्ता द्वारा नियंत्रित नहीं किया जा सकता है। अत: कथन 1 सही है।
  • ब्लॉकचेन न केवल क्रिप्टोक्यूरेंसी के बारे में है, बल्कि यह पता चला है कि ब्लॉकचेन वास्तव में अन्य प्रकार के लेनदेन के बारे में डेटा संग्रहीत करने का एक बहुत ही विश्वसनीय तरीका है।
  • वास्तव में, ब्लॉकचेन तकनीक का उपयोग संपत्ति के आदान-प्रदान, बैंक लेनदेन, स्वास्थ्य सेवा, स्मार्ट अनुबंध, आपूर्ति श्रृंखला और यहां तक ​​कि एक उम्मीदवार के लिए मतदान में भी किया जा सकता है। अत: कथन 2 सही नहीं है।
  • हालाँकि क्रिप्टोकरेंसी को विनियमित किया जाता है और इसे केंद्रीय अधिकारियों के अनुमोदन की आवश्यकता होती है, ब्लॉकचेन तकनीक केवल क्रिप्टोकरेंसी के बारे में नहीं है। इसके विभिन्न उपयोग हो सकते हैं, और प्रौद्योगिकी की बुनियादी विशेषताओं के आधार पर अनुप्रयोगों को बिना किसी की स्वीकृति के विकसित किया जा सकता है। अत: कथन 3 सही है।

इसलिए, विकल्प (डी) सही उत्तर है।

21

सरकार की संसदीय प्रणाली वह है जिसमें

व्याख्या
  • सरकार की संसदीय प्रणाली वह है जिसमें सरकार संसद के प्रति उत्तरदायी होती है और इसे इसके द्वारा हटाया जा सकता है। ऐसी प्रणाली में, राष्ट्रपति या सम्राट की भूमिका मुख्य रूप से औपचारिक होती है और कैबिनेट के साथ-साथ प्रधान मंत्री प्रभावी शक्ति का उत्पादन करते हैं।
  • भारत के संविधान के अनुच्छेद 75(3) के अनुसार, मंत्रिपरिषद सामूहिक रूप से लोक सभा (लोकसभा) के प्रति उत्तरदायी है जो संसद का एक घटक है। लोकसभा के नियम इस सामूहिक जिम्मेदारी के परीक्षण के लिए एक तंत्र प्रदान करते हैं। वे किसी भी लोकसभा सांसद को, जो 50 सहयोगियों का समर्थन प्राप्त कर सकते हैं, मंत्रिपरिषद के खिलाफ अविश्वास प्रस्ताव पेश करने की अनुमति देते हैं। यदि लोकसभा में अविश्वास प्रस्ताव पारित हो जाता है तो सरकार गिर जाती है।

अतः विकल्प (ब) सही उत्तर है।

22

भारत के संविधान का कौन सा भाग कल्याणकारी राज्य के आदर्श की घोषणा करता है?

व्याख्या
  • राज्य के नीति निर्देशक सिद्धांतों (DPSP) का उल्लेख भारतीय संविधान के भाग IV में अनुच्छेद 36 से 51 तक किया गया है। DPSP एक आधुनिक और कल्याणकारी राज्य के लिए एक व्यापक सामाजिक, आर्थिक और राजनीतिक कार्यक्रम का गठन करता है। ये सिद्धांत इस बात पर जोर देते हैं कि राज्य लोगों को आश्रय, भोजन और वस्त्र जैसी बुनियादी सुविधाएं प्रदान करके उनके कल्याण को बढ़ावा देने का प्रयास करेगा। वे एक 'कल्याणकारी राज्य' की अवधारणा को मूर्त रूप देते हैं जो औपनिवेशिक युग के दौरान अनुपस्थित था।

इसलिए, विकल्प (ए) सही उत्तर है।

23

निम्नलिखित कथनों पर विचार करें:

  1. भारत का संविधान संघवाद, धर्मनिरपेक्षता, मौलिक अधिकारों और लोकतंत्र के संदर्भ में अपनी 'बुनियादी संरचना' को परिभाषित करता है।
  2. भारत का संविधान नागरिकों की स्वतंत्रता की रक्षा और उन आदर्शों को संरक्षित करने के लिए 'न्यायिक समीक्षा' का प्रावधान करता है जिन पर संविधान आधारित है।

ऊपर दिए गए कथनों में से कौन-सा/से सही है/हैं?

व्याख्या
  • भारत का संविधान बुनियादी ढांचे को परिभाषित नहीं करता है, यह एक न्यायिक नवाचार है।
  • केशवानंद भारती बनाम केरल राज्य मामले (1973) में, सर्वोच्च न्यायालय ने फैसला सुनाया कि संसद संविधान के किसी भी हिस्से में तब तक संशोधन कर सकती है जब तक कि वह संविधान की मूल संरचना या आवश्यक विशेषताओं में परिवर्तन या संशोधन नहीं करती है।
  • हालाँकि, अदालत ने 'बुनियादी संरचना' शब्द को परिभाषित नहीं किया, और केवल कुछ सिद्धांतों को सूचीबद्ध किया - संघवाद, धर्मनिरपेक्षता, लोकतंत्र - इसके हिस्से के रूप में।
  • तब से 'मूल संरचना' सिद्धांत की व्याख्या संविधान की सर्वोच्चता, कानून के शासन, न्यायपालिका की स्वतंत्रता, शक्तियों के पृथक्करण के सिद्धांत, संप्रभु लोकतांत्रिक गणराज्य, सरकार की संसदीय प्रणाली, स्वतंत्र और निष्पक्ष के सिद्धांत को शामिल करने के लिए की गई है। चुनाव, कल्याणकारी राज्य, आदि। इसलिए, कथन 1 सही नहीं है।
  • संविधान के अनुच्छेद 13(2) में कहा गया है कि संघ या राज्य ऐसा कोई कानून नहीं बनाएंगे जो किसी भी मौलिक अधिकार को छीनता या कम करता है, और उपरोक्त जनादेश के उल्लंघन में बनाया गया कोई भी कानून, उल्लंघन की सीमा तक, शून्य हो। इस प्रकार, संविधान स्वयं नागरिक की स्वतंत्रता और अधिकारों की रक्षा के लिए न्यायिक समीक्षा का प्रावधान करता है। अत: कथन 2 सही है।

अतः विकल्प (ब) सही उत्तर है।

24

गांधीवाद और मार्क्सवाद के बीच एक आम सहमति है

व्याख्या
  • गांधीवाद और मार्क्सवाद दोनों का अंतिम उद्देश्य एक राज्यविहीन और वर्गविहीन समाज की स्थापना करना है, हालाँकि इस लक्ष्य को प्राप्त करने के उनके साधन अलग-अलग हैं। महात्मा गांधी इस लक्ष्य को अहिंसक तरीकों से हासिल करना चाहते थे लेकिन मार्क्स इसे हिंसक तरीकों से हासिल करना चाहते थे।
  • वर्गहीन समाज को प्राप्त करने के उपाय
    • मार्क्सवाद: पूंजीवाद को उखाड़ फेंका; सामाजिक उत्पादन के साधनों का सामाजिक स्वामित्व; अनिवार्य श्रम।
    • गांधीवाद: ट्रस्टीशिप का सिद्धांत: खुद को सार्वजनिक संपत्ति का 'ट्रस्टी' मानने के लिए राजधानियों का नैतिक अनुनय; श्रम की गरिमा बहाल करो।

इसलिए, विकल्प (ए) सही उत्तर है।

25

भारत के संदर्भ में, निम्नलिखित में से कौन-सी विशेषता नौकरशाही के लिए उपयुक्त है?

व्याख्या
  • सिविल सेवा या नौकरशाही उन पेशेवरों का गठन करती है जो स्थायी और वेतनभोगी कर्मचारी होते हैं जो सरकार के कार्यकारी अंग का हिस्सा होते हैं। वे राजनीतिक रूप से तटस्थ हैं और उनका मुख्य काम विभिन्न सरकारी विभागों के प्रभावी कामकाज और सार्वजनिक नीति कार्यान्वयन को सुविधाजनक बनाना है। हालाँकि, वे मंत्रियों के नियंत्रण और नेतृत्व में काम करते हैं। नौकरशाही व्यवस्था को एक संगठन के भीतर एकरूपता और नियंत्रण बनाए रखने के लिए डिज़ाइन किया गया है।
  • अच्छी नीतियां और कानून वास्तव में अपने उद्देश्यों की पूर्ति तभी कर सकते हैं जब इन्हें सिविल सेवकों द्वारा कुशलता से लागू किया जाए।

इसलिए, विकल्प (डी) सही उत्तर है।

26

भारत के संविधान की प्रस्तावना है

व्याख्या
  • प्रस्तावना संविधान का परिचयात्मक भाग है। इसमें संविधान के आदर्श, उद्देश्य और बुनियादी सिद्धांत शामिल हैं। संविधान की मुख्य विशेषताएं इन उद्देश्यों से प्रत्यक्ष और अप्रत्यक्ष रूप से विकसित हुई हैं जो प्रस्तावना से निकलती हैं।
  • केशवानंद भारती केस (1973) में, सर्वोच्च न्यायालय ने कहा कि:
    • संविधान की प्रस्तावना को संविधान का अंग माना जाए।
    • प्रस्तावना सर्वोच्च शक्ति या किसी प्रतिबंध या निषेध का स्रोत नहीं है, लेकिन यह संविधान की विधियों और प्रावधानों की व्याख्या में महत्वपूर्ण भूमिका निभाती है।
  • केंद्र सरकार बनाम एलआईसी ऑफ इंडिया केस (1995) में, सुप्रीम कोर्ट ने एक बार फिर यह माना है कि प्रस्तावना संविधान का अभिन्न अंग है, लेकिन भारत में कानून की अदालत में सीधे लागू नहीं है। इसके उद्देश्यों को विभिन्न अधिनियमों और नीतियों के माध्यम से कार्यान्वित किया जाता है।

इसलिए, विकल्प (डी) सही उत्तर है।

27

"गोल्ड ट्रेंच" (रिजर्व ट्रेंच) को संदर्भित करता है

व्याख्या
  • गोल्ड ट्रेंच या रिजर्व ट्रेंच अंतरराष्ट्रीय मुद्रा कोष (आईएमएफ) के साथ सदस्य देश के कोटे का घटक है यानी सोने या विदेशी मुद्रा के रूप में। किसी भी सदस्य देश के लिए कुल कोटे में से 25% का भुगतान विदेशी मुद्रा या सोने के रूप में किया जाना चाहिए। इसलिए, इसे आरक्षित किश्त या स्वर्ण किश्त कहा जाता है। शेष 75% घरेलू मुद्राओं में हो सकता है और इसे क्रेडिट किश्त कहा जाता है। यह मूल रूप से एक आपातकालीन खाता है जिसे IMF के सदस्य शर्तों से सहमत हुए बिना या सेवा शुल्क का भुगतान किए बिना एक्सेस कर सकते हैं।

इसलिए, विकल्प (डी) सही उत्तर है।

28

भारत के संविधान के भाग IV में निहित प्रावधानों के संदर्भ में, निम्नलिखित में से कौन सा/से कथन सही है/हैं?

  1. वे न्यायालयों द्वारा प्रवर्तनीय होंगे।
  2. वे किसी भी न्यायालय द्वारा प्रवर्तनीय नहीं होंगे।
  3. इस भाग में निर्धारित सिद्धांत राज्य द्वारा कानून बनाने को प्रभावित करना है।

नीचे दिए गए कूट का प्रयोग कर सही उत्तर चुनिए:

व्याख्या
  • मौलिक अधिकारों (भाग III) के विपरीत, संविधान के भाग IV (अनुच्छेद 36-51) में निहित राज्य नीति के निदेशक सिद्धांत (डीपीएसपी) प्रकृति में गैर-न्यायिक हैं यानी उनके उल्लंघन के लिए अदालतों द्वारा उन्हें लागू नहीं किया जा सकता है। अत: कथन 1 सही नहीं है, जबकि कथन 2 सही है।
  • संविधान (अनुच्छेद 37) ही कहता है कि ये सिद्धांत देश के शासन में मौलिक हैं। कानून बनाने में इन सिद्धांतों को लागू करना राज्य का कर्तव्य होगा। साथ ही, निर्देशक सिद्धांत कानून की संवैधानिक वैधता की जांच और निर्धारण में अदालतों की मदद करते हैं। अत: कथन 3 सही है।

इसलिए, विकल्प (डी) सही उत्तर है।

29

निम्नलिखित कथनों पर विचार करें:

  1. भारत के संविधान के अनुसार, एक व्यक्ति जो वोट देने के योग्य है, उसे किसी राज्य में छह महीने के लिए मंत्री बनाया जा सकता है, भले ही वह उस राज्य के विधानमंडल का सदस्य न हो।
  2. जनप्रतिनिधित्व अधिनियम, 1951 के अनुसार, एक व्यक्ति को एक आपराधिक अपराध का दोषी ठहराया गया और पांच साल के कारावास की सजा सुनाई गई, उसे जेल से रिहा होने के बाद भी चुनाव लड़ने के लिए स्थायी रूप से अयोग्य घोषित कर दिया गया।

ऊपर दिए गए कथनों में से कौन-सा/से सही है/हैं?

व्याख्या
  • संविधान के अनुच्छेद 164 के अनुसार, एक मंत्री जो लगातार छह महीने तक राज्य विधानमंडल का सदस्य नहीं है, वह मंत्री नहीं रहेगा। यह प्रावधान एक गैर-विधायक को छह महीने के लिए मुख्यमंत्री के कार्यालय सहित मंत्रिपरिषद में एक पद पर कब्जा करने की अनुमति देता है। छह महीने के भीतर, उसे राज्य विधानमंडल के किसी भी सदन का सदस्य (या तो चुनाव या नामांकन द्वारा) बनना होगा, अन्यथा, वह मंत्री नहीं रह जाएगा।
  • इसके अलावा, राज्य विधानमंडल का सदस्य बनने के लिए विधान परिषद के मामले में आयु 30 वर्ष से कम नहीं होनी चाहिए और विधान सभा के मामले में 25 वर्ष से कम नहीं होनी चाहिए। ऐसा व्यक्ति स्वतः ही मतदान के योग्य हो जाता है क्योंकि अनुच्छेद 326 के तहत मतदाता के रूप में पंजीकृत होने की न्यूनतम आयु 18 वर्ष है। अत: कथन 1 सही है।
  • जनप्रतिनिधित्व अधिनियम, 1951 की धारा 8 (3) के अनुसार, किसी भी अपराध के लिए दोषी व्यक्ति और कम से कम दो साल के कारावास की सजा दी गई सजा की तारीख से चुनाव (विधायक या सांसद) चुनाव लड़ने के लिए अयोग्य घोषित किया जाएगा। और उनकी रिहाई के बाद से छह साल की एक और अवधि के लिए अयोग्य घोषित किया जाना जारी रहेगा। अत: कथन 2 सही नहीं है।

इसलिए, विकल्प (ए) सही उत्तर है।

30

निम्नलिखित कथनों पर विचार करें:

  1. भारत का राष्ट्रपति संसद का सत्र ऐसी जगह पर बुला सकता है जो वह ठीक समझे।
  2. भारत का संविधान एक वर्ष में संसद के तीन सत्रों का प्रावधान करता है, लेकिन सभी तीन सत्रों का संचालन करना अनिवार्य नहीं है।
  3. एक वर्ष में संसद की बैठक के लिए आवश्यक न्यूनतम दिनों की संख्या नहीं है।

ऊपर दिए गए कथनों में से कौन-सा/से सही है/हैं?

व्याख्या
  • संविधान का अनुच्छेद 85(1) राष्ट्रपति को यह अधिकार देता है कि वह संसद के प्रत्येक सदन को ऐसे समय और स्थान पर बैठक करने के लिए बुलाए, जो वह ठीक समझे, लेकिन एक सत्र में उसकी अंतिम बैठक और उसकी पहली बैठक के लिए नियत तारीख के बीच छह महीने का अंतराल नहीं होगा। अगले सत्र में। अत: कथन 1 सही है।
  • सम्मेलन द्वारा (भारत के संविधान द्वारा प्रदान नहीं किया गया), संसद एक वर्ष में तीन सत्रों के लिए मिलती है। बजट सत्र वर्ष की शुरुआत में आयोजित किया जाता है; तीन सप्ताह का मानसून सत्र जुलाई से अगस्त तक चलता है; और फिर नवंबर-दिसंबर में शीतकालीन सत्र होता है। अत: कथन 2 सही नहीं है।
  • एक वर्ष में संसद की बैठक के लिए आवश्यक न्यूनतम दिनों की कोई संख्या नहीं है - वास्तव में, संसद की बैठक के दिनों की संख्या वर्षों में कम हो गई है। संसद के पहले दो दशकों के दौरान, लोकसभा की बैठक औसतन एक वर्ष में 120 दिन से थोड़ी अधिक होती है। पिछले दशक में यह घटकर लगभग 70 दिन रह गया है। हालाँकि, कई समितियों ने सिफारिश की है कि संसद को एक वर्ष में कम से कम 120 दिनों के लिए मिलना चाहिए। अत: कथन 3 सही है।

इसलिए, विकल्प (सी) सही उत्तर है।

31

निम्नलिखित कथनों पर विचार करें:

  1. आधार मेटाडेटा को तीन महीने से अधिक समय तक संग्रहीत नहीं किया जा सकता है।
  2. आधार डेटा साझा करने के लिए राज्य निजी निगमों के साथ कोई अनुबंध नहीं कर सकता है।
  3. बीमा उत्पाद प्राप्त करने के लिए आधार अनिवार्य है।
  4. भारत की संचित निधि से लाभ प्राप्त करने के लिए आधार अनिवार्य है।

ऊपर दिए गए कथनों में से कौन-सा/से सही है/हैं?

व्याख्या
  • सितंबर, 2018 के सुप्रीम कोर्ट के फैसले के अनुसार, आधार मेटाडेटा को छह महीने से अधिक समय तक संग्रहीत नहीं किया जा सकता है। सुप्रीम कोर्ट ने आधार अधिनियम की धारा 2 (डी) को पढ़ लिया है, जिसमें सरकारी अधिकारियों को लेनदेन के मेटाडेटा को संग्रहीत करने से रोकने के लिए इस तरह के डेटा को पांच साल की अवधि के लिए संग्रहीत करने की अनुमति है। अतः कथन 1 सही नहीं है।
  • SC ने आधार विनियमन 26 (c) को भी रद्द कर दिया है, जिसने भारतीय विशिष्ट पहचान प्राधिकरण (UIDAI) को निजी फर्मों के लिए आधार आधारित प्रमाणीकरण या प्रमाणीकरण इतिहास से संबंधित मेटाडेटा संग्रहीत करने की अनुमति दी थी। तदनुसार, भारतीय बीमा नियामक और विकास प्राधिकरण (IRDAI) ने बीमा कंपनियों को निर्देश दिया है कि वे अपने ग्राहक को जानें (केवाईसी) आवश्यकताओं के लिए अनिवार्य रूप से आधार विवरण न मांगें या यूआईडीएआई से ई-केवाईसी का उपयोग करके प्रमाणीकरण न करें। अत: कथन 2 सही है और कथन 3 सही नहीं है।
  • इसके अलावा, आधार (वित्तीय और अन्य सब्सिडी, लाभ और सेवाओं का लक्षित वितरण) अधिनियम, 2016 की धारा 7 में किए गए संशोधन को बरकरार रखा गया है। यह एक शर्त निर्धारित करता है कि राज्य सरकार सब्सिडी, लाभ या सेवा प्राप्त करने के लिए लाभार्थियों के लिए आधार प्रमाणीकरण के उपयोग को अनिवार्य कर सकती है, जिसके लिए भारत की संचित निधि से व्यय किया जाता है। अत: कथन 4 सही है।

अतः विकल्प (ब) सही उत्तर है।

32

राज्यसभा के पास लोकसभा के समान अधिकार हैं

व्याख्या
  • राज्यसभा को लोकसभा की तुलना में कुछ विशेष शक्तियां प्राप्त हैं जो इस प्रकार हैं:
    • एक निश्चित अवधि के लिए किसी विषय को राज्य सूची से संघ सूची में स्थानांतरित करने की शक्ति (अनुच्छेद 249)।
    • अतिरिक्त अखिल भारतीय सेवाओं का सृजन करना (अनुच्छेद 312)।
    • अनुच्छेद 352 के तहत एक सीमित अवधि के लिए आपातकाल का समर्थन करने के लिए जब लोकसभा भंग रहती है।
  • इसलिए, विकल्प (ए) सही नहीं है।
  • अन्य महत्वपूर्ण मामले जिनके संबंध में दोनों सदनों को समान शक्तियां प्राप्त हैं, वे हैं राष्ट्रपति का चुनाव और महाभियोग, उपराष्ट्रपति का चुनाव, संविधान में संशोधन, आपातकाल की घोषणा को मंजूरी, राज्यों में संवैधानिक तंत्र की विफलता और वित्तीय आपातकाल की घोषणा। इसलिए, विकल्प (बी) सही है।
  • मंत्रिपरिषद सामूहिक रूप से लोकसभा के प्रति उत्तरदायी होती है, जिसका अर्थ है कि मंत्रालय तब तक अपने पद पर बना रहता है जब तक उसे लोकसभा के अधिकांश सदस्यों का विश्वास प्राप्त है। इसलिए, विकल्प (सी) सही नहीं है।
  • कटौती प्रस्ताव लोकसभा के सदस्यों में निहित एक विशेष शक्ति है जो अनुदान की मांग के हिस्से के रूप में वित्त विधेयक में सरकार द्वारा विशिष्ट आवंटन के लिए चर्चा की जा रही मांग का विरोध करती है। यदि प्रस्ताव को स्वीकार कर लिया जाता है, तो यह अविश्वास मत के बराबर होता है, और यदि सरकार निचले सदन में संख्या बढ़ाने में विफल रहती है, तो वह सदन के मानदंडों के अनुसार इस्तीफा देने के लिए बाध्य होती है। इसलिए, विकल्प (डी) सही नहीं है।

अतः विकल्प (ब) सही उत्तर है।

33

संसद सदस्य स्थानीय क्षेत्र विकास योजना (MPLADS) के तहत निधियों के संदर्भ में, निम्नलिखित में से कौन सा कथन सही है?

  1. MPLADS फंड का उपयोग स्वास्थ्य, शिक्षा आदि के लिए भौतिक बुनियादी ढांचे जैसी टिकाऊ संपत्ति बनाने के लिए किया जाना चाहिए।
  2. प्रत्येक सांसद की निधि के एक निर्दिष्ट हिस्से से अनुसूचित जाति/अनुसूचित जनजाति की आबादी को लाभ होना चाहिए।
  3. एमपीलैड्स निधियां वार्षिक आधार पर स्वीकृत की जाती हैं और अप्रयुक्त निधियों को अगले वर्ष के लिए आगे नहीं ले जाया जा सकता है।
  4. जिला प्राधिकरण को हर साल कार्यान्वयन के तहत सभी कार्यों का कम से कम 10% निरीक्षण करना चाहिए।

नीचे दिए गए कूट का प्रयोग कर सही उत्तर चुनिए:

व्याख्या
  • MPLAD योजना 1993 में तैयार की गई थी ताकि संसद सदस्य (सांसद) टिकाऊ सामुदायिक संपत्ति के निर्माण पर जोर देने के साथ अपने निर्वाचन क्षेत्र में प्रति वर्ष र 5 करोड़ के कार्यों के लिए जिला कलेक्टर को सिफारिश कर सकें। स्थानीय रूप से महसूस की गई आवश्यकता के आधार पर। टिकाऊ संपत्ति में पेयजल, प्राथमिक शिक्षा, सार्वजनिक स्वास्थ्य, स्वच्छता और सड़कें आदि शामिल हैं। इसलिए, कथन 1 सही है।
  • 5 करोड़ की राशि में से एक सांसद अनुसूचित जाति (एससी) आबादी वाले क्षेत्रों के लिए कम से कम 15% और अनुसूचित जनजाति (एसटी) आबादी वाले क्षेत्रों के लिए 7.5% की सिफारिश करेगा। अत: कथन 2 सही है।
  • एमपीलैड्स के तहत प्रत्येक वर्ष सांसदों को र 2.5 करोड़ की दो किस्तों में र 5 करोड़ मिलते हैं। एमपीलैड्स के अंतर्गत निधियां व्यपगत नहीं होती हैं अर्थात किसी विशेष वर्ष में निधि जारी न होने की स्थिति में इसे अगले वर्ष के लिए आगे ले जाया जाता है। अत: कथन 3 सही नहीं है।
  • जिला प्राधिकरण को हर साल कम से कम 10% कार्यों का दौरा और निरीक्षण करना चाहिए। इस तरह की गतिविधियों में सांसदों के शामिल होने की भी उम्मीद है। अत: कथन 4 सही है।

इसलिए, विकल्प (डी) सही उत्तर है।

34

मौलिक अधिकारों की निम्नलिखित श्रेणियों में से किस एक में भेदभाव के रूप में अस्पृश्यता के विरुद्ध संरक्षण शामिल है?

व्याख्या
  • भारतीय संविधान के तहत मौलिक अधिकारों की छह श्रेणियां हैं:
    • समानता का अधिकार (अनुच्छेद 14-18)
    • स्वतंत्रता का अधिकार (अनुच्छेद 19-22)
    • शोषण के विरुद्ध अधिकार (अनुच्छेद 23-24)
    • धर्म की स्वतंत्रता का अधिकार (अनुच्छेद 25-28)
    • सांस्कृतिक और शैक्षिक अधिकार (अनुच्छेद 29-30)
    • संवैधानिक उपचार का अधिकार (अनुच्छेद 32)
  • समानता के अधिकार (अनुच्छेद 14-18) के तहत अनुच्छेद 17, अस्पृश्यता के उन्मूलन और इसके अभ्यास के निषेध के बारे में बात करता है।
  • यह 'अस्पृश्यता' को समाप्त करता है और किसी भी रूप में इसके अभ्यास को मना करता है। अस्पृश्यता से उत्पन्न होने वाली किसी भी अक्षमता को लागू करना कानून के अनुसार दंडनीय अपराध होगा।

इसलिए, विकल्प (डी) सही उत्तर है।

35

भारत में न्यायपालिका को कार्यपालिका से अलग करने का आदेश किसके द्वारा दिया जाता है?

व्याख्या
  • राज्य के नीति निदेशक तत्व संविधान के भाग IV (अनुच्छेद 36 से 51) में वर्णित हैं।
  • संविधान का अनुच्छेद 50 निम्नानुसार पढ़ता है: न्यायपालिका को कार्यपालिका से अलग करना - राज्य की सार्वजनिक सेवाओं में न्यायपालिका को कार्यपालिका से अलग करने के लिए राज्य कदम उठाएगा।

अतः विकल्प (ब) सही उत्तर है।

36

बजट के साथ-साथ वित्त मंत्री अन्य दस्तावेज भी संसद के समक्ष रखते हैं जिनमें 'द मैक्रो इकोनॉमिक फ्रेमवर्क स्टेटमेंट' शामिल है। उपरोक्त दस्तावेज प्रस्तुत किया गया है क्योंकि यह किसके द्वारा अनिवार्य है

व्याख्या
  • भारतीय संविधान के अनुच्छेद 112 के अनुसार, एक वर्ष के केंद्रीय बजट को वार्षिक वित्तीय विवरण (AFS) कहा जाता है। यह एक वित्तीय वर्ष में सरकार की अनुमानित प्राप्तियों और व्यय का विवरण है।
  • एक 'मनी बिल' को अनुच्छेद 110(1) के तहत परिभाषित किया गया है।
  • अनुच्छेद 113 के अनुसार, भारत की संचित निधि से व्यय के अनुमान (अनुदान की मांगों के रूप में) को वार्षिक वित्तीय विवरण में शामिल किया जाना चाहिए और लोकसभा द्वारा मतदान किया जाना आवश्यक है।
  • राजकोषीय उत्तरदायित्व और बजट प्रबंधन (एफआरबीएम) अधिनियम, 2003 एक संतुलित बजट की ओर बढ़ते हुए वित्तीय अनुशासन को संस्थागत बनाने, भारत के राजकोषीय घाटे को कम करने, व्यापक आर्थिक प्रबंधन और सार्वजनिक निधियों के समग्र प्रबंधन में सुधार करने का प्रयास करता है।
  • यह अधिनियम केंद्र सरकार को वार्षिक वित्तीय विवरण और अनुदान की मांगों के साथ संसद के सदनों, मैक्रो-इकोनॉमिक फ्रेमवर्क स्टेटमेंट, मध्यम अवधि के राजकोषीय नीति वक्तव्य और राजकोषीय नीति रणनीति वक्तव्य के समक्ष रखने के लिए अनिवार्य करता है।

इसलिए, विकल्प (डी) सही उत्तर है।

37

परिभाषा के अनुसार एक संवैधानिक सरकार है

व्याख्या
  • एक संवैधानिक सरकार को एक संविधान के अस्तित्व से परिभाषित किया जाता है, जो लिखित या अलिखित हो सकता है, जिसे राजनीति के मौलिक कानून के रूप में स्वीकार किया जाता है जो राजनीतिक शक्ति के प्रयोग को प्रभावी ढंग से नियंत्रित करता है।
  • संवैधानिक सरकार का मुख्य घटक "कानून का नियम" या "बुनियादी कानूनों" का एक सेट है जो सार्वजनिक कार्यालय धारकों और समाज के सभी सदस्यों (यानी नागरिकों) को किसी दिए गए क्षेत्र के भीतर बांधता है।
  • संवैधानिकता का सार सरकार की कार्यकारी, विधायी और न्यायिक शाखाओं के बीच सत्ता के वितरण द्वारा सत्ता का नियंत्रण है, इन शक्तियों को संतुलित करने में एक विशेष दृष्टिकोण के साथ।
  • संवैधानिक सरकार एक प्रकार का शासन है जो इस तथ्य की विशेषता है कि सरकार कानूनी और संस्थागत बाधाओं के एक सेट के भीतर काम करती है जो दोनों अपनी शक्ति (सीमित सरकार) को सीमित करती है और व्यक्तिगत स्वतंत्रता की रक्षा करती है।
  • एक संवैधानिक सरकार में विधायिका नहीं हो सकती है, एक द्विदलीय सरकार हो सकती है और एक लोकप्रिय सरकार होने की आवश्यकता नहीं है और इसके बजाय एक राजशाही हो सकती है।

इसलिए, विकल्प (डी) सही उत्तर है।

38

मौलिक अधिकारों के अलावा, भारत के संविधान के निम्नलिखित में से कौन सा भाग मानवाधिकारों की सार्वभौम घोषणा (1948) के सिद्धांतों और प्रावधानों को दर्शाता है/प्रतिबिंबित करता है?

  1. प्रस्तावना
  2. राज्य के नीति निर्देशक सिद्धांत
  3. मौलिक कर्तव्य

नीचे दिए गए कूट का प्रयोग कर सही उत्तर चुनिए:

व्याख्या
  • 1948 में संयुक्त राष्ट्र महासभा (यूएनजीए) द्वारा स्वीकृत और घोषित मानवाधिकारों की सार्वभौम घोषणा (यूडीएचआर) प्रत्येक इंसान की समानता और गरिमा को स्थापित करती है और यह निर्धारित करती है कि हर सरकार का एक मुख्य कर्तव्य है कि सभी लोगों को सभी का आनंद लेने में सक्षम बनाया जाए। उनके अक्षम्य अधिकार और स्वतंत्रता।
  • प्रस्तावना: प्रस्तावना के उद्देश्य जैसे न्याय (सामाजिक, आर्थिक और राजनीतिक), समानता और स्वतंत्रता भी यूडीएचआर के सिद्धांतों को दर्शाते हैं। अत: कथन 1 सही है।
  • राज्य नीति के निदेशक सिद्धांत (डीपीएसपी): अनुच्छेद 36 से 51 के तहत प्रदान किए गए, डीपीएसपी ऐसे सिद्धांत हैं जिनका उद्देश्य सामाजिक और आर्थिक न्याय प्रदान करना है और कल्याणकारी राज्य की ओर मार्ग निर्धारित करना है। ये डीपीएसपी राज्य पर दायित्व के रूप में कार्य करते हैं और मानवाधिकारों के अनुरूप हैं। कुछ डीपीएसपी जो मानव अधिकारों के साथ तालमेल बिठाते हैं, वे इस प्रकार हैं:
    • अनुच्छेद 38: कल्याणकारी राज्य को बढ़ावा देना
    • अनुच्छेद 39: असमानताओं को कम करना
    • अनुच्छेद 39ए: मुफ्त कानूनी सहायता
    • अनुच्छेद 41: बेरोजगार, बीमार, विकलांग और वृद्ध व्यक्तियों जैसे समाज के कमजोर वर्गों का समर्थन करना।
    • अनुच्छेद 43: निर्वाह मजदूरी सुरक्षित करना
  • अत: कथन 2 सही है।
  • मौलिक कर्तव्य (अनुच्छेद 51ए): ये मूल रूप से भारत के सभी नागरिकों के नागरिक और नैतिक दायित्व हैं। अभी तक, भारत में 11 मौलिक कर्तव्य हैं, जो संविधान के भाग IV A में लिखे गए हैं। अनुच्छेद 51 ए (के) माता-पिता या अभिभावक द्वारा 6 से 14 वर्ष की आयु के बच्चे को शिक्षा के अवसर प्रदान करने की बात करता है। यह पहलू किसी न किसी तरह शिक्षा के माध्यम से बच्चों की गरिमा सुनिश्चित करने से संबंधित है। अत: कथन 3 सही है।

इसलिए, विकल्प (डी) सही उत्तर है।

39

भारत में, कानूनी सेवा प्राधिकरण निम्नलिखित में से किस प्रकार के नागरिकों को मुफ्त कानूनी सेवाएं प्रदान करते हैं?

  1. 1,00,000 . से कम वार्षिक आय वाले व्यक्ति
  2. 2,00,000 . से कम वार्षिक आय वाले ट्रांसजेंडर
  3. 3,00,000 . से कम वार्षिक आय वाले अन्य पिछड़ा वर्ग (ओबीसी) के सदस्य
  4. सभी वरिष्ठ नागरिक

नीचे दिए गए कूट का प्रयोग कर सही उत्तर चुनिए:

व्याख्या
  • 1987 में, संसद द्वारा कानूनी सेवा प्राधिकरण अधिनियम बनाया गया था जो समान अवसर के आधार पर समाज के कमजोर वर्गों को सक्षम कानूनी सेवाएं प्रदान करने के लिए एक राष्ट्रव्यापी वर्दी नेटवर्क स्थापित करने के लिए 9 नवंबर, 1995 को लागू हुआ था।
  • अधिनियम की धारा 12 के तहत, मुफ्त कानूनी सेवाएं प्राप्त करने के लिए पात्र व्यक्तियों में शामिल हैं:
    • महिलाएं और बच्चे
    • अनुसूचित जाति/अनुसूचित जनजाति के सदस्य
    • औद्योगिक कामगार
    • आपदा, हिंसा, बाढ़, सूखा, भूकंप के शिकार
    • विकलांग व्यक्तियों
    • हिरासत में व्यक्ति
    • मानव तस्करी के शिकार या बेगार
  • इसके अलावा, अधिनियम की धारा 12 (एच) के अनुसार, 9000 रुपये से कम वार्षिक आय प्राप्त करने वाले व्यक्ति या राज्य सरकार द्वारा निर्धारित अन्य उच्च राशि, यदि मामला सर्वोच्च न्यायालय के अलावा किसी अन्य अदालत के समक्ष है, और ?12000 से कम या केंद्र सरकार द्वारा निर्धारित ऐसी अन्य उच्च राशि, यदि मामला सर्वोच्च न्यायालय के समक्ष है, तो भी मुफ्त कानूनी सेवाओं के लिए पात्र हैं। अधिनियम की धारा 12 (एच) के तहत निर्धारित आय सीमा सीमा राज्यों में भिन्न होती है। यह लक्षद्वीप में 9000 से लेकर आंध्र प्रदेश में 3,00,000 तक है। अत: कथन 1 सही है।
  • मुफ्त कानूनी सहायता के लिए वरिष्ठ नागरिकों की पात्रता इस संबंध में संबंधित राज्य सरकारों द्वारा बनाए गए नियमों पर निर्भर करती है। इसलिए, सभी वरिष्ठ नागरिक मुफ्त कानूनी सहायता के पात्र नहीं हैं। अत: कथन 4 सही नहीं है।
  • अन्य पिछड़ा वर्ग (ओबीसी) के लिए ऐसा कोई स्पष्ट सीमांकन नहीं है, अर्थात क्या ओबीसी का सदस्य जिसकी वार्षिक आय 3,00,000 से कम है, अधिनियम के तहत मुफ्त कानूनी सहायता प्राप्त करेगा। अत: कथन 3 सही नहीं है।

इसलिए, विकल्प (ए) सही उत्तर है।

40

निम्नलिखित जोड़े पर विचार करें:

अंतर्राष्ट्रीय समझौता/सेट-अपविषय
1. अल्मा-अता घोषणा:लोगों का स्वास्थ्य
2. हेग कन्वेंशन:जैविक और रासायनिक हथियार
3. तलानोआ डायलॉग:वैश्विक जलवायु परिवर्तन
4. अंडर2 गठबंधन:बाल अधिकार


ऊपर दिए गए युग्मों में से कौन-सा/से सही सुमेलित है/हैं?

व्याख्या
  • अल्मा-अता घोषणा: इसे प्राथमिक स्वास्थ्य देखभाल (पीएचसी) पर अंतर्राष्ट्रीय सम्मेलन में अपनाया गया था जो 1978 में अल्माटी, कजाकिस्तान में आयोजित किया गया था। इसने सभी सरकारों, स्वास्थ्य देखभाल कार्यकर्ताओं और विकास कार्यकर्ताओं से सभी के प्राथमिक स्वास्थ्य को बढ़ावा देने और उनकी रक्षा करने का आग्रह किया। लोग। अत: युग्म 1 सही सुमेलित है।
  • हेग कन्वेंशन: विभिन्न विषयों पर हेग कन्वेंशन की एक श्रृंखला है जैसे कि सशस्त्र संघर्ष की स्थिति में सांस्कृतिक संपत्ति के संरक्षण के लिए कन्वेंशन, अंतर्राष्ट्रीय बाल अपहरण के नागरिक पहलुओं पर हेग कन्वेंशन, आदि। लेकिन यह जैविक से संबंधित नहीं है और रसायनिक शस्त्र। अत: युग्म 2 सुमेलित नहीं है।
  • तलानोआ डायलॉग: डायलॉग को 2017 में बॉन (जर्मनी) में संयुक्त राष्ट्र जलवायु परिवर्तन सम्मेलन (सीओपी 23) में लॉन्च किया गया था। तालानोआ एक पारंपरिक शब्द है जिसका इस्तेमाल फिजी और पूरे प्रशांत क्षेत्र में समावेशी, भागीदारी और पारदर्शी संवाद की प्रक्रिया को प्रतिबिंबित करने के लिए किया जाता है। तलानोआ का उद्देश्य कहानियों को साझा करना, सहानुभूति का निर्माण करना और सामूहिक भलाई के लिए बुद्धिमान निर्णय लेना है। अत: युग्म 3 सही सुमेलित है।
  • अंडर 2 गठबंधन: अंडर 2 गठबंधन राज्य और क्षेत्रीय सरकारों का एक वैश्विक समुदाय है जो पेरिस समझौते के अनुरूप महत्वाकांक्षी जलवायु कार्रवाई के लिए प्रतिबद्ध है। गठबंधन 220 से अधिक उपराष्ट्रीय सरकारों को एक साथ लाता है जो 1.3 बिलियन से अधिक लोगों और वैश्विक अर्थव्यवस्था के 43% का प्रतिनिधित्व करते हैं। वर्तमान में, तेलंगाना और छत्तीसगढ़ भारत की ओर से इस समझौते के हस्ताक्षरकर्ता हैं। हस्ताक्षरकर्ता 1.5 डिग्री सेल्सियस तक पहुंचने के प्रयासों के साथ वैश्विक तापमान वृद्धि को 2 डिग्री सेल्सियस से नीचे रखने के लिए प्रतिबद्ध हैं। अत: युग्म 4 सुमेलित नहीं है।

इसलिए, विकल्प (सी) सही उत्तर है।

41

भारत के इतिहास के संदर्भ में निम्नलिखित युग्मों पर विचार कीजिएः

1. औरंग-राज्य के खजाने का प्रभारी
2. बनियां-ईस्ट इंडिया कंपनी के भारतीय एजेंट
3. मिरासीदार-राज्य को नामित राजस्व दाता


ऊपर दिए गए युग्मों में से कौन-सा/से सही सुमेलित है/हैं?

व्याख्या
  • औरंग: यह एक वेयरहाउस के लिए फ़ारसी शब्द है जहां बेचने से पहले माल एकत्र किया जाता है। अत: युग्म 1 सुमेलित नहीं है।
  • बनियन: बनिया (वानिया भी) शब्द संस्कृत के वनिज से लिया गया है, जिसका अर्थ है 'व्यापारी'। इस शब्द का व्यापक रूप से भारत के पारंपरिक व्यापारिक या व्यापारिक जातियों के सदस्यों की पहचान करने के लिए उपयोग किया जाता है। बनिया बैंकर, साहूकार, व्यापारी और दुकानदार थे। ब्रिटिश शासन के दौरान, बनियन ने बंगाल में यूरोपीय व्यापारियों के लिए बिचौलियों के रूप में काम किया। अत: युग्म 2 सही सुमेलित है।
  • मिरासीदार: दक्षिणी भारत में रैयतवारी बंदोबस्त प्रणाली के तहत, ईस्ट इंडिया कंपनी ने मीरासीदारों को भूमि के एकमात्र मालिक के रूप में मान्यता दी, किरायेदारों के अधिकारों को पूरी तरह से खारिज कर दिया। मीरासीदारों को अपनी जमीन बेचने से कानून द्वारा प्रतिबंधित किया गया था, लेकिन इसकी खेती के बदले में इसे किराएदारों को मामूली शुल्क पर पट्टे पर दिया जा सकता है। अत: युग्म 3 सही सुमेलित है।

अतः विकल्प (ब) सही उत्तर है।

42

भारत के धार्मिक इतिहास के संदर्भ में निम्नलिखित कथनों पर विचार कीजिएः

  1. स्थवीरवादिन महायान बौद्ध धर्म के हैं।
  2. लोकोत्तरवादिन संप्रदाय बौद्ध धर्म के महासंघिका संप्रदाय की एक शाखा थी।
  3. महासंघिकों द्वारा बुद्ध के विचलन ने महायान बौद्ध धर्म को बढ़ावा दिया।

ऊपर दिए गए कथनों में से कौन-सा/से सही है/हैं?

व्याख्या
  • महासंघिका, जिसका अर्थ है ग्रेट मण्डली, विचार का एक प्रारंभिक बौद्ध स्कूल था, जो दूसरी बौद्ध परिषद (383 ईसा पूर्व) के बाद बना था, जब यह मठवासी प्रथाओं में मतभेदों के कारण खुद को एक और स्कूल, स्थविरवाद (बुजुर्गों का संप्रदाय) से अलग कर दिया (जिसे जाना जाता है) विनय)।
  • परिषद में, स्थवीरवाद ने भिक्षुओं के लिए आचरण के अधिक कठोर नियमों पर जोर दिया, लेकिन उनके सुझावों को परिषद में बहुमत से खारिज कर दिया गया, जिन्होंने तब खुद को "महासंघिक" कहा।
  • बुद्ध और अर्हत (संत) की प्रकृति पर महासंघिकों के विचारों ने बौद्ध धर्म के महायान रूप के अग्रदूत के रूप में कार्य किया। अत: कथन 1 सही नहीं है, जबकि कथन 3 सही है।
  • अगली सात शताब्दियों में महासंघिकों के और उप-विभाजनों में लोकोत्तरवादिन, एकव्यवाहरिका और कौक्कुटिक शामिल थे। अत: कथन 2 सही है।

अतः विकल्प (ब) सही उत्तर है।

43

निम्नलिखित में से कौन सा कथन उन्नीसवीं शताब्दी के पूर्वार्द्ध के दौरान भारत पर औद्योगिक क्रांति के प्रभाव की सही व्याख्या करता है?

व्याख्या
  • औद्योगिक क्रांति ने भारतीय समाज के लिए गंभीर परिणाम लाए। ब्रिटिश शासन के सबसे महत्वपूर्ण परिणामों में से एक शहरी और ग्रामीण हस्तशिल्प उद्योगों का पतन और विनाश था।
  • वर्ष 1815 से भाप शक्ति की खोज और इसके अभ्यास ने भारतीय कपड़ा उद्योग के लिए एक खतरा पैदा कर दिया। कताई खच्चर और पावरलूम के आविष्कार भाप की शक्ति के कारण अधिक कुशल और प्रभावी हो गए। नतीजतन, इसने ब्रिटिश वस्त्रों की लागत को बहुत कम कर दिया और इसे अंतरराष्ट्रीय स्तर पर प्रतिस्पर्धी बना दिया।
  • ब्रिटिश कपड़ा उद्योग का समर्थन करने के लिए, भारतीय वस्त्रों के निर्यात पर भारी शुल्क लगाया गया था। भारतीय किसानों को कपास का उत्पादन करने के लिए मजबूर किया गया ताकि वह अंग्रेजी कारखानों को बढ़ावा दे सके।
  • हालाँकि, औद्योगिक क्रांति के कुछ सकारात्मक प्रभाव भी थे। उदाहरण के लिए,
    • कारखानों और असेंबली लाइनों की शुरूआत,
    • विद्युत विकास,
    • रेलमार्ग (भारत में पहला रेलवे: 1853 में बॉम्बे से ठाणे तक)
  • इन सभी चीजों ने वस्तुओं और सामग्रियों के तेजी से और अधिक कुशल उत्पादन में योगदान दिया। लेकिन ये विकास मुख्य रूप से 19 वीं शताब्दी के उत्तरार्ध में हुए।

इसलिए, विकल्प (ए) सही उत्तर है।

44

भारत के इतिहास में निम्नलिखित घटनाओं पर विचार करें:

  1. राजा भोज के अधीन प्रतिहारों का उदय
  2. महेन्द्रवर्मन प्रथम के अधीन पल्लव सत्ता की स्थापना
  3. परान्तक प्रथम द्वारा चोल शक्ति की स्थापना
  4. पाल वंश की स्थापना गोपाल ने की थी

प्राचीन काल से आरंभ करके उपरोक्त घटनाओं का सही कालानुक्रमिक क्रम क्या है?

व्याख्या
  • पल्लव वंश 275 सीई से 897 सीई तक अस्तित्व में था, जो दक्षिणी भारत के एक हिस्से पर शासन करता था। महेंद्रवर्मन प्रथम (571-630 सीई) के शासनकाल के दौरान पल्लव एक प्रमुख शक्ति बन गए, जिन्होंने वर्तमान आंध्र क्षेत्र के दक्षिणी हिस्से और वर्तमान तमिलनाडु के उत्तरी क्षेत्रों पर शासन किया।
  • पाल वंश ने 8वीं से 12 वीं शताब्दी तक बिहार और बंगाल में शासन किया । इसके संस्थापक, गोपाल (750-770 सीई), एक स्थानीय सरदार थे जो अराजकता की अवधि के दौरान आठवीं शताब्दी के मध्य में सत्ता में आए थे।
  • आठवीं शताब्दी के मध्य से मध्यदेश पर प्रभुत्व राजस्थान में आदिवासी लोगों के बीच दो विशेष कुलों की महत्वाकांक्षा बन गया, जिन्हें गुर्जर और प्रतिहार के नाम से जाना जाता है। 851 सीई के एक समकालीन अरब खाते के अनुसार, राजा मिहिर भोज (840-851 सीई), प्रतिहार राजाओं में सबसे महान, भारत के उन राजकुमारों में से थे जिन्होंने अरब आक्रमणकारियों के खिलाफ लड़ाई लड़ी थी।
  • चोल साम्राज्य की स्थापना विजयालय ने की थी। चोलों का शासन 9 वीं शताब्दी में शुरू हुआ जब उन्होंने सत्ता में आने के लिए पल्लवों को हराया। मध्ययुगीन काल चोलों के लिए पूर्ण शक्ति और विकास का युग था। परान्तक प्रथम (907-953) ने राज्य की नींव रखी। वह उत्तरी सीमा को नेल्लोर (आंध्र प्रदेश) तक ले गया, जहाँ राष्ट्रकूट राजा कृष्ण III के हाथों हार के कारण उसकी उन्नति रुक ​​गई। परान्तक दक्षिण में अधिक सफल रहा, जहाँ उसने पांड्य और गंगा दोनों को हराया।

इसलिए, विकल्प (सी) सही उत्तर है।

45

निम्नलिखित में से कौन सा वाक्यांश 'हुंडी' की प्रकृति को परिभाषित करता है जिसे आमतौर पर हर्ष के बाद के स्रोतों में संदर्भित किया जाता है?

व्याख्या
  • हुंडिका या हुंडी मध्यकालीन भारत में व्यापार और क्रेडिट लेनदेन में उपयोग के लिए एक वित्तीय प्रणाली के रूप में विकसित विनिमय का एक बिल था।
  • इसने देश के एक हिस्से से दूसरे हिस्से में पैसे के आसान संचरण की सुविधा प्रदान की।
  • हुंडियों के माध्यम से, व्यापारियों ने क्रेडिट बनाया जो प्रचलन में धन को पूरक करता था और वाणिज्य, विशेष रूप से लंबी दूरी और अंतर्राष्ट्रीय व्यापार को वित्तपोषित करता था।
  • सर्राफ (श्रॉफ) जो पैसे बदलने में माहिर थे, हुंडियों से निपटने में भी माहिर थे।

इसलिए, विकल्प (सी) सही उत्तर है।

46

स्वतंत्रता संग्राम के दौरान सखाराम गणेश देउस्कर द्वारा लिखित पुस्तक 'देशेर कथा' के संदर्भ में निम्नलिखित कथनों पर विचार कीजिएः

  1. इसने औपनिवेशिक राज्य की मन की कृत्रिम निद्रावस्था की विजय के खिलाफ चेतावनी दी।
  2. इसने स्वदेशी नुक्कड़ नाटकों और लोक गीतों के प्रदर्शन को प्रेरित किया।
  3. देउस्कर द्वारा 'देश' का प्रयोग बंगाल क्षेत्र के विशिष्ट सन्दर्भ में किया गया था।

ऊपर दिए गए कथनों में से कौन सा सही है?

व्याख्या
  • श्री अरबिंदो के एक करीबी सहयोगी सखाराम गणेश देउस्कर (1869-1912) ने 1904 में 'देशेर कथा' नामक एक पुस्तक प्रकाशित की। इस पुस्तक में भारत के ब्रिटिश वाणिज्यिक और औद्योगिक शोषण का विस्तृत विवरण दिया गया है। 1910 में बंगाल सरकार ने इस पुस्तक पर प्रतिबंध लगा दिया और सभी प्रतियां जब्त कर लीं।
  • इस पुस्तक का बंगाल में अत्यधिक प्रभाव पड़ा, इसने युवा बंगाल के मन पर कब्जा कर लिया और स्वदेशी आंदोलन की तैयारी में किसी भी चीज़ से अधिक सहायता की। अत: कथन 1 और 2 सही हैं।
  • देउस्कर ने अपनी पुस्तक में 'देश' का प्रयोग पूरे देश के सन्दर्भ में किया है। अत: कथन 3 सही नहीं है।

इसलिए, विकल्प (ए) सही उत्तर है।

47

गांधी-इरविन समझौते में निम्नलिखित में से क्या शामिल था?

  1. गोलमेज सम्मेलन में भाग लेने के लिए कांग्रेस को निमंत्रण
  2. सविनय अवज्ञा आंदोलन के संबंध में प्रख्यापित अध्यादेशों को वापस लेना
  3. पुलिस ज्यादतियों की जांच के लिए गांधीजी के सुझाव की स्वीकृति
  4. केवल उन कैदियों की रिहाई जिन पर हिंसा का आरोप नहीं लगाया गया था

नीचे दिए गए कूट का प्रयोग कर सही उत्तर चुनिए:

व्याख्या
  • दिल्ली पैक्ट (5 मार्च 1931), जिसे गांधी-इरविन पैक्ट के नाम से भी जाना जाता है, ने कांग्रेस को सरकार के बराबरी पर ला खड़ा किया।
  • लॉर्ड इरविन, वायसराय और भारत के गवर्नर-जनरल (1926-1931), ब्रिटिश सरकार की ओर से, निम्नलिखित पर महात्मा गांधी से सहमत थे:
    • हिंसा के दोषी नहीं पाए गए सभी राजनीतिक बंदियों की तत्काल रिहाई;
    • अभी तक एकत्र नहीं किए गए सभी जुर्माने की छूट;
    • सभी भूमि की वापसी जो अभी तक तीसरे पक्ष को नहीं बेची गई है;
    • उन सरकारी सेवकों के प्रति उदार व्यवहार जिन्होंने इस्तीफा दे दिया था;
    • तटीय गांवों में निजी उपभोग के लिए नमक बनाने का अधिकार (बिक्री के लिए नहीं);
    • शांतिपूर्ण और गैर-आक्रामक धरना का अधिकार; तथा
    • आपातकालीन अध्यादेशों को वापस लेना।

अत: कथन 2 और 4 सही हैं।

  • हालाँकि, वायसराय ने गांधीजी की दो मांगों को ठुकरा दिया:
    • पुलिस ज्यादतियों की सार्वजनिक जांच, और
    • भगत सिंह और उनके साथियों की मौत की सजा को उम्रकैद में बदला।

अत: कथन 3 सही नहीं है।

  • कांग्रेस की ओर से गांधीजी सहमत हुए:
    • सविनय अवज्ञा आंदोलन को निलंबित करने के लिए, और
    • अगले गोलमेज सम्मेलन में भाग लेने के लिए (सम्मेलन का पहला दौर नवंबर 1930 से जनवरी 1931 तक पहले ही आयोजित किया गया था) संघ के तीन लिंचपिन, भारतीय जिम्मेदारी, और आरक्षण और सुरक्षा उपायों के आसपास संवैधानिक प्रश्न पर जो भारत के हित में आवश्यक हो सकते हैं ( रक्षा, विदेश मामलों, अल्पसंख्यकों की स्थिति, भारत के वित्तीय ऋण और अन्य दायित्वों के निर्वहन जैसे क्षेत्रों को कवर करना)।

अत: कथन 1 सही है।

अतः विकल्प (ब) सही उत्तर है।

48

अछूत लोगों को लक्षित दर्शकों के रूप में रखने वाली पहली मासिक पत्रिका वाइटल-विद्वान्सक किसके द्वारा प्रकाशित की गई थी?

व्याख्या
  • गोपाल बाबा वालांगकर (1840-1900), जिन्हें गोपाल कृष्ण के नाम से भी जाना जाता है, का जन्म महाराष्ट्र के रायगढ़ जिले में अछूत महार जाति के एक परिवार में हुआ था।
  • वह महाराष्ट्र में महारों के अधिकारों के लिए लड़ने वाले पहले व्यक्ति थे।
  • वह 1886 में सैन्य सेवा से सेवानिवृत्त हुए और लोगों को लामबंद किया और उन्हें उनके मानवाधिकारों के प्रति जागरूक किया।
  • 1888 में, वालंगकर ने वाइटल-विधवंशक (ब्राह्मणवादी या औपचारिक प्रदूषण का विनाशक) नामक मासिक पत्रिका प्रकाशित करना शुरू किया, जो अछूत लोगों को अपने लक्षित दर्शकों के रूप में रखने वाला पहला था।

इसलिए, विकल्प (ए) सही उत्तर है।

49

भारत के इतिहास के संदर्भ में, "कुल्यवपा" और "द्रोणवपा" शब्द निरूपित करते हैं

व्याख्या
  • गुप्त युग में, कुल्यवपा, द्रोणवपा और अधवपा शब्द मुख्य रूप से बंगाल में पाए गए तांबे की प्लेट शिलालेखों में वर्णित भूमि माप से संबंधित थे।
  • ये शब्द भूमि के उस क्षेत्र को इंगित करते हैं जो क्रमशः एक कुल, द्रोण और अधक के वजन के बीज अनाज बोने के लिए आवश्यक था। यौगिक "कुल्यवप" में "वाप" तत्व, "वाप" मूल से व्युत्पन्न है, जिसका अर्थ है बोने (फेंकने या बिखरने) की क्रिया।
  • सुभाकीर्ति की मिदनापुर प्लेट में दम्यस्वामी नाम के एक ब्राह्मण को कुम्भरापदराका गाँव में 40 द्रोण भूमि और एक द्रोणवपा रियासत भूमि के उपहार का उल्लेख है।
  • बंगाल के शिलालेखों में द्रोणवप के साथ-साथ इसके अन्य प्रभागों और गुणकों जैसे कि अध्वप और कुल्यवपा का भी उपयोग किया गया था।
  • संस्कृत के शब्दकोषों के अनुसार, एक द्रोणवपा कुल्यवप के एक-आठ के बराबर होता था और इसकी पुष्टि पुरालेखीय साक्ष्यों से भी होती है।

इसलिए, विकल्प (ए) सही उत्तर है।

50

निम्नलिखित में से किस शासक ने इस शिलालेख के माध्यम से अपनी प्रजा को सलाह दी थी?

"जो कोई भी अपने संप्रदाय की प्रशंसा करता है या अपने संप्रदाय की अत्यधिक भक्ति से अन्य संप्रदायों को दोष देता है, अपने संप्रदाय की महिमा करने की दृष्टि से, वह अपने ही संप्रदाय को बहुत गंभीर रूप से घायल करता है।"

व्याख्या
  • अपने आदेश XII में, अशोक ने किसी और की कीमत पर अपने धर्म को ऊपर उठाने की प्रथा की निंदा की:
    • आवश्यक वस्तुओं की वृद्धि अलग-अलग तरीकों से की जा सकती है, लेकिन उन सभी का मूल वाणी में संयम है, अर्थात अपने धर्म की प्रशंसा नहीं करना, या बिना किसी अच्छे कारण के दूसरों के धर्म की निंदा करना। और अगर आलोचना का कारण है, तो उसे हल्के ढंग से करना चाहिए। लेकिन इस कारण से दूसरे धर्मों का सम्मान करना बेहतर है।
    • ऐसा करने से अपने धर्म को लाभ होता है और दूसरे धर्मों को भी, जबकि ऐसा करने से अपने धर्म और दूसरों के धर्मों का नुकसान होता है।
    • जो व्यक्ति अत्यधिक भक्ति के कारण अपने ही धर्म की प्रशंसा करता है और दूसरों की निंदा इस विचार से करता है कि 'मुझे अपने धर्म की महिमा करने दो', वह अपने ही धर्म को हानि पहुँचाता है। दूसरों द्वारा बताए गए सिद्धांतों को सुनना और उनका सम्मान करना चाहिए।
  • इस आदेश का समापन इस सलाह के साथ होता है कि एक व्यक्ति का धर्म धम्म के माध्यम से बढ़ता है और इसलिए सभी धर्मों को सहिष्णुता और समझ से सुधारा जाता है।

इसलिए, विकल्प (ए) सही उत्तर है।

51

कृषि में फर्टिगेशन के क्या लाभ हैं?

  1. सिंचाई के पानी की क्षारीयता को नियंत्रित करना संभव है।
  2. रॉक फॉस्फेट और अन्य सभी फॉस्फेट उर्वरकों का कुशल अनुप्रयोग संभव है।
  3. पौधों को पोषक तत्वों की उपलब्धता में वृद्धि संभव है।
  4. रासायनिक पोषक तत्वों की लीचिंग में कमी संभव है।

नीचे दिए गए कूट का प्रयोग कर सही उत्तर चुनिए:

व्याख्या
  • सिंचाई के पानी के माध्यम से खेतों में उर्वरकों के साथ फसलों की आपूर्ति करने की प्रथा को फर्टिगेशन कहा जाता है। यह एक आधुनिक कृषि तकनीक है जो उर्वरक उपयोग दक्षता को बढ़ाकर, उर्वरक आवेदन को कम करके और निवेश किए गए उर्वरक पर रिटर्न बढ़ाकर उच्च उपज और कम पर्यावरण प्रदूषण की सुविधा प्रदान करती है।
  • पारंपरिक या ड्रॉप-निषेचन विधियों पर फर्टिगेशन विधियों के लाभों में शामिल हैं:
    • पौधों द्वारा पोषक तत्वों के अवशोषण में वृद्धि।
    • पोषक तत्व का सटीक स्थान, जहां पानी जाता है पोषक तत्व भी जाता है।
    • 'सूक्ष्म खुराक' करने की क्षमता, पौधों को पर्याप्त मात्रा में खिलाना ताकि पोषक तत्वों को अवशोषित किया जा सके और अगली बार बारिश होने पर तूफानी पानी में धुलने के लिए नहीं छोड़ा जा सके।
    • आवश्यक उर्वरक, रसायन और पानी की कमी।
    • पानी की आपूर्ति में रसायनों की कम लीचिंग।
    • पौधे की बढ़ी हुई जड़ द्रव्यमान की पानी को फंसाने और धारण करने की क्षमता के कारण पानी की खपत कम हो जाती है।
    • पोषक तत्वों के अनुप्रयोग को सटीक समय और आवश्यक दर पर नियंत्रित किया जा सकता है।
    • दूषित मिट्टी के माध्यम से जड़ों के सिकुड़ने से मिट्टी जनित रोगों का जोखिम कम होता है।
    • मिट्टी के कटाव के मुद्दों को कम करना क्योंकि पोषक तत्वों को वाटर ड्रिप सिस्टम के माध्यम से पंप किया जाता है। फर्टिगेशन को नियोजित करने के लिए उपयोग की जाने वाली विधियों के माध्यम से अक्सर लीचिंग कम हो जाती है।
    • फर्टिगेशन के जरिए सिंचाई के पानी के पीएच को मैनेज किया जा सकता है।
  • कृषि में फर्टिगेशन विधि की सलाह नहीं दी जाती है यदि उपयोग किया गया पदार्थ अवक्षेप बना सकता है और फॉस्फेट उच्च कैल्शियम और मैग्नीशियम सामग्री वाले पानी में अवक्षेप बना सकता है। अत: कथन 2 सही नहीं है।

इसलिए, विकल्प (सी) सही उत्तर है।

52

निम्नलिखित खनिजों पर विचार करें:

  1. बेंटोनाइट
  2. क्रोमाइट
  3. क्यानाइट
  4. सिलिमनाइट

भारत में, उपरोक्त में से कौन आधिकारिक तौर पर प्रमुख खनिजों के रूप में नामित है/हैं?

व्याख्या
  • राष्ट्रीय लेखा सांख्यिकी (2007) के अनुसार, खनिजों को मोटे तौर पर प्रमुख और लघु खनिजों में वर्गीकृत किया गया है।
  • प्रमुख खनिजों में निम्नलिखित शामिल हैं:
    • ईंधन खनिज: कोयला, लिग्नाइट, प्राकृतिक गैस, पेट्रोलियम (कच्चा)।
    • धात्विक खनिज: बॉक्साइट, क्रोमाइट, तांबा अयस्क, सोना, लौह अयस्क, सीसा (सांद्रता), जस्ता (सांद्रता), मैंगनीज अयस्क, चांदी, टिन (सांद्रता), टंगस्टन (सांद्रता)।
    • गैर-धातु खनिज: एगेट, अंडालूसाइट, एपेटाइट, एस्बेस्टस, बॉल क्ले, बैराइट्स, कैल्साइट, चाक, क्ले, कोरंडम, कैलकेरियस रेत, हीरा, डायस्पोर, डोलोमाइट, कानाइट, लेटराइट, चूना पत्थर, चूना पत्थर कंकर, चूना शैल, मैग्नेसाइट, अभ्रक (कच्चा), गेरू, पाइराइट्स, पायरोफिलाइट, फॉस्फोराइट, क्वार्ट्ज, अशुद्ध क्वार्ट्ज, क्वार्टजाइट, फुचसाइट क्वार्टजाइट, सिलिका सैंड, नमक (रॉक), नमक (वाष्पित), शेल, स्लेट, स्टीटाइट, सिलिमेनाइट, वर्मीक्यूलाइट, वोलास्टोनाइट।
  • माइनर मिनरल्स में बेंटोनाइट, बोल्डर, ब्रिक अर्थ, बिल्डिंग स्टोन्स, चेलेडोनी या कोरंडम, फुलर अर्थ, बजरी, लाइम स्टोन, ड्यूनाइट, फेलस्पर, फायर क्ले, फेलसाइट, फ्लोराइट (ग्रेडेड), फ्लोराइट (कंसेंट्रेट्स), जिप्सम, गार्नेट (एब्रेसिव्स) शामिल हैं। , गार्नेट (रत्न), ग्रेफाइट रन-ऑन-माइन्स, जैस्पर, काओलिन, मार्बल, मुर्रम, साधारण मिट्टी, साधारण रेत, साधारण पृथ्वी, कंकड़ या कंकड़, क्वार्टजाइट और सैंड स्टोन, रोड मेटल, सॉल्ट पेट्रे, शेल, शिंगल, स्लेट .
  • क्रोमाइट, कानाइट और सिलीमेनाइट प्रमुख खनिज हैं, जबकि बेंटोनाइट एक लघु खनिज है।

इसलिए, विकल्प (डी) सही उत्तर है।

53

महासागर माध्य तापमान (OMT) के संदर्भ में, निम्नलिखित में से कौन सा/से कथन सही है/हैं?

  1. ओएमटी को 26ºC इज़ोटेर्म की गहराई तक मापा जाता है जो जनवरी-मार्च के दौरान दक्षिण-पश्चिमी हिंद महासागर में 129 मीटर है।
  2. जनवरी-मार्च के दौरान एकत्र किए गए ओएमटी का उपयोग यह आकलन करने में किया जा सकता है कि मानसून में वर्षा की मात्रा एक निश्चित दीर्घकालिक माध्य से कम या अधिक होगी।

नीचे दिए गए कूट का प्रयोग कर सही उत्तर चुनिए:

व्याख्या
  • पुणे के भारतीय उष्णकटिबंधीय मौसम विज्ञान संस्थान (आईआईटीएम) के वैज्ञानिकों ने पाया है कि ओएमटी में समुद्री सतह के तापमान (एसएसटी) की तुलना में भारतीय ग्रीष्मकालीन मानसून की भविष्यवाणी करने की बेहतर क्षमता है।
  • SST समुद्र की ऊपरी परत के कुछ मिलीमीटर तक ही सीमित है और बड़े पैमाने पर तेज़ हवाओं, वाष्पीकरण या घने बादलों से प्रभावित होता है। इसके विपरीत, ओएमटी, जिसे 26ºC इज़ोटेर्म की गहराई तक मापा जाता है, अधिक स्थिर और सुसंगत है, और स्थानिक फैलाव भी कम है।
  • 26ºC समतापी 50-100 मीटर की गहराई पर देखा जाता है। जनवरी-मार्च के दौरान, दक्षिण-पश्चिमी हिंद महासागर में औसत 26ºC इज़ोटेर्म गहराई 59 मीटर है। अतः कथन 1 सही नहीं है।
  • ओएमटी के साथ, बेहतर भविष्य कहनेवाला सफलता के अलावा, मॉनसून वर्षा की मात्रा लंबी अवधि के औसत से अधिक या कम होगी, इसकी जानकारी दक्षिण-पश्चिम मानसून के आने से दो महीने पहले अप्रैल की शुरुआत तक उपलब्ध होगी। यह है क्योंकि ओएमटी का विश्लेषण जनवरी से मार्च की अवधि के दौरान महासागरीय तापीय ऊर्जा को मापकर किया जाता है। अत: कथन 2 सही है।

अतः विकल्प (ब) सही उत्तर है।

54

भारत में रासायनिक उर्वरकों के संदर्भ में, निम्नलिखित कथनों पर विचार कीजिएः

  1. वर्तमान में, रासायनिक उर्वरकों का खुदरा मूल्य बाजार संचालित है और सरकार द्वारा प्रशासित नहीं है।
  2. अमोनिया, जो यूरिया का एक इनपुट है, प्राकृतिक गैस से उत्पन्न होता है।
  3. सल्फर, जो फॉस्फोरिक एसिड उर्वरक के लिए एक कच्चा माल है, तेल रिफाइनरियों का उप-उत्पाद है।

ऊपर दिए गए कथनों में से कौन-सा/से सही है/हैं?

व्याख्या
  • भारत सरकार उर्वरकों को सब्सिडी देती है ताकि यह सुनिश्चित हो सके कि किसानों को उर्वरक आसानी से उपलब्ध हो और देश कृषि उत्पादन में आत्मनिर्भर बना रहे। यह काफी हद तक उर्वरक की कीमत और उत्पादन की मात्रा को नियंत्रित करके हासिल किया गया है। अतः कथन 1 सही नहीं है।
  • प्राकृतिक गैस से अमोनिया (NH3 ) को संश्लेषित किया गया है। इस प्रक्रिया में, प्राकृतिक गैस के अणु कार्बन और हाइड्रोजन में अपचित हो जाते हैं। फिर हाइड्रोजन को शुद्ध किया जाता है और अमोनिया के उत्पादन के लिए नाइट्रोजन के साथ प्रतिक्रिया की जाती है। इस सिंथेटिक अमोनिया का उपयोग उर्वरक के रूप में किया जाता है, या तो सीधे अमोनिया के रूप में या परोक्ष रूप से यूरिया, अमोनियम नाइट्रेट, और मोनोअमोनियम या डायमोनियम फॉस्फेट के रूप में संश्लेषण के बाद। अत: कथन 2 सही है।
  • सल्फर तेल शोधन और गैस प्रसंस्करण का एक प्रमुख उप-उत्पाद है। अधिकांश कच्चे तेल ग्रेड में कुछ सल्फर होता है, जिनमें से अधिकांश को परिष्कृत उत्पादों में सल्फर सामग्री की सख्त सीमा को पूरा करने के लिए शोधन प्रक्रिया के दौरान हटाया जाना चाहिए। यह आमतौर पर हाइड्रोट्रीटिंग के माध्यम से किया जाता है और इसके परिणामस्वरूप एच 2 एस गैस का उत्पादन होता है, जिसे मौलिक सल्फर में परिवर्तित किया जाता है। सल्फर का खनन भूमिगत, प्राकृतिक रूप से पाए जाने वाले निक्षेपों से भी किया जा सकता है, लेकिन यह तेल और गैस से प्राप्त करने की तुलना में अधिक महंगा है और इसे काफी हद तक बंद कर दिया गया है। सल्फ्यूरिक एसिड का उपयोग मोनोअमोनियम फॉस्फेट (एमएपी) और डायमोनियम फॉस्फेट (डीएपी) दोनों के उत्पादन में किया जाता है। अत: कथन 3 सही है।

अतः विकल्प (ब) सही उत्तर है।

55

भारत के डेजर्ट नेशनल पार्क के संदर्भ में, निम्नलिखित में से कौन सा कथन सही है?

  1. यह दो जिलों में फैला हुआ है।
  2. पार्क के अंदर कोई मानव निवास नहीं है।
  3. यह ग्रेट इंडियन बस्टर्ड के प्राकृतिक आवासों में से एक है।

नीचे दिए गए कूट का प्रयोग कर सही उत्तर चुनिए:

व्याख्या
  • डेजर्ट नेशनल पार्क (DNP), 3162 वर्ग किमी का क्षेत्रफल, राजस्थान के जैसलमेर और बाड़मेर जिलों में फैला हुआ है। यह क्षेत्र देश के बहुत कम वर्षा वाले क्षेत्र (<100 मिमी) के अत्यधिक गर्म और शुष्क क्षेत्र में पड़ता है। डीएनबी को 1980 में सीमांकित और अधिसूचित किया गया था। इसलिए, कथन 1 सही है।
  • थार मरुस्थल दुनिया का सबसे घनी आबादी वाला मरुस्थल है जिसका औसत घनत्व 83 व्यक्ति/किमी² है। हालांकि, डीएनपी के भीतर मानव आबादी कम है (प्रति किमी² 4-5 व्यक्ति)। पार्क के भीतर लगभग 70 गाँव और बस्तियाँ या ढाणी मौजूद हैं। अत: कथन 2 सही नहीं है।
  • थार रेगिस्तान में वनस्पतियों और जीवों की प्रजातियों की एक विस्तृत श्रृंखला है। यह एकमात्र ऐसा स्थान है जहाँ प्राकृतिक रूप से राजस्थान राज्य पक्षी (ग्रेट इंडियन बस्टर्ड) पाया जाता है। अत: कथन 3 सही है।

इसलिए, विकल्प (सी) सही उत्तर है।

56

सियाचिन ग्लेशियर स्थित है

व्याख्या
  • सियाचिन ग्लेशियर हिमालय में पूर्वी काराकोरम रेंज में स्थित है, जो प्वाइंट NJ9842 के उत्तर-पूर्व में है, जहां भारत और पाकिस्तान के बीच नियंत्रण रेखा समाप्त होती है।
  • इसे ध्रुवीय और उपध्रुवीय क्षेत्रों के बाहर सबसे बड़ा हिमनद होने का गौरव प्राप्त है।
  • यह अक्साई चिन के पश्चिम में, नुब्रा घाटी के उत्तर में और गिलगित के लगभग पूर्व में स्थित है।

इसलिए, विकल्प (डी) सही उत्तर है।

57

भारत के इतिहास के संदर्भ में निम्नलिखित युग्मों पर विचार कीजिएः

प्रसिद्ध स्थलवर्तमान स्थिति
1. भीलसामध्य प्रदेश
2. द्वारसमुद्र:महाराष्ट्र
3. गिरिनगरगुजरात
4. स्थानेश्वर:उतार प्रदेश


ऊपर दिए गए युग्मों में से कौन-सा सही सुमेलित है?

व्याख्या
  • विदिशा (पहले भेलसा के नाम से जाना जाता था और प्राचीन काल में बेसनगर के नाम से जाना जाता था) मध्य प्रदेश राज्य का एक शहर है। यह राज्य की राजधानी भोपाल से लगभग 60 किमी उत्तर पूर्व में स्थित है। अत: युग्म 1 सही सुमेलित है।
  • हलेबिदु, जिसे पहले दोरासमुद्र या द्वारसमुद्र के नाम से जाना जाता था, प्राचीन होयसल राजधानी, अलंकृत होयसलेश्वर और केदारेश्वर मंदिर हैं, और यह कर्नाटक राज्य के पर्यटन आकर्षणों में से एक है। अत: युग्म 2 सुमेलित नहीं है।
  • गिरनार, जिसे गिरिनगर (शहर-पर-पहाड़ी) या रेवतक पर्वत के नाम से भी जाना जाता है, गुजरात के जूनागढ़ जिले में पहाड़ों का एक समूह है। यह स्थान जैनियों के लिए पवित्र है क्योंकि यह वह स्थान है जहां भगवान निमिनाथ मोक्ष प्राप्त करने गए थे। अत: युग्म 3 सही सुमेलित है।
  • थानेसर या स्थानेश्वर, एक ऐतिहासिक शहर, जो अब हरियाणा में नव निर्मित कुरुक्षेत्र शहर के निकट स्थित है। अत: युग्म 4 सुमेलित नहीं है।

इसलिए, विकल्प (ए) सही उत्तर है।

58

निम्नलिखित कथनों पर विचार करें:

  1. भारत के 36 प्रतिशत जिलों को केंद्रीय भूजल प्राधिकरण (सीजीडब्ल्यूए) द्वारा "अतिशोषित" या "गंभीर" के रूप में वर्गीकृत किया गया है।
  2. सीजीडब्ल्यूए का गठन पर्यावरण (संरक्षण) अधिनियम के तहत किया गया था।
  3. भारत में दुनिया में भूजल सिंचाई के तहत सबसे बड़ा क्षेत्र है।

ऊपर दिए गए कथनों में से कौन-सा/से सही है/हैं।

व्याख्या
  • भूजल स्तर के आधार पर, देश भर के क्षेत्रों को तीन श्रेणियों में बांटा गया है: अति-शोषित, गंभीर और अर्ध-महत्वपूर्ण। पहला यह है कि भूजल को पुनर्भरण से अधिक निकाला जा रहा है अर्थात निष्कर्षण 100% से अधिक है। गंभीर जहां निकाला गया भूजल रिचार्ज किए गए पानी का 90-100% है और अर्ध-महत्वपूर्ण है जहां निष्कर्षण दर 70% -90% है।
  • सीजीडब्ल्यूए की रिपोर्ट 'भारत के गतिशील भूजल संसाधनों पर राष्ट्रीय संकलन, 2017' के अनुसार, देश में कुल 6881 मूल्यांकन इकाइयों (ब्लॉकों/मंडलों/तालुकाओं) में से, विभिन्न राज्यों (17%) में 1186 इकाइयों को वर्गीकृत किया गया है। 'अति-शोषित', 313 इकाइयाँ (5%) 'गंभीर' हैं, और 972 अर्ध-महत्वपूर्ण इकाइयाँ (14%) हैं। अत: कथन 1 सही नहीं है।
  • केंद्रीय भूजल प्राधिकरण (सीजीडब्ल्यूए) का गठन पर्यावरण (संरक्षण) अधिनियम, 1986 की धारा 3 (3) के तहत भूजल संसाधनों के विकास और प्रबंधन को विनियमित करने, नियंत्रित करने के लिए किया गया था। अत: कथन 2 सही है।
  • संयुक्त राष्ट्र के खाद्य और कृषि संगठन (एफएओ) की रिपोर्ट के अनुसार, भारत (39 मिलियन हेक्टेयर), चीन (19 मिलियन हेक्टेयर) और यू.एस.ए. 17 मिलियन हेक्टेयर)। अत: कथन 3 सही है।

अतः विकल्प (ब) सही उत्तर है।

59

निम्नलिखित कथनों पर विचार करें:

  1. जेट धाराएँ उत्तरी गोलार्ध में ही होती हैं।
  2. केवल कुछ चक्रवात ही नेत्र विकसित करते हैं।
  3. चक्रवात की आंख के अंदर का तापमान परिवेश की तुलना में लगभग 10°C कम होता है।

ऊपर दिए गए कथनों में से कौन-सा/से सही है/हैं?

व्याख्या
  • जेट स्ट्रीम एक भूस्थैतिक हवा है जो क्षोभमंडल की ऊपरी परतों के माध्यम से क्षैतिज रूप से बहती है, आमतौर पर पश्चिम से पूर्व की ओर, 20,000-50,000 फीट की ऊंचाई पर। जेट स्ट्रीम विकसित होती हैं जहां विभिन्न तापमानों के वायु द्रव्यमान मिलते हैं। इसलिए, आमतौर पर सतह का तापमान निर्धारित करता है कि जेट स्ट्रीम कहाँ बनेगी। तापमान में जितना अधिक अंतर होता है, जेट स्ट्रीम के अंदर हवा का वेग उतना ही तेज होता है। जेट धाराएँ 20° अक्षांश से दोनों गोलार्द्धों के ध्रुवों तक फैली हुई हैं। अतः कथन 1 सही नहीं है।
  • चक्रवात दो प्रकार के होते हैं, उष्ण कटिबंधीय चक्रवात और शीतोष्ण चक्रवात। उष्णकटिबंधीय चक्रवात के केंद्र को 'आंख' के रूप में जाना जाता है, जहां केंद्र में हवा शांत होती है और वर्षा नहीं होती है। हालांकि, समशीतोष्ण चक्रवात में एक भी स्थान ऐसा नहीं है जहां हवाएं और बारिश निष्क्रिय हो, इसलिए आंख नहीं मिलती है। अत: कथन 2 सही है।
  • सबसे गर्म तापमान आंखों में ही पाया जाता है, न कि आईवॉल बादलों में जहां गुप्त गर्मी होती है। हवा केवल वहीं संतृप्त होती है जहां संवहन ऊर्ध्वाधर गति उड़ान स्तर से गुजरती है। आंख के अंदर का तापमान 28 डिग्री सेल्सियस से अधिक होता है और ओस बिंदु 0 डिग्री सेल्सियस से कम होता है। ये गर्म और शुष्क स्थितियां अत्यंत तीव्र उष्णकटिबंधीय चक्रवातों की आंखों के लिए विशिष्ट हैं। अत: कथन 3 सही नहीं है।

इसलिए, विकल्प (सी) सही उत्तर है।

60

निम्नलिखित टाइगर रिजर्व में से, "क्रिटिकल टाइगर हैबिटेट" के तहत सबसे बड़ा क्षेत्र कौन सा है?

व्याख्या
  • क्रिटिकल टाइगर हैबिटेट्स (सीटीएच), जिन्हें टाइगर रिजर्व के मुख्य क्षेत्रों के रूप में भी जाना जाता है, की पहचान वन्य जीवन संरक्षण अधिनियम, 1972 के तहत वैज्ञानिक प्रमाणों के आधार पर की जाती है कि "ऐसे क्षेत्रों को बाघ संरक्षण के उद्देश्य से उल्लंघनकारी के रूप में रखा जाना आवश्यक है, बिना प्रभावित किए। अनुसूचित जनजातियों या ऐसे अन्य वनवासियों के अधिकार ”।
  • सीटीएच को राज्य सरकार द्वारा इस उद्देश्य के लिए गठित विशेषज्ञ समिति के परामर्श से अधिसूचित किया जाता है।
  • मुख्य/महत्वपूर्ण बाघ आवास का क्षेत्र:
    • कॉर्बेट (उत्तराखंड): 821.99 वर्ग किमी
    • रणथंभौर (राजस्थान): 1113.36 वर्ग किमी
    • सुंदरवन (पश्चिम बंगाल): 1699.62 वर्ग किमी
    • नागार्जुनसागर श्रीशैलम (आंध्र प्रदेश का हिस्सा): 2595.72 वर्ग किमी

इसलिए, विकल्प (सी) सही उत्तर है।

61

यदि किसी विशेष पौधे की प्रजाति को वन्यजीव संरक्षण अधिनियम, 1972 की अनुसूची VI के तहत रखा गया है, तो इसका क्या निहितार्थ है?

व्याख्या
  • वन्य जीव संरक्षण अधिनियम, 1972 पौधों और जानवरों की प्रजातियों के संरक्षण के लिए अधिनियमित किया गया है। अधिनियम में जंगली जानवरों, पक्षियों और पौधों की सुरक्षा का प्रावधान है। इसमें छह अनुसूचियां हैं जो अलग-अलग स्तर की सुरक्षा प्रदान करती हैं।
  • अनुसूची I और अनुसूची II का भाग II पूर्ण सुरक्षा प्रदान करता है - इनके तहत अपराध उच्चतम दंड निर्धारित हैं।
  • अनुसूची III और अनुसूची IV में सूचीबद्ध प्रजातियां भी संरक्षित हैं, लेकिन दंड बहुत कम हैं।
  • अनुसूची V में वे जानवर शामिल हैं जिनका शिकार किया जा सकता है।
  • अनुसूची VI में निर्दिष्ट स्थानिक पौधों को खेती और रोपण से प्रतिबंधित किया गया है।
  • अनुसूची VI में निम्नलिखित पौधों को शामिल किया गया है:
    • बेडडोम्स साइकैड (साइकस बेडडोमी) ,
    • ब्लू वांडा (वांडा सोरुलेक) ,
    • कुथ (सौसुरिया लप्पा) ,
    • लेडीज़ स्लिपर ऑर्किड (पैपिओपेडिलम एसपीपी।) ,
    • पिचर प्लांट (नेपेंथेस खासियाना) ,
    • लाल वंदा
  • हालांकि, आगे यह भी कहा गया है कि बिना लाइसेंस के निर्दिष्ट पौधों की खेती प्रतिबंधित है। अधिनियम की धारा 17सी के अनुसार कोई भी व्यक्ति मुख्य वन्य जीव वार्डन या राज्य सरकार द्वारा इस संबंध में अधिकृत किसी अन्य अधिकारी द्वारा दिए गए लाइसेंस के तहत और उसके अनुसार के अलावा किसी निर्दिष्ट पौधे की खेती नहीं करेगा।

इसलिए, विकल्प (ए) सही उत्तर है।

62

प्राचीन भारत में गुप्त वंश के काल के सन्दर्भ में घंटाशाला, कदुरा और चौल नगरों को किस नाम से जाना जाता था?

व्याख्या
  • प्राचीन काल के दौरान, भारत का मिस्र, रोम, यूनानियों, अरबों, चीन और लगभग सभी दक्षिण पूर्व एशियाई देशों के साथ व्यापार और सांस्कृतिक संपर्क था।
  • भारत के दोनों तट बंदरगाहों की एक श्रृंखला से जुड़े हुए हैं।
    • पश्चिमी तट पर बंदरगाह थे बरीगया, सुप्पारा, कैलिएना, सेमिला, मंडागोर, पालेपटमे, मालिज़िगारा, और्रोनोबास, नौरा, टिंडिस, मुज़िरिस और नेल्सिंडा।
    • पूर्वी तट पर बंदरगाह थे ताम्रलिप्ति, चरित्रपुर, पलुरु, दंतापुर, कलिंगपट्टनम, पिथुंडा, सोपटमा, घंटाशाला, कडुरा, पोडुका, पुहार, कोरकाई और कैमारा।
  • पश्चिमी तट पर मालवान, सोपारा, एलीफेंटा द्वीप, चौल, उदयवारा, होन्नावर, गोपाकपटना और भारत के पूर्वी तट पर पूमपुहार और ट्रांक्यूबार के आसपास के क्षेत्र मानिकपटना, कलिंगपटना में तटवर्ती अन्वेषण किए गए हैं।

इसलिए, विकल्प (ए) सही उत्तर है।

63

कृषि में शून्य जुताई के क्या/क्या लाभ हैं/हैं?

  1. पिछली फसल के अवशेषों को जलाए बिना गेहूं की बुवाई संभव है।
  2. धान के पौधों की नर्सरी की आवश्यकता के बिना गीली मिट्टी में धान के बीज की सीधी बुवाई संभव है।
  3. मिट्टी में कार्बन ज़ब्ती संभव है।

नीचे दिए गए कूट का प्रयोग कर सही उत्तर चुनिए:

व्याख्या
  • ज़ीरो टिलेज, जिसे नो-टिल फार्मिंग भी कहा जाता है, एक खेती की तकनीक है जिसमें मिट्टी को केवल भट्ठा के साथ या उस छेद में परेशान किया जाता है जिसमें बीज लगाए जाते हैं, पिछली फसलों से आरक्षित डिट्रिटस बीज को कवर और संरक्षित करता है।
  • एक अध्ययन के अनुसार, यह पाया गया है कि उत्तर भारत में किसान न केवल वायु प्रदूषण को कम करने में मदद कर सकते हैं, बल्कि अपनी मिट्टी की उत्पादकता में भी सुधार कर सकते हैं और अधिक लाभ कमा सकते हैं यदि वे अपने फसल अवशेषों को जलाना बंद कर दें और इसके बजाय नो-टिल खेती की अवधारणा को अपनाएं। .
  • जीरो टिलेज के तहत बिना जुताई वाली मिट्टी में गेहूं की सीधी बिजाई और चावल के अवशेषों को छोड़कर बहुत फायदेमंद साबित हुआ है। इसने पानी, श्रम और कृषि-रसायनों के उपयोग को बचाया, ग्रीनहाउस गैस उत्सर्जन को कम किया, और मिट्टी के स्वास्थ्य और फसल की उपज में सुधार किया और इस तरह किसानों और समाज दोनों को बड़े पैमाने पर लाभ हुआ। अत: कथन 1 सही है।
  • अनुत्पादक जल प्रवाह को कम करने के लिए प्रत्यक्ष बीज चावल (डीएसआर) एक व्यवहार्य विकल्प है। डीएसआर का तात्पर्य नर्सरी से रोपाई के बजाय खेत में बोए गए बीजों से चावल की फसल को स्थापित करने की प्रक्रिया से है। पारंपरिक चावल स्थापना प्रणाली के लिए पर्याप्त मात्रा में पानी की आवश्यकता होती है। यह बताया गया है कि 1 किलो कच्चे चावल के उत्पादन के लिए 5000 लीटर तक पानी का उपयोग किया जाता है। हालांकि, पानी की बढ़ती कमी के साथ, न्यूनतम या शून्य जुताई के साथ शुष्क-डीएसआर श्रम की बचत करके इस तकनीक के लाभों को और बढ़ाता है। अत: कथन 2 सही है।
  • कोई भी जुताई वाली मिट्टी दूसरों की तुलना में ठंडी नहीं होती है, आंशिक रूप से क्योंकि पौधे के अवशेषों की एक सतह परत मौजूद होती है। मिट्टी में कार्बन जमा हो जाता है, जिससे इसकी गुणवत्ता में वृद्धि होती है, जिससे ग्लोबल वार्मिंग का खतरा कम होता है। अत: कथन 3 सही है।

इसलिए, विकल्प (डी) सही उत्तर है।

64

जैव ईंधन पर भारत की राष्ट्रीय नीति के अनुसार जैव ईंधन के उत्पादन के लिए निम्नलिखित में से किसका उपयोग कच्चे माल के रूप में किया जा सकता है?

  1. कसावा
  2. क्षतिग्रस्त गेहूं के दाने
  3. मूंगफली के बीज
  4. चने की दाल
  5. सड़े हुए आलू
  6. मीठे चुक़ंदर

नीचे दिए गए कूट का प्रयोग कर सही उत्तर चुनिए:

व्याख्या
  • जैव ईंधन पर राष्ट्रीय नीति, 2018, क्षतिग्रस्त खाद्यान्न जैसे गेहूं, टूटे चावल, आदि से इथेनॉल के उत्पादन की अनुमति देती है, जो मानव उपभोग के लिए अनुपयुक्त हैं।
  • यह नीति राष्ट्रीय जैव ईंधन समन्वय समिति के अनुमोदन के आधार पर खाद्यान्न की अधिशेष मात्रा को इथेनॉल में बदलने की भी अनुमति देती है।
  • यह नीति गन्ने के रस, चीनी युक्त सामग्री जैसे चुकंदर, मीठा चारा, स्टार्च युक्त सामग्री जैसे मकई, कसावा, गेहूं, टूटे चावल, सड़े हुए आलू, के लिए अनुपयुक्त अनाज के उपयोग की अनुमति देकर इथेनॉल उत्पादन के लिए कच्चे माल के दायरे का विस्तार करती है। इथेनॉल उत्पादन के लिए मानव उपभोग।

इसलिए, विकल्प (ए) सही उत्तर है।

65

निम्नलिखित में से कौन सा कथन 'कार्बन की सामाजिक लागत' शब्द का सबसे अच्छा वर्णन करता है?

यह एक उपाय है, मौद्रिक मूल्य में,

व्याख्या
  • कार्बन की सामाजिक लागत (एससीसी) डॉलर में एक अनुमान है, जो कि वातावरण में एक अतिरिक्त टन ग्रीनहाउस गैसों के उत्सर्जन के परिणामस्वरूप होने वाले आर्थिक नुकसान का अनुमान है।
  • SCC नीति निर्माताओं और अन्य निर्णय निर्माताओं को उन निर्णयों के आर्थिक प्रभावों को समझने में मदद करने के लिए जलवायु परिवर्तन के प्रभावों को आर्थिक संदर्भ में रखता है जो उत्सर्जन में वृद्धि या कमी करेंगे।
  • कार्बन उत्सर्जन की भारत की देश-स्तरीय सामाजिक लागत $86 प्रति टन CO2 के उच्चतम होने का अनुमान लगाया गया था । इसका मतलब है कि प्रत्येक अतिरिक्त टन CO2 उत्सर्जित करने से भारतीय अर्थव्यवस्था को $86 का नुकसान होगा । भारत के बाद अमेरिका ($48) और सऊदी अरब ($47) का स्थान है।

इसलिए, विकल्प (ए) सही उत्तर है।

66

भारत में दलहन उत्पादन के संदर्भ में निम्नलिखित कथनों पर विचार कीजिएः

  1. काले चने की खेती खरीफ और रबी दोनों फसल के रूप में की जा सकती है।
  2. दलहन उत्पादन में अकेले हरे चने का योगदान होता है।
  3. पिछले तीन दशकों में जहां खरीफ दलहन का उत्पादन बढ़ा है, वहीं रबी दलहन के उत्पादन में कमी आई है।

ऊपर दिए गए कथनों में से कौन-सा/से सही है/हैं?

व्याख्या
  • भारत में, सर्दियों (रबी) में उगाई जाने वाली महत्वपूर्ण दलहन फसलें हैं चना, मसूर, लैथिरस, मटर और राजमा। हालांकि, हरे चने, काले चने और लोबिया वसंत और बरसात दोनों मौसमों में उगाए जाते हैं।
  • काला चना एक गर्म मौसम की फसल है और 600 से 1000 मिमी तक वार्षिक वर्षा वाले क्षेत्रों में आती है। इसकी खेती मुख्य रूप से अनाज-दाल फसल प्रणाली में की जाती है, मुख्य रूप से मिट्टी के पोषक तत्वों को संरक्षित करने और विशेष रूप से चावल की खेती के बाद बची हुई मिट्टी की नमी का उपयोग करने के लिए। इसलिए, हालांकि इसे सभी मौसमों में उगाया जा सकता है, काले चने की अधिकांश खेती रबी या देर से रबी के मौसम में होती है, खासकर प्रायद्वीपीय भारत में। अत: कथन 1 सही है।
  • अर्थशास्त्र और सांख्यिकी निदेशालय (डीईएस) के अनुसार, 2018-19 में दलहन उत्पादन का हिस्सा तूर (15.34%), चना (43.29%), मूंग (हरा चना, 10.04%), उड़द (काला चना) शामिल था। 13.93%), मसूर (6.67%), और अन्य दालें (10%)। अत: कथन 2 सही नहीं है।
  • पिछले तीन दशकों में, खरीफ दालों के उत्पादन और रबी दलहनों के उत्पादन दोनों में वृद्धि हुई है। अत: कथन 3 सही नहीं है।

इसलिए, विकल्प (ए) सही उत्तर है।

67

"फसल प्रकृति में उपोष्णकटिबंधीय है। एक कठिन ठंढ इसके लिए हानिकारक है। इसकी वृद्धि के लिए कम से कम 210 ठंढ मुक्त दिन और 50 से 100 सेंटीमीटर वर्षा की आवश्यकता होती है। नमी बनाए रखने में सक्षम हल्की अच्छी तरह से सूखा मिट्टी आदर्श रूप से अनुकूल है फसल की खेती।"

निम्नलिखित में से कौन-सी वह फसल है?

व्याख्या
  • कपास:
    • तापमान: 21-30 डिग्री सेल्सियस के बीच, 210 ठंढ मुक्त दिन।
    • वर्षा : लगभग 50-100 सेमी.
    • मिट्टी का प्रकार: दक्कन के पठार की अच्छी जल निकासी वाली काली कपास मिट्टी।
  • जूट:
    • तापमान: 15-34 डिग्री सेल्सियस के बीच
    • वर्षा: लगभग 100-250 सेमी।
    • मिट्टी का प्रकार: जूट को मिट्टी से लेकर बलुई दोमट तक सभी प्रकार की मिट्टी में उगाया जा सकता है, लेकिन दोमट जलोढ़ सबसे उपयुक्त हैं।
  • गन्ना:
    • तापमान: 28-32 डिग्री सेल्सियस के बीच
    • वर्षा: लगभग 75-120 सेमी।
    • मिट्टी का प्रकार: गन्ना विभिन्न प्रकार की मिट्टी में उग सकता है जिसमें काली कपास मिट्टी, दोमट, भूरी या लाल दोमट, चिकनी दोमट और यहाँ तक कि लैटेराइट भी शामिल हैं।
  • चाय:
    • तापमान: 20-30 डिग्री सेल्सियस के बीच।
    • वर्षा: लगभग 150-300 सेमी।
    • मिट्टी का प्रकार: गहरी और उपजाऊ अच्छी जल निकासी वाली मिट्टी, ह्यूमस और कार्बनिक पदार्थों से भरपूर।

इसलिए, विकल्प (ए) सही उत्तर है।

68

सौर जल पंपों के संदर्भ में, निम्नलिखित कथनों पर विचार कीजिएः

  1. सौर ऊर्जा का उपयोग सतह के पंप चलाने के लिए किया जा सकता है न कि सबमर्सिबल पंपों के लिए।
  2. सौर ऊर्जा का उपयोग सेंट्रीफ्यूगल पंप चलाने के लिए किया जा सकता है न कि पिस्टन वाले पंपों के लिए।

ऊपर दिए गए कथनों में से कौन-सा/से सही है/हैं?

व्याख्या
  • सोलर पंपिंग सिस्टम में मुख्य घटकों में एक फोटोवोल्टिक (पीवी) सरणी, एक इलेक्ट्रिक मोटर और एक पंप शामिल हैं।
  • उनके कार्यात्मक तंत्र के आधार पर कई अलग-अलग प्रकार के सौर-संचालित पंप हैं। लेकिन मुख्य रूप से चार प्रकार के सौर जल पंप हैं - सबमर्सिबल पंप, सतह पंप, प्रत्यक्ष वर्तमान (डीसी) पंप और वैकल्पिक वर्तमान (एसी) पंप। अतः कथन 1 सही नहीं है।
  • सौर ऊर्जा का उपयोग केन्द्रापसारक और पिस्टन पंप दोनों को चलाने के लिए किया जा सकता है। अत: कथन 2 सही नहीं है।

इसलिए, विकल्प (डी) सही उत्तर है।

69

भारत में गन्ने की खेती की वर्तमान प्रवृत्तियों के संदर्भ में निम्नलिखित कथनों पर विचार कीजिएः

  1. बीज सामग्री में पर्याप्त बचत तब होती है जब एक नर्स में 'बड चिप सेटलिंग्स' को उठाया जाता है, और मुख्य क्षेत्र में प्रत्यारोपित किया जाता है।
  2. जब सेटों की सीधी रोपाई की जाती है, तो कई कलियों वाले सेटों की तुलना में सिंगल-बडेड सेटों के साथ अंकुरण प्रतिशत बेहतर होता है।
  3. यदि सेटों को सीधे रोपने पर खराब मौसम की स्थिति बनी रहती है, तो बड़े सेटों की तुलना में सिंगल-बडेड सेटों की बेहतर उत्तरजीविता होती है।
  4. गन्ने की खेती टिशू कल्चर से तैयार बस्तियों का उपयोग करके की जा सकती है।

ऊपर दिए गए कथनों में से कौन-सा/से सही है/हैं?

व्याख्या
  • ऊतक संवर्धन प्रौद्योगिकी:
    • टिश्यू कल्चर एक ऐसी तकनीक है जिसमें पौधों के टुकड़ों को एक प्रयोगशाला में सुसंस्कृत और उगाया जाता है।
    • यह मौजूदा वाणिज्यिक किस्मों के रोग मुक्त बीज गन्ना का तेजी से उत्पादन और आपूर्ति करने का एक नया तरीका प्रदान करता है।
    • यह मदर प्लांट को क्लोन करने के लिए मेरिस्टेम का उपयोग करता है।
    • यह आनुवंशिक पहचान को भी बरकरार रखता है।
    • टिशू कल्चर तकनीक, अपने बोझिल पहनावे और शारीरिक सीमाओं के कारण, गैर-आर्थिक हो रही है।
  • बड चिप तकनीक:
    • ऊतक संवर्धन के एक व्यवहार्य विकल्प के रूप में, यह द्रव्यमान को कम करता है और बीजों के त्वरित गुणन को सक्षम बनाता है।
    • यह विधि दो से तीन कलियों को रोपने की पारंपरिक विधि की तुलना में अधिक किफायती और सुविधाजनक साबित हुई है।
    • रोपण के लिए उपयोग की जाने वाली बीज सामग्री पर पर्याप्त बचत के साथ, रिटर्न अपेक्षाकृत बेहतर है।
  • अत: कथन 1 सही है।
  • शोधकर्ताओं ने पाया है कि दो कलियों वाले सेट बेहतर उपज के साथ लगभग 65 से 70% तक अंकुरण दे रहे हैं। अत: कथन 2 सही नहीं है।
  • खराब मौसम में बड़े सेटों का बेहतर अस्तित्व होता है लेकिन रासायनिक उपचार से संरक्षित होने पर सिंगल बडेड सेट भी 70% अंकुरण देते हैं। अत: कथन 3 सही नहीं है।

इसलिए, विकल्प (सी) सही उत्तर है।

70

भारत के संदर्भ में, निम्नलिखित में से किसे पर्यावरण के अनुकूल कृषि का अभ्यास माना जाता है/हैं?

  1. फसल विविधीकरण
  2. फलियां गहनता
  3. टेन्सियोमीटर उपयोग
  4. खड़ी खेती

नीचे दिए गए कूट का प्रयोग कर सही उत्तर चुनिए:

व्याख्या
  • फसल विविधीकरण: यह एक विशेष खेत पर कृषि उत्पादन के लिए नई फसलों या फसल प्रणालियों को जोड़ने के लिए पूरक विपणन अवसरों के साथ मूल्य वर्धित फसलों से अलग-अलग रिटर्न को ध्यान में रखता है। किस्मों की एक बड़ी रेंज पेश करने से कृषि उत्पादन में विविधता आती है जिससे प्राकृतिक जैव विविधता में वृद्धि हो सकती है। इसके अलावा, कृषि का विविधीकरण भूमि और पानी के पुनर्जनन और संरक्षण के लिए एक वैकल्पिक तरीका है। अत: कथन 1 सही है।
  • फली गहनता:
    • एक फलीदार पौधा (फलियां) पौधों का एक समूह है जिसमें सब्जियां होती हैं या जमीन के घुंडी से उगाए गए खाद्य पदार्थ होते हैं जो नाइट्रोजन युक्त सामग्री की क्षमता को बढ़ाते हैं। उदाहरणों में बबूल, मटर, तिपतिया घास, सेम आदि शामिल हैं।
    • फलियां नाइट्रोजन और कार्बनिक पदार्थों को जोड़कर और संभावित क्षरण और लीचिंग नुकसान को कम करके, विशेष रूप से परती की तुलना में मिट्टी के स्वास्थ्य में सुधार करती हैं। फलियां नाइट्रोजन उर्वरक की आवश्यकता को कम करके फसल प्रणालियों के ऊर्जा पदचिह्न को कम कर सकती हैं, और कृषि-पारिस्थितिकी तंत्र की स्थिरता और स्वास्थ्य में सुधार कर सकती हैं। अत: कथन 2 सही है।
  • टेन्सियोमीटर: इसका शाब्दिक अर्थ है तनाव माप। मिट्टी से पानी को अवशोषित करने के लिए, पौधे को मिट्टी के चूषण तनाव को दूर करना होगा। इस तनाव को टेन्सियोमीटर द्वारा मापा जाता है, इस प्रकार मिट्टी की नमी को उस गहराई पर इंगित करता है जिसमें इसे रखा गया था। टेन्सियोमीटर किसानों और अन्य सिंचाई प्रबंधकों को यह निर्धारित करने में मदद करता है कि मिट्टी को कब पानी देना है। अत: कथन 3 सही है।
  • खड़ी खेती: यह खड़ी खड़ी परतों में फसल उगाने की प्रथा है और इसमें अक्सर नियंत्रित-पर्यावरण कृषि शामिल होती है, जिसका उद्देश्य पौधों की वृद्धि और मिट्टी रहित कृषि तकनीकों को अनुकूलित करना है। अत: कथन 4 सही है।

इसलिए, विकल्प (डी) सही उत्तर है।

71

भारत में, निम्नलिखित में से किसे कृषि में सार्वजनिक निवेश माना जा सकता है?

  1. सभी फसलों की कृषि उपज का न्यूनतम समर्थन मूल्य तय करना
  2. प्राथमिक कृषि ऋण समितियों का कम्प्यूटरीकरण
  3. सामाजिक पूंजी विकास
  4. किसानों को मुफ्त बिजली की आपूर्ति
  5. बैंकिंग प्रणाली द्वारा कृषि ऋण की छूट
  6. सरकारों द्वारा कोल्ड स्टोरेज सुविधाओं की स्थापना

नीचे दिए गए कूट का प्रयोग कर सही उत्तर चुनिए:

व्याख्या
  • सार्वजनिक निवेश राज्य (केंद्र, राज्य और स्थानीय सरकारों या सार्वजनिक स्वामित्व वाली कंपनियों के माध्यम से) द्वारा बुनियादी भौतिक बुनियादी ढांचे (जैसे सड़कों, पुलों, रेल लाइनों, हवाई अड्डों, और जल वितरण) के लिए संसाधनों को समर्पित करके देश के पूंजीगत स्टॉक का निर्माण करने के लिए एक निवेश है। ), नवीन गतिविधि (बुनियादी अनुसंधान), हरित निवेश (स्वच्छ ऊर्जा स्रोत), और शिक्षा जो उच्च उत्पादकता और/या उच्च जीवन स्तर की ओर ले जाती है।
  • न्यूनतम समर्थन मूल्य (एमएसपी) किसानों को कीमतों में गिरावट से बचाने के लिए एक बीमा पॉलिसी के रूप में काम करने के लिए एक बाजार हस्तक्षेप तंत्र प्रदान करता है। यह कृषि क्षेत्र में उत्पादकता बढ़ाने में मदद नहीं कर रहा है। इस प्रकार, सभी फसलों की कृषि उपज के लिए एमएसपी तय करने से राष्ट्रीय पूंजी स्टॉक में वृद्धि नहीं होती है या उच्च उत्पादकता नहीं होती है। अतः कथन 1 सही नहीं है।
  • प्राथमिक कृषि ऋण समितियों के कम्प्यूटरीकरण से कृषि क्षेत्र में उत्पादकता में वृद्धि होगी, क्योंकि ऋण की आसान और समय पर पहुँच होगी। कोल्ड स्टोरेज की सुविधा स्थापित करने का भी ऐसा ही असर होगा। अत: कथन 2 और 6 सही हैं।
  • सामाजिक पूंजी वह मूल्य है जिसे नेटवर्किंग और लोगों और संगठनों के बीच और उनके बीच निर्मित विश्वास के माध्यम से बनाया जा सकता है। एक समुदाय के भीतर एकजुटता का निर्माण एक साथ काम करने की लेन-देन की लागत को कम करता है और बढ़ा हुआ विश्वास समुदायों को सामाजिक दुविधाओं को दूर करने में सक्षम बना सकता है। मजबूत सामाजिक नेटवर्क संसाधन गरीब व्यक्तियों या समुदायों को झटके से निपटने में मदद करने के लिए सुरक्षा जाल के रूप में काम कर सकते हैं, खासकर जब औपचारिक प्रकार के जोखिम प्रबंधन जैसे क्रेडिट या बीमा अनुपलब्ध हैं। कृषक समुदायों के भीतर, सामाजिक पूंजी भी उत्पादकता में सुधार कर सकती है क्योंकि यह 'प्राकृतिक संसाधनों के प्रबंधन या नई प्रथाओं और प्रौद्योगिकियों को अपनाने के लिए एक पूर्व-आवश्यकता है। छोटे जोतदारों के लिए सामाजिक पूंजी का निर्माण भी नई तकनीकों को अपनाने पर सकारात्मक प्रभाव डाल सकता है जैसे कि उन्नत बीजों का उपयोग,अत: कथन 3 सही है।
  • किसानों को मुफ्त बिजली देने से राज्य पर वित्तीय दबाव के अलावा, पानी का अत्यधिक उपयोग, अधिक पंप सेटों की स्थापना को प्रोत्साहन और भूजल स्तर में गिरावट आई है। अत: कथन 4 सही नहीं है।
  • कृषि ऋण की माफी निवेश नहीं है क्योंकि यह बैंकिंग क्षेत्र के स्वास्थ्य को प्रभावित करती है। अत: कथन 5 सही नहीं है।

इसलिए, विकल्प (सी) सही उत्तर है।

72

भारत में एक फर्म के "ब्याज कवरेज अनुपात" शब्द का क्या महत्व है?

  1. यह एक फर्म के वर्तमान जोखिम को समझने में मदद करता है जिसे बैंक ऋण देने जा रहा है।
  2. यह एक फर्म के उभरते जोखिम का मूल्यांकन करने में मदद करता है जिसे बैंक ऋण देने जा रहा है।
  3. एक उधार लेने वाली फर्म का ब्याज कवरेज अनुपात का स्तर जितना अधिक होता है, उसकी ऋण चुकाने की क्षमता उतनी ही खराब होती है।

नीचे दिए गए कूट का प्रयोग कर सही उत्तर चुनिए:

व्याख्या
  • ब्याज कवरेज अनुपात (ICR) एक ऋण अनुपात और लाभप्रदता अनुपात है जिसका उपयोग यह निर्धारित करने के लिए किया जाता है कि कोई कंपनी अपने बकाया ऋण पर कितनी आसानी से ब्याज का भुगतान कर सकती है। यह मापता है कि कोई कंपनी अपनी उपलब्ध आय के साथ अपने वर्तमान ब्याज भुगतान को कितनी बार कवर कर सकती है। इसकी गणना एक निश्चित अवधि के दौरान ब्याज और करों (ईबीआईटी) से पहले कंपनी की कमाई को उसी अवधि के भीतर कंपनी के ब्याज भुगतान से विभाजित करके की जा सकती है। अत: कथन 1 सही है।

  • आईसीआर का उपयोग आमतौर पर उधारदाताओं, लेनदारों और निवेशकों द्वारा कंपनी के वर्तमान ऋण या भविष्य के उधार के सापेक्ष जोखिम का निर्धारण करने के लिए किया जाता है। अत: कथन 2 सही है।
  • ब्याज कवरेज अनुपात जितना अधिक होगा, उतना ही बेहतर होगा। अनुपात जितना कम होगा, कंपनी उतनी ही अधिक कर्ज के बोझ से दब जाएगी। जब किसी कंपनी का ब्याज कवरेज अनुपात केवल 1.5 या उससे कम होता है, तो ब्याज व्यय को पूरा करने की उसकी क्षमता संदिग्ध हो सकती है। अत: कथन 3 सही नहीं है।

इसलिए, विकल्प (ए) सही उत्तर है।

73

निम्नलिखित में से कौन से कारक/नीतियाँ हाल के दिनों में भारत में चावल की कीमत को प्रभावित कर रही थीं?

  1. न्यूनतम समर्थन मूल्य
  2. सरकार का व्यापार
  3. सरकार का भंडार
  4. उपभोक्ता सब्सिडी

नीचे दिए गए कूट का प्रयोग कर सही उत्तर चुनिए:

व्याख्या
  • न्यूनतम समर्थन मूल्य (MSP) सरकार द्वारा तय किया गया न्यूनतम समर्थन मूल्य है जिस पर वह किसानों से फसल खरीदती है। एमएसपी का उद्देश्य किसानों को अच्छी फसल के मौसम में कृषि कीमतों में तेज गिरावट से बचाना है। हालांकि, किसी भी फसल पर निर्धारित एमएसपी किसानों को उस विशिष्ट फसल का उत्पादन करने के लिए आकर्षित करता है जिससे इसका अधिक उत्पादन हो सकता है। एमएसपी भी बाजार को विकृत करता है क्योंकि सरकारी खरीद एजेंसियां ​​​​निजी खिलाड़ियों को मजबूर करके 70-80% चावल खरीदती हैं। यदि सरकार बाजार दर से अधिक एमएसपी निर्धारित करती है, तो निजी खिलाड़ी किसानों से फसल नहीं खरीदेंगे और फसलों की खरीद सरकार द्वारा की जाती है। जो बाजारों में अक्षमता पैदा करता है। किसान आमतौर पर ऐसी अन्य फसलों का उत्पादन करना पसंद नहीं करते हैं, जिन पर कोई निश्चित एमएसपी नहीं होता है, जिससे अंततः उन फसलों की कीमत बढ़ जाती है।इसलिए, कारक/नीति 1 सही है।
  • भारत में सरकार आम तौर पर देश के चावल के उत्पादन का एक तिहाई से अधिक एक निश्चित मूल्य पर खरीदती है, जिसका चावल की कीमत पर सीधा प्रभाव पड़ता है। इसलिए, कारक/नीति 2 सही है।
  • भारत में, सरकार गरीबों को वितरण के लिए गेहूं और चावल जैसे अनाज का भंडार करती है जिससे चावल की कीमत प्रभावित होती है। सरकार सार्वजनिक वितरण प्रणाली के माध्यम से खरीदी गई फसलों को रियायती दरों पर बेचती है। इसलिए, कारक/नीति 3 सही है।
  • सब्सिडी का मतलब है कि सरकार लागत का कुछ हिस्सा चुकाती है। यह निर्माता और उपभोक्ता दोनों को प्रदान किया जा सकता है। भारत में, रियायती दर पर खाद्यान्न टीपीडीएस के माध्यम से वितरित किया जाता है जो उचित मूल्य की दुकानों के माध्यम से बेचे जाने वाले खाद्यान्न की कीमत को प्रभावित करता है। इसलिए, कारक/नीति 4 सही है।

इसलिए, विकल्प (डी) सही उत्तर है।

74

निम्नलिखित कथनों पर विचार करें:

  1. पिछले एक दशक में भारत-श्रीलंका व्यापार के मूल्य में लगातार वृद्धि हुई है।
  2. "कपड़ा और कपड़ा लेख" भारत और बांग्लादेश के बीच व्यापार की एक महत्वपूर्ण वस्तु है।
  3. पिछले पांच वर्षों में नेपाल दक्षिण एशिया में भारत का सबसे बड़ा व्यापारिक भागीदार रहा है।

ऊपर दिए गए कथनों में से कौन-सा/से सही है/हैं?

व्याख्या
  • वाणिज्य विभाग के आंकड़ों के अनुसार, एक दशक (2007 से 2016) के लिए भारत-श्रीलंका द्विपक्षीय व्यापार मूल्य 3.0, 3.4, 2.1, 3.8, 5.2, 4.5, 5.3, 7.0, 6.3, 4.8 (अरब अमरीकी डालर में) था। यह व्यापार मूल्य की प्रवृत्ति में निरंतर उतार-चढ़ाव को दर्शाता है। समग्र वृद्धि हुई है लेकिन इसे व्यापार मूल्य में लगातार वृद्धि के रूप में नहीं कहा जा सकता है। अतः कथन 1 सही नहीं है।
  • निर्यात में 5% से अधिक और आयात में 7% से अधिक की हिस्सेदारी के साथ बांग्लादेश भारत के लिए एक प्रमुख कपड़ा व्यापार भागीदार रहा है। जबकि बांग्लादेश को सालाना कपड़ा निर्यात औसतन 2,000 मिलियन डॉलर है, आयात 400 डॉलर (वर्ष: 2016-17) के लायक है।
  • निर्यात की प्रमुख वस्तुएं कपास के फाइबर और यार्न, मानव निर्मित स्टेपल फाइबर और मानव निर्मित फिलामेंट हैं, जबकि प्रमुख आयात वस्तुओं में परिधान और कपड़े, कपड़े और अन्य निर्मित कपड़ा लेख शामिल हैं। अत: कथन 2 सही है।
  • आंकड़ों के अनुसार, 2016-17 में, बांग्लादेश दक्षिण एशिया में भारत का सबसे बड़ा व्यापारिक भागीदार है, इसके बाद नेपाल, श्रीलंका, पाकिस्तान, भूटान, अफगानिस्तान और मालदीव हैं। भारतीय निर्यात का स्तर भी इसी क्रम का अनुसरण करता है। अत: कथन 3 सही नहीं है।

अतः विकल्प (ब) सही उत्तर है।

75

निम्नलिखित में से किस समूह में सभी चार देश G20 के सदस्य हैं?

व्याख्या
  • G20 अंतर्राष्ट्रीय मुद्रा कोष और विश्व बैंक के प्रतिनिधियों के साथ 19 देशों और यूरोपीय संघ का एक अनौपचारिक समूह है।
  • एक मजबूत वैश्विक आर्थिक विकास प्राप्त करने के लिए, सदस्य देश जो वैश्विक सकल घरेलू उत्पाद का 80% से अधिक का प्रतिनिधित्व करते हैं और योगदान करते हैं, अंतरराष्ट्रीय आर्थिक सहयोग के लिए प्रमुख मंच पर आए, जिस पर पेन्सिलवेनिया (यूएसए) में पिट्सबर्ग शिखर सम्मेलन में नेताओं द्वारा सहमति व्यक्त की गई थी। सितंबर 2009।
  • G20 के सदस्यों में अर्जेंटीना, ऑस्ट्रेलिया, ब्राजील, कनाडा, चीन, फ्रांस, जर्मनी, भारत, इंडोनेशिया, इटली, जापान, मैक्सिको, कोरिया गणराज्य, रूस, सऊदी अरब, दक्षिण अफ्रीका, तुर्की, यूनाइटेड किंगडम, संयुक्त राज्य अमेरिका और यूरोपीय संघ (ईयू)।

इसलिए, विकल्प (ए) सही उत्तर है।

76

किसान क्रेडिट कार्ड योजना के तहत, किसानों को निम्न में से किस उद्देश्य के लिए अल्पकालिक ऋण सहायता दी जाती है?

  1. कृषि संपत्ति के रखरखाव के लिए कार्यशील पूंजी
  2. कंबाइन हार्वेस्टर, ट्रैक्टर और मिनी ट्रक की खरीद
  3. खेतिहर परिवारों की उपभोग आवश्यकताएं
  4. फसल के बाद का खर्च
  5. पारिवारिक आवास का निर्माण एवं ग्राम कोल्ड स्टोरेज सुविधा की स्थापना

नीचे दिए गए कूट का प्रयोग कर सही उत्तर चुनिए:

व्याख्या
  • किसान क्रेडिट कार्ड (केसीसी) योजना 1998 में किसानों को उनकी खेती के लिए लचीली और सरलीकृत प्रक्रिया के साथ एकल खिड़की के तहत बैंकिंग प्रणाली से पर्याप्त और समय पर ऋण सहायता प्रदान करने के लिए शुरू की गई थी और अन्य जरूरतों जैसे कि कृषि आदानों की खरीद जैसे बीज, उर्वरकों, कीटनाशकों आदि का उपयोग करते हैं और अपनी उत्पादन आवश्यकताओं के लिए नकद आहरित करते हैं।
  • किसानों की संबद्ध और गैर-कृषि गतिविधियों की निवेश ऋण आवश्यकता के लिए योजना को वर्ष 2004 में आगे बढ़ाया गया था।
  • किसान क्रेडिट कार्ड निम्नलिखित उद्देश्यों के साथ प्रदान किया जाता है:
    • फसलों की खेती के लिए अल्पकालिक ऋण आवश्यकताएं
    • फसल के बाद का खर्च
    • उत्पादन विपणन ऋण
    • किसान परिवार की खपत आवश्यकताएँ
    • कृषि संपत्ति और कृषि से संबंधित गतिविधियों, जैसे डेयरी पशु, अंतर्देशीय मत्स्य पालन, आदि के रखरखाव के लिए कार्यशील पूंजी।
    • कृषि और संबद्ध गतिविधियों जैसे पंपसेट, स्प्रेयर, डेयरी पशु आदि के लिए निवेश ऋण की आवश्यकता।
  • किसान क्रेडिट कार्ड योजना वाणिज्यिक बैंकों, आरआरबी, लघु वित्त बैंकों और सहकारी समितियों द्वारा कार्यान्वित की जाती है।

अतः विकल्प (ब) सही उत्तर है।

77

निम्नलिखित कथनों पर विचार करें:

  1. उपभोक्ता मूल्य सूचकांक (सीपीआई) में भोजन का भार थोक मूल्य सूचकांक (डब्ल्यूपीआई) की तुलना में अधिक है।
  2. WPI सेवाओं की कीमतों में परिवर्तन पर कब्जा नहीं करता है, जो CPI करता है।
  3. भारतीय रिजर्व बैंक ने अब WPI को मुद्रास्फीति के अपने प्रमुख उपाय के रूप में अपनाया है और प्रमुख नीतिगत दरों को बदलने का निर्णय लिया है।

ऊपर दिए गए कथनों में से कौन-सा/से सही है/हैं?

व्याख्या
  • थोक मूल्य सूचकांक (WPI) थोक बाजार में या थोक स्तर पर माल की कीमतों में औसत परिवर्तन का एक उपाय है। यह आर्थिक सलाहकार कार्यालय, वाणिज्य और उद्योग मंत्रालय द्वारा प्रकाशित किया जाता है।
  • उपभोक्ता मूल्य सूचकांक (सीपीआई) घरों द्वारा खरीदी गई उपभोक्ता वस्तुओं और सेवाओं की एक टोकरी के मूल्य स्तर में परिवर्तन का माप है। आइटम बास्केट के आधार पर CPI चार प्रकार के होते हैं:
    • औद्योगिक श्रमिकों के लिए सीपीआई (आईडब्ल्यू)
    • कृषि मजदूर के लिए सीपीआई (एएल)
    • ग्रामीण मजदूर के लिए सीपीआई (आरएल)
    • सीपीआई (ग्रामीण/शहरी/संयुक्त)
  • इनमें से पहले तीन को श्रम और रोजगार मंत्रालय में श्रम ब्यूरो द्वारा संकलित किया गया है। चौथा सांख्यिकी और कार्यक्रम कार्यान्वयन मंत्रालय में केंद्रीय सांख्यिकी संगठन (सीएसओ) द्वारा संकलित किया गया है।
  • सीपीआई में मदों का भार उपभोक्ता व्यय सर्वेक्षणों से लिए गए औसत घरेलू व्यय पर आधारित है। सीपीआई में भोजन का भार डब्ल्यूपीआई (लगभग 24%) की तुलना में कहीं अधिक (लगभग 46%) है। थोक मूल्य सूचकांक मदों की टोकरी का एक महत्वपूर्ण अनुपात विनिर्माण आदानों और मध्यवर्ती वस्तुओं जैसे खनिज, मूल धातु, मशीनरी आदि का प्रतिनिधित्व करता है। इसलिए, कथन 1 सही है।
  • इसके अलावा, WPI सेवाओं की कीमतों में परिवर्तन पर कब्जा नहीं करता है, जो CPI करता है। अत: कथन 2 सही है।
  • WPI का उपयोग कुछ अर्थव्यवस्थाओं में मुद्रास्फीति के एक प्रमुख उपाय के रूप में किया जाता है। हालाँकि, RBI अब इसका उपयोग नीतिगत उद्देश्यों के लिए नहीं करता है, जिसमें रेपो दरें निर्धारित करना भी शामिल है। अप्रैल 2014 में, आरबीआई ने मौद्रिक और क्रेडिट नीति निर्धारित करने के लिए मुद्रास्फीति के एक प्रमुख उपाय के रूप में सीपीआई या खुदरा मुद्रास्फीति को अपनाया। अत: कथन 3 सही नहीं है।

इसलिए विकल्प (ए) सही उत्तर है।

78

निम्नलिखित जोड़े पर विचार करें:

नदीउसमे बहती है
1. मेकांगअंडमान सागर
2. थेम्सआयरिश सागर
3. वोल्गाकैस्पियन सागर
4. ज़ाम्बेज़िकहिंद महासागर


ऊपर दिए गए युग्मों में से कौन-सा/से सही सुमेलित है/हैं?

व्याख्या
  • मेकांग नदी, तिब्बती हाइलैंड्स के बर्फीले हेडवाटर में उत्पन्न होती है, वियतनाम में एक विस्तृत डेल्टा बनाने और खाली होने से पहले, निचले बेसिन देशों म्यांमार, लाओस, थाईलैंड और कंबोडिया के माध्यम से चीन के खड़ी घाटियों के माध्यम से बहती है, जिसे ऊपरी बेसिन के रूप में जाना जाता है। दक्षिण चीन सागर। अत: युग्म 1 सुमेलित नहीं है।
  • टेम्स नदी, इंग्लैंड की सबसे लंबी नदी, कॉटस्वोल्ड्स से उत्तरी सागर तक 215 मील बहती है। टेम्स की मुख्य सहायक नदियाँ बसकोट, रीडिंग और किंग्स्टन हैं। अत: युग्म 2 सुमेलित नहीं है।
  • वोल्गा नदी, यूरोप की सबसे लंबी नदी, रूस से होकर गुजरती है और इसका डेल्टा कजाकिस्तान सीमा के दक्षिण में कैस्पियन सागर में बहती है। अत: युग्म 3 सही सुमेलित है।
  • ज़ाम्बेजी अफ्रीका में कांगो/ज़ायर, नील और नाइजर के बाद चौथी सबसे बड़ी नदी है। यह उत्तर-पश्चिमी जाम्बिया में कालेन पहाड़ियों में उगता है और हिंद महासागर में लगभग 3000 किमी के लिए पूर्व की ओर बहता है। अत: युग्म 4 सही सुमेलित है।

इसलिए, विकल्प (सी) सही उत्तर है।

79

निम्नलिखित कथनों पर विचार करें:

  1. सभी अनाजों, दालों और तिलहनों के मामले में, भारत के किसी भी राज्य/संघ राज्य क्षेत्र में न्यूनतम समर्थन मूल्य (एमएसपी) पर खरीद असीमित है।
  2. अनाज और दालों के मामले में, एमएसपी किसी भी राज्य/केंद्र शासित प्रदेश में उस स्तर पर तय किया जाता है, जिस स्तर तक बाजार मूल्य कभी नहीं बढ़ेगा।

ऊपर दिए गए कथनों में से कौन-सा/से सही है/हैं?

व्याख्या
  • भारत सरकार कृषि लागत और मूल्य आयोग (CACP) की सिफारिशों को ध्यान में रखते हुए प्रत्येक वर्ष दोनों फसल मौसमों में 22 प्रमुख कृषि वस्तुओं के लिए न्यूनतम समर्थन मूल्य (MSP) की घोषणा करती है।
  • सीएसीपी बाईस (22) फसलों के लिए एमएसपी और गन्ने के लिए उचित और लाभकारी मूल्य (एफआरपी) की सिफारिश करता है।
    • 7 अनाज (धान, गेहूं, मक्का, ज्वार, बाजरा, जौ और रागी),
    • 5 दालें (चना, अरहर, मूंग, उड़द, मसूर),
    • 7 तिलहन (मूंगफली, रेपसीड-सरसों, सोयाबीन, तिल, सूरजमुखी, कुसुम, नाइजरसीड), और
    • 4 व्यावसायिक फसलें (खोपरा, गन्ना, कपास और कच्चा जूट)।
  • खाद्य और सार्वजनिक वितरण विभाग चीनी के लिए उचित और लाभकारी मूल्य (FRP) घोषित करता है।
  • कुल खरीद मात्रा आम तौर पर उस विशेष वर्ष/मौसम के लिए वस्तु के वास्तविक उत्पादन के 25% से अधिक नहीं होनी चाहिए। 25% की खरीद सीमा से अधिक, यदि कोई हो, कृषि विभाग (डीएसी) के पूर्व अनुमोदन की आवश्यकता होगी। अतः कथन 1 सही नहीं है।
  • एमएसपी विभिन्न राज्यों द्वारा किए गए एमएसपी प्रस्तावों के औसत के आधार पर केंद्र सरकार द्वारा तय किया जाता है, जिनमें से कुछ केंद्र की सिफारिश से अधिक हो सकते हैं।
  • जबकि इनपुट लागत पर आधारित प्रस्ताव अलग-अलग राज्यों में अलग-अलग होते हैं, मूल्य असमानता से बचने के लिए एमएसपी तय किया जाता है। जब बाजार की कीमतें एमएसपी से नीचे के स्तर तक गिर जाती हैं, तो सरकारी एजेंसियां ​​​​किसानों की सुरक्षा के लिए उपज को खरीद लेती हैं। ऐसे में बाजार में कीमतें एमएसपी से ऊपर जा सकती हैं। अत: कथन 2 सही नहीं है।

इसलिए, विकल्प (डी) सही उत्तर है।

80

भारतीय अर्थव्यवस्था के संदर्भ में निम्नलिखित कथनों पर विचार कीजिएः

  1. 'कमर्शियल पेपर' एक अल्पकालिक असुरक्षित वचन पत्र है।
  2. 'जमा प्रमाणपत्र' भारतीय रिजर्व बैंक द्वारा एक निगम को जारी किया गया एक दीर्घकालिक साधन है।
  3. 'कॉल मनी' एक अल्पकालिक वित्त है जिसका उपयोग अंतरबैंक लेनदेन के लिए किया जाता है।
  4. 'शून्य-कूपन बांड' अनुसूचित वाणिज्यिक बैंकों द्वारा निगमों को जारी किए गए ब्याज वाले अल्पकालिक बांड हैं।

ऊपर दिए गए कथनों में से कौन-सा/से सही है/हैं?

व्याख्या
  • वाणिज्यिक पत्र (सीपी) एक असुरक्षित मुद्रा बाजार साधन है जो एक वचन पत्र के रूप में जारी किया जाता है और सेबी द्वारा अनुमोदित और पंजीकृत किसी भी डिपॉजिटरी के माध्यम से एक डीमैट रूप में रखा जाता है। अत: कथन 1 सही है।
  • जमा प्रमाणपत्र (सीडी) एक परक्राम्य मुद्रा बाजार साधन है और एक निर्दिष्ट समय अवधि के लिए बैंक या अन्य पात्र वित्तीय संस्थान में जमा किए गए धन के लिए डीमैटरियलाइज्ड रूप में या एक यूसेंस प्रॉमिसरी नोट के रूप में जारी किया जाता है। सीडी (i) क्षेत्रीय ग्रामीण बैंकों (आरआरबी) और स्थानीय क्षेत्र बैंकों (एलएबी) को छोड़कर अनुसूचित वाणिज्यिक बैंकों द्वारा जारी किए जा सकते हैं; और (ii) उन अखिल भारतीय वित्तीय संस्थानों (एफआई) का चयन करें जिन्हें आरबीआई द्वारा आरबीआई द्वारा निर्धारित छत्र सीमा के भीतर अल्पकालिक संसाधन जुटाने की अनुमति दी गई है। अत: कथन 2 सही नहीं है।
  • कॉल मनी एक वित्तीय संस्थान द्वारा किसी अन्य वित्तीय संस्थान को दिया जाने वाला 1 से 14 दिनों का एक अल्पकालिक, ब्याज-भुगतान वाला ऋण है। अत: कथन 3 सही है।
  • एक शून्य-कूपन बांड एक ऋण सुरक्षा है जो ब्याज का भुगतान नहीं करता है, बल्कि एक गहरी छूट पर ट्रेड करता है, परिपक्वता पर लाभ प्रदान करता है, जब बांड को उसके पूर्ण अंकित मूल्य के लिए भुनाया जाता है। अत: कथन 4 सही नहीं है।

इसलिए, विकल्प (सी) सही उत्तर है।

81

भारत में प्रत्यक्ष विदेशी निवेश के सन्दर्भ में निम्नलिखित में से कौन-सी इसकी प्रमुख विशेषता मानी जाती है?

व्याख्या
  • प्रत्यक्ष विदेशी निवेश (FDI) भारत से बाहर के निवासी व्यक्ति द्वारा पूंजी लिखतों के माध्यम से किया गया निवेश है:
    • एक असूचीबद्ध भारतीय कंपनी; या
    • एक सूचीबद्ध भारतीय कंपनी के पूरी तरह से पतला आधार पर पेड-अप इक्विटी पूंजी का 10% या अधिक।
  • इसलिए FDI किसी लिस्टेड या अनलिस्टेड कंपनी में हो सकता है। इसलिए, विकल्प (ए) सही नहीं है।
  • एफडीआई के माध्यम से भारत में निवेश की गई पूंजी गैर कर्जदार है और कर्ज चुकाने की अनुमति नहीं है। इसलिए, विकल्प (बी) सही है और (सी) सही नहीं है।
  • एक निवेश को विदेशी पोर्टफोलियो निवेश कहा जाता है, यदि भारत के बाहर निवासी किसी व्यक्ति (या संस्थागत निवेशक) द्वारा पूंजी लिखतों में किया गया निवेश है:
    • एक सूचीबद्ध भारतीय कंपनी के पूरी तरह से पतला आधार पर पेड-अप इक्विटी पूंजी के 10% से कम, या
    • किसी सूचीबद्ध भारतीय कंपनी के पूंजीगत लिखतों की प्रत्येक श्रृंखला के प्रदत्त मूल्य के 10% से कम।

इसलिए, विकल्प (डी) सही नहीं है।

अतः विकल्प (ब) सही उत्तर है।

82

वर्तमान में भारत के अंतर्राष्ट्रीय व्यापार के संदर्भ में, निम्नलिखित में से कौन सा/से कथन सही है/हैं?

  1. भारत का व्यापारिक निर्यात इसके व्यापारिक आयात से कम है।
  2. भारत के लौह और इस्पात, रसायन, उर्वरक और मशीनरी के आयात में हाल के वर्षों में कमी आई है।
  3. भारत का सेवाओं का निर्यात उसकी सेवाओं के आयात से अधिक है।
  4. भारत एक समग्र व्यापार/चालू खाता घाटे से ग्रस्त है।

नीचे दिए गए कूट का प्रयोग कर सही उत्तर चुनिए:

व्याख्या
  • अगस्त 2020 तक, भारत का व्यापारिक निर्यात 22.70 बिलियन अमरीकी डालर था और व्यापारिक आयात 29.47 बिलियन अमरीकी डालर था, जिसका स्पष्ट अर्थ है कि व्यापारिक निर्यात उसके व्यापारिक आयात से कम है। अत: कथन 1 सही है।
  • आर्थिक सर्वेक्षण 2020 के अनुसार, भारत के लौह और इस्पात के आयात में कमी आई है लेकिन रसायनों, उर्वरकों और मशीनरी के आयात में वृद्धि हुई है। अत: कथन 2 सही नहीं है।
  • अप्रैल-अगस्त 2020-21 तक, 49.56 बिलियन अमरीकी डालर के अनुमानित सेवा आयात की तुलना में सेवा निर्यात 84.47 बिलियन अमरीकी डालर होने का अनुमान है। इसका मतलब है कि भारत का सेवाओं का निर्यात उसकी सेवाओं के आयात से अधिक है। अत: कथन 3 सही है।
  • व्यापार घाटा एक ऐसी स्थिति है जब माल का निर्यात उसके आयात से कम होता है जबकि चालू खाता घाटा एक ऐसी स्थिति होती है जब वस्तुओं और सेवाओं का समग्र व्यापार नकारात्मक पक्ष पर होता है (अर्थात देश ने निर्यात से अधिक आयात किया)। अभी तक, भारत का माल का निर्यात उसके आयात से कम है लेकिन सेवाओं का निर्यात उसके आयात से अधिक है। इसके अलावा, उच्च व्यापार घाटे के कारण समग्र व्यापार संतुलन नकारात्मक है। इसलिए, भारत एक समग्र व्यापार/चालू खाता घाटे से ग्रस्त है। अत: कथन 4 सही है।

इसलिए, विकल्प (डी) सही उत्तर है।

83

शब्द 'वेस्ट टेक्सास इंटरमीडिएट', जो कभी-कभी समाचारों में पाया जाता है, एक ग्रेड को संदर्भित करता है

व्याख्या
  • वेस्ट टेक्सास इंटरमीडिएट (WTI), जिसे टेक्सास लाइट स्वीट के रूप में भी जाना जाता है, कच्चे तेल का एक ग्रेड है जिसका उपयोग तेल मूल्य निर्धारण में बेंचमार्क के रूप में किया जाता है।
    • डब्ल्यूटीआई को अपेक्षाकृत कम घनत्व के कारण हल्के कच्चे तेल के रूप में वर्णित किया जाता है, और इसकी कम सल्फर सामग्री के कारण मीठा होता है।
    • यह मुख्य रूप से टेक्सास, लुइसियाना और नॉर्थ डकोटा में अमेरिकी तेल क्षेत्रों से प्राप्त होता है।

इसलिए, विकल्प (ए) सही उत्तर है।

84

भारतीय अर्थव्यवस्था के संदर्भ में, गैर-वित्तीय ऋण में निम्नलिखित में से क्या शामिल है?

  1. परिवारों पर बकाया आवास ऋण
  2. क्रेडिट कार्ड पर बकाया राशि
  3. राजकोष चालान

नीचे दिए गए कूट का प्रयोग कर सही उत्तर चुनिए:

व्याख्या
  • ऋण मौद्रिक ऋण चुकाने के लिए संविदात्मक दायित्व होते हैं, अक्सर संबंधित ब्याज व्यय के साथ।
  • गैर-वित्तीय ऋण:
    • इसमें सरकारी संस्थाओं, घरों और व्यवसायों द्वारा जारी किए गए क्रेडिट उपकरण शामिल हैं जो वित्तीय क्षेत्र में शामिल नहीं हैं।
    • इसमें औद्योगिक या वाणिज्यिक ऋण, ट्रेजरी बिल और क्रेडिट कार्ड शेष शामिल हैं।
    • वे अधिकांश समान विशेषताओं को वित्तीय ऋण के साथ साझा करते हैं, सिवाय इसके कि जारीकर्ता गैर-वित्तीय हैं।

अत: कथन 1, 2 और 3 सही हैं।

इसलिए, विकल्प (डी) सही उत्तर है।

85

भारत में, कुछ परमाणु रिएक्टरों को "IAEA सुरक्षा उपायों" के तहत क्यों रखा जाता है जबकि अन्य को नहीं?

व्याख्या
  • परमाणु सुविधाओं को अंतर्राष्ट्रीय परमाणु ऊर्जा एजेंसी (IAEA) सुरक्षा उपायों के तहत रखा जाता है यदि यूरेनियम का स्रोत जो परमाणु रिएक्टर के लिए विखंडनीय सामग्री है, भारत के क्षेत्र से बाहर है या यदि नए रिएक्टर संयंत्र विदेशी सहयोग से स्थापित किए गए हैं।
  • यह सुनिश्चित करने के लिए है कि आयातित यूरेनियम को सैन्य उपयोग के लिए डायवर्ट नहीं किया गया था और यह सुनिश्चित करने के लिए कि आयातित यूरेनियम का उपयोग नागरिक उद्देश्यों के लिए परमाणु ऊर्जा उत्पन्न करने के लिए किया जाता है।
  • वर्तमान में 22 परिचालन रिएक्टर हैं, जिनमें से 14 अंतर्राष्ट्रीय परमाणु ऊर्जा एजेंसी (आईएईए) के सुरक्षा उपायों के तहत हैं क्योंकि ये आयातित ईंधन का उपयोग करते हैं।
  • सुरक्षा उपायों के समझौते के तहत, अंतर्राष्ट्रीय परमाणु ऊर्जा एजेंसी (आईएईए) के पास यह सुनिश्चित करने का अधिकार और दायित्व है कि विशेष उद्देश्य के लिए राज्य के क्षेत्र, अधिकार क्षेत्र या नियंत्रण में सभी परमाणु सामग्री पर सुरक्षा उपायों को लागू किया जाता है।

अतः विकल्प (ब) सही उत्तर है।

86

व्यापार-संबंधित निवेश उपाय (TRIMS) के संदर्भ में, निम्नलिखित में से कौन सा/से कथन सही है/हैं?

  1. विदेशी निवेशकों द्वारा आयात पर मात्रात्मक प्रतिबंध निषिद्ध हैं।
  2. वे वस्तुओं और सेवाओं दोनों में व्यापार से संबंधित निवेश उपायों पर लागू होते हैं।
  3. उनका विदेशी निवेश के नियमन से कोई सरोकार नहीं है।

नीचे दिए गए कूट का प्रयोग कर सही उत्तर चुनिए:

व्याख्या
  • विश्व व्यापार संगठन (डब्ल्यूटीओ) के व्यापार से संबंधित निवेश उपायों (टीआरआईएम) पर समझौते के तहत, जिसे आमतौर पर टीआरआईएमएस समझौते के रूप में जाना जाता है (उरुग्वे दौर 1986-1994 के दौरान बातचीत की गई), डब्ल्यूटीओ के सदस्यों ने कुछ निवेश उपायों को लागू नहीं करने पर सहमति व्यक्त की है जो भेदभाव करते हैं। विदेशी माल के खिलाफ जो व्यापार को प्रतिबंधित या विकृत करता है (GATT अनुच्छेद III के तहत राष्ट्रीय उपचार) या मात्रात्मक प्रतिबंध (अनुच्छेद XI) की ओर जाता है, जो दोनों मूल विश्व व्यापार संगठन के सिद्धांतों का उल्लंघन करते हैं। अत: कथन 1 सही है।
  • यह समझौता केवल उन उपायों पर लागू होता है जो माल के व्यापार को प्रभावित करते हैं। अत: कथन 2 सही नहीं है।
  • समझौते का संबंध विदेशी निवेश के नियमन से नहीं है। TRIMS समझौते के विषय निवेश उपायों पर ध्यान केंद्रित करते हैं जो GATT के अनुच्छेद III और XI का उल्लंघन करते हैं। दूसरे शब्दों में, यह उन निवेश उपायों पर ध्यान केंद्रित करता है जो आयातित और निर्यात किए गए उत्पादों के बीच भेदभाव करते हैं। अत: कथन 3 सही नहीं है।

इसलिए, विकल्प (सी) सही उत्तर है।

87

यदि भारतीय रिजर्व बैंक एक विस्तारवादी मौद्रिक नीति अपनाने का निर्णय लेता है, तो वह निम्नलिखित में से क्या नहीं करेगा?

  1. वैधानिक तरलता अनुपात में कटौती और अनुकूलन
  2. सीमांत स्थायी सुविधा दर बढ़ाएँ
  3. बैंक रेट और रेपो रेट में कटौती करें

नीचे दिए गए कूट का प्रयोग कर सही उत्तर चुनिए:

व्याख्या
  • विस्तारित मौद्रिक नीति, या आसान मौद्रिक नीति, तब होती है जब एक केंद्रीय बैंक अर्थव्यवस्था को प्रोत्साहित करने के लिए अपने उपकरणों का उपयोग करता है। यह मुद्रा आपूर्ति को बढ़ाता है, ब्याज दरों को कम करता है और मांग को बढ़ाता है। यह आर्थिक विकास को बढ़ावा देता है।
  • वैधानिक तरलता अनुपात (एसएलआर) एक मौद्रिक नीति उपकरण है जिसका उपयोग भारतीय रिजर्व बैंक (आरबीआई) बैंकों के निपटान में तरलता का आकलन करने के लिए करता है। यह जमा का न्यूनतम प्रतिशत है जिसे एक वाणिज्यिक बैंक को नकद, सोना या अन्य प्रतिभूतियों के रूप में बनाए रखना होता है। यह मूल रूप से आरक्षित आवश्यकता है जो बैंकों से ग्राहकों को ऋण देने से पहले रखने की अपेक्षा की जाती है। एसएलआर बढ़ाने से बैंक सरकारी प्रतिभूतियों में अधिक पैसा जमा करते हैं और अर्थव्यवस्था में नकदी के स्तर को कम करते हैं। इसके विपरीत करने से अर्थव्यवस्था में नकदी प्रवाह को बनाए रखने में मदद मिलती है। एसएलआर कम करने से बैंकों के पास अधिक तरलता बच जाती है, जो बदले में अर्थव्यवस्था में विकास और मांग को बढ़ावा दे सकती है। अतः कथन 1 सही नहीं है।
  • सीमांत स्थायी सुविधा (MSF) अनुसूचित बैंकों के लिए एक आपातकालीन स्थिति में आरबीआई से रातोंरात उधार लेने के लिए एक खिड़की है, जब इंटरबैंक तरलता पूरी तरह से सूख जाती है। एमएसएफ दर में वृद्धि के साथ, बैंकों के लिए उधार लेने की लागत बढ़ जाती है जिसके परिणामस्वरूप उधार देने के लिए उपलब्ध संसाधन कम हो जाते हैं। अत: कथन 2 सही है।
  • रेपो दर, या पुनर्खरीद दर, ब्याज की प्रमुख मौद्रिक नीति दर है जिस पर केंद्रीय बैंक या भारतीय रिजर्व बैंक (RBI) बैंकों को तरलता समायोजन सुविधा के तहत सरकार और अन्य अनुमोदित प्रतिभूतियों के संपार्श्विक के खिलाफ अल्पकालिक धन उधार देता है। (एलएएफ)। बैंक दर वह ब्याज दर है जो आरबीआई अपने दीर्घकालिक ऋणों पर वसूल करता है। विस्तारवादी मौद्रिक नीति के तहत, आरबीआई बैंकिंग क्षेत्र में तरलता बढ़ाने के लिए रेपो दर और बैंक दर को कम करता है। अत: कथन 3 सही नहीं है।

अतः विकल्प (ब) सही उत्तर है।

88

1991 के आर्थिक उदारीकरण के बाद भारतीय अर्थव्यवस्था के संदर्भ में, निम्नलिखित कथनों पर विचार कीजिएः

  1. शहरी क्षेत्रों में श्रमिक उत्पादकता (2004-05 की कीमतों पर प्रति कार्यकर्ता रुपये) में वृद्धि हुई जबकि ग्रामीण क्षेत्रों में यह घट गई।
  2. कार्यबल में ग्रामीण क्षेत्रों की प्रतिशत हिस्सेदारी में लगातार वृद्धि हुई।
  3. ग्रामीण क्षेत्रों में, गैर-कृषि अर्थव्यवस्था में वृद्धि में वृद्धि हुई।
  4. ग्रामीण रोजगार में वृद्धि दर में कमी आई है।

ऊपर दिए गए कथनों में से कौन-सा/से सही है/हैं?

व्याख्या
  • 2017 की नीति आयोग की रिपोर्ट, "भारत की ग्रामीण अर्थव्यवस्था की बदलती संरचना, रोजगार और विकास के लिए निहितार्थ", ग्रामीण अर्थव्यवस्था के बारे में निम्नलिखित जानकारी प्रदान करती है।
  • ग्रामीण और शहरी दोनों क्षेत्रों के लिए श्रमिक उत्पादकता में वृद्धि हुई है। ग्रामीण क्षेत्रों के लिए यह 2004-05 में 37273 और 2011-12 में 101755 था, जबकि शहरी क्षेत्रों के लिए 2004-05 में यह 120419 और 2011-12 में 282515 था। अतः कथन 1 सही नहीं है।
  • कुल कार्यबल में ग्रामीण हिस्सा 1999-2000 में 76.1% से लगातार गिरकर 2011-12 में 70.9% हो गया। अत: कथन 2 सही नहीं है।
  • ग्रामीण उत्पादन संरचना में महत्वपूर्ण परिवर्तनों में से एक गैर-कृषि क्षेत्र का बढ़ता हिस्सा है, जो 1980-81 में 37% से बढ़कर 2009-10 में 65% हो गया, और इस प्रकार यह दर्शाता है कि उत्पादन के मूल्य के संदर्भ में, ग्रामीण अब केवल कृषि नहीं रह गया है। अत: कथन 3 सही है।
  • सुधार पूर्व अवधि के दौरान ग्रामीण रोजगार में 2.16% वार्षिक वृद्धि दर दिखाई गई, जो सुधार के बाद की अवधि में घटकर 1.45% हो गई और आर्थिक त्वरण की अवधि में नकारात्मक (-0.28%) हो गई। अत: कथन 4 सही है।

अतः विकल्प (ब) सही उत्तर है।

89

निम्नलिखित कथनों पर विचार करें:

  1. कृषि क्षेत्र को अल्पकालिक ऋण वितरण के मामले में, जिला केंद्रीय सहकारी बैंक (डीसीसीबी) अनुसूचित वाणिज्यिक बैंकों और क्षेत्रीय ग्रामीण बैंकों की तुलना में अधिक ऋण प्रदान करते हैं।
  2. डीसीसीबी के सबसे महत्वपूर्ण कार्यों में से एक प्राथमिक कृषि ऋण समितियों को धन उपलब्ध कराना है।

ऊपर दिए गए कथनों में से कौन-सा/से सही है/हैं?

व्याख्या
  • सहकारी बैंक सहकारी आधार पर स्थापित और साधारण बैंकिंग व्यवसाय में काम करने वाली संस्था है।
  • ग्रामीण भारत में, एक त्रिस्तरीय ग्रामीण सहकारी संरचना मौजूद है।
    • टियर- I: इसमें राज्य स्तर पर राज्य सहकारी बैंक (StCB) शामिल हैं;
    • टियर- II: इसमें जिला स्तर पर केंद्रीय सहकारी बैंक (CCB) शामिल हैं; तथा
    • टियर- III: इसमें प्राथमिक कृषि ऋण समितियां (पीएसीएस) शामिल हैं।
  • आरबीआई की एक रिपोर्ट के अनुसार, 2016-17 में, अनुसूचित वाणिज्यिक बैंकों ने कृषि और संबद्ध ऋण में प्रमुख हिस्सेदारी (78- ​​80%) का योगदान दिया। सहकारी संस्थाएं भी कृषि ऋण प्रदान करने में महत्वपूर्ण भूमिका निभाती हैं और सभी सहकारी बैंकों/संस्थाओं (अर्थात एसटीसीबी, डीसीसीबी और पैक्स को मिलाकर) का हिस्सा 15-16 प्रतिशत है। आरआरबी ने कृषि ऋण के शेष 5% का योगदान दिया। अत: कथन 1 सही नहीं है।
  • जिला केंद्रीय सहकारी बैंक का सबसे महत्वपूर्ण कार्य जिले में इससे संबद्ध प्राथमिक सहकारी समितियों को वित्तीय सहायता प्रदान करना है। अत: कथन 2 सही है।

अतः विकल्प (ब) सही उत्तर है।

90

भारत में, व्यक्तियों के लिए साइबर बीमा के तहत, धन की हानि और अन्य लाभों के भुगतान के अलावा, आम तौर पर निम्नलिखित में से कौन से लाभ कवर किए जाते हैं?

  1. किसी के कंप्यूटर तक पहुंच को बाधित करने वाले मैलवेयर के मामले में कंप्यूटर सिस्टम की बहाली की लागत
  2. एक नए कंप्यूटर की कीमत अगर कुछ बदमाश जानबूझकर इसे नुकसान पहुंचाते हैं, अगर ऐसा साबित होता है
  3. साइबर जबरन वसूली के मामले में नुकसान को कम करने के लिए एक विशेष सलाहकार को काम पर रखने की लागत
  4. यदि कोई तीसरा पक्ष मुकदमा दायर करता है तो न्यायालय में बचाव की लागत

नीचे दिए गए कूट का प्रयोग कर सही उत्तर चुनिए:

व्याख्या
  • साइबर बीमा को व्यवसायों को साइबर हमलों के संभावित प्रभावों से बचाने के लिए डिज़ाइन किया गया है। यह साइबर हमले/उल्लंघन होने के बाद, लागतों की भरपाई करके एक संगठन को जोखिम जोखिम को कम करने में मदद करता है। सरल शब्दों में, साइबर बीमा को साइबर उल्लंघनों से जुड़े शुल्क, व्यय और कानूनी लागतों को कवर करने के लिए डिज़ाइन किया गया है।
  • कवरेज में शामिल हैं:
    • उल्लंघन की घटनाओं पर प्रतिक्रिया (अधिसूचना, कॉल सेंटर सेवा, उल्लंघन समाधान, शमन सेवाएं, जनसंपर्क और संकट प्रबंधन)।
    • वकीलों, पेशेवर शुल्क, प्रशासन लागत आदि सहित जांच और जुर्माना।
    • फोरेंसिक, आईटी ऑडिट, संकट प्रबंधन, कानूनी लागत जैसे खर्च।
    • गोपनीयता और डेटा दायित्व
    • व्यक्तिगत पहचान योग्य जानकारी का नुकसान।
    • कॉर्पोरेट गोपनीय जानकारी का नुकसान।
    • नेटवर्क दायित्व जैसे डीडीओएस अटैक।
    • कॉपीराइट मुद्दों सहित मल्टीमीडिया कवर।
    • व्यापार व्याख्या
    • आय की हानि, व्यवसाय में रुकावट की लागत, सिस्टम की क्षति और बहाली की लागत, कोई भी अतिरिक्त खर्च।
    • साइबर चोरी
    • फंड ट्रांसफर धोखाधड़ी
    • ई-चोरी नुकसान
    • ई-संचार हानि
    • साइबर जबरन वसूली
  • अत: 1, 3 और 4 उल्लिखित लाभ सही हैं।

अतः विकल्प (ब) सही उत्तर है।

91

भारत के सांस्कृतिक इतिहास के संदर्भ में निम्नलिखित युग्मों पर विचार कीजिएः

  1. परिव्राजक - त्यागी और पथिक
  2. श्रमण - उच्च पद के साथ पुजारी
  3. उपासक - बौद्ध धर्म के अनुयायी

ऊपर दिए गए युग्मों में से कौन-सा सही सुमेलित है?

व्याख्या
  • यात्रा करने वाले भिक्षुओं को आमतौर पर परिव्राजक कहा जाता था। वे सत्य के साधक थे जो एक स्थान से दूसरे स्थान पर विचरण करते हुए स्थायी रूप से किसी एक स्थान पर नहीं रहते थे। अत: युग्म 1 सही सुमेलित है।
  • संस्कृत में श्रमण का अर्थ है "जो प्रयास करता है" अर्थात सत्य का एहसास करने के लिए। श्रमणों ने आध्यात्मिक मुक्ति की खोज में एक तपस्वी, या सख्त और आत्म-निषेध, जीवन शैली का अभ्यास किया। वे आमतौर पर भिक्षुओं के रूप में जाने जाते थे। अत: युग्म 2 सुमेलित नहीं है।
  • Up?saka संस्कृत और P?li शब्द "अटेंडेंट" के लिए है। यह बौद्ध धर्म के अनुयायियों (या, ऐतिहासिक रूप से, गौतम बुद्ध के) का शीर्षक है, जो बौद्ध क्रम में भिक्षु, नन या नौसिखिए मठवासी नहीं हैं, और जो कुछ प्रतिज्ञा करते हैं। अत: युग्म 3 सही सुमेलित है।

अतः विकल्प (ब) सही उत्तर है।

92

भारतीय हाथियों के संदर्भ में निम्नलिखित कथनों पर विचार कीजिएः

  1. हाथी समूह की नेता मादा होती है।
  2. गर्भधारण की अधिकतम अवधि 22 महीने हो सकती है।
  3. एक हाथी सामान्य रूप से केवल 40 वर्ष की आयु तक ही ब्याने पर जा सकता है।
  4. भारत के राज्यों में हाथियों की सर्वाधिक जनसंख्या केरल में है।

ऊपर दिए गए कथनों में से कौन-सा/से सही है/हैं?

व्याख्या
  • हाथियों के झुंड का नेतृत्व सबसे पुरानी और सबसे बड़ी महिला सदस्य (मातृक के रूप में जाना जाता है) द्वारा किया जाता है। इस झुंड में कुलपिता की बेटियां और उनकी संतान शामिल हैं। अत: कथन 1 सही है।
  • हाथियों में सभी स्तनधारियों की सबसे लंबी ज्ञात गर्भकालीन (गर्भावस्था) अवधि होती है, जो 680 दिनों (22 महीने) तक चलती है। अत: कथन 2 सही है।
  • 14 से 45 वर्ष के बीच की महिलाएं लगभग हर चार साल में बछड़ों को जन्म दे सकती हैं, जिसमें औसत अंतर जन्म अंतराल 52 वर्ष की आयु तक पांच वर्ष और 60 वर्ष की आयु तक छह वर्ष तक बढ़ जाता है। इसलिए, कथन 3 सही नहीं है।
  • हाथी जनगणना (2017) के अनुसार, कर्नाटक में हाथियों की संख्या सबसे अधिक (6,049) है, इसके बाद असम (5,719) और केरल (3,054) का स्थान है। अत: कथन 4 सही नहीं है।

इसलिए, विकल्प (ए) सही उत्तर है।

93

निम्नलिखित में से कौन से संरक्षित क्षेत्र कावेरी बेसिन में स्थित हैं?

  1. नागरहोल राष्ट्रीय उद्यान
  2. पापिकोंडा राष्ट्रीय उद्यान
  3. सत्यमंगलम टाइगर रिजर्व
  4. वायनाड वन्यजीव अभयारण्य

नीचे दिए गए कूट का प्रयोग कर सही उत्तर चुनिए:

व्याख्या
  • नागरहोल राष्ट्रीय उद्यान:
    • इस पार्क को राजीव गांधी राष्ट्रीय उद्यान के नाम से भी जाना जाता है। यह कर्नाटक के दो जिलों मैसूर और कोडागु में स्थित है।
    • पार्क को 1955 में एक वन्यजीव अभयारण्य के रूप में स्थापित किया गया था और इसे 1988 में एक राष्ट्रीय उद्यान में अपग्रेड किया गया था।
    • 1999 में इस पार्क को 37वां टाइगर रिजर्व घोषित किया गया था।
    • कावेरी नदी की एक सहायक नदी काबिनी, पार्क में बहने वाली सबसे बड़ी नदी है।

अत: विकल्प 1 सही है।

  • पापिकोंडा राष्ट्रीय उद्यान:
    • यह पार्क आंध्र प्रदेश के पूर्वी और पश्चिमी गोदावरी जिलों में 1012.86 वर्ग किलोमीटर में फैला हुआ है।
    • इसने ऐतिहासिक रूप से संरक्षण के विभिन्न स्तरों का अनुभव किया है, जिसकी शुरुआत 1882 में एक आरक्षित वन, 1978 में एक वन्यजीव अभयारण्य और 2008 से एक राष्ट्रीय उद्यान के रूप में हुई थी।
    • पार्क गोदावरी नदी के बाएं और दाएं किनारे पर स्थित है और पूर्वी घाट की पापिकोंडा पहाड़ी श्रृंखला से होकर गुजरता है।

अतः विकल्प 2 सही नहीं है।

  • सत्यमंगलम टाइगर रिजर्व:
    • सत्यमंगलम वन्यजीव अभयारण्य और टाइगर रिजर्व तमिलनाडु राज्य के इरोड जिले में पश्चिमी घाट के साथ एक संरक्षित क्षेत्र और बाघ अभयारण्य है।
    • इरोड जिले के उत्तरी भाग में, पलार नदी बहती है और कावेरी नदी में गिरती है।

अत: विकल्प 3 सही है।

  • वायनाड वन्यजीव अभयारण्य:
    • केरल में स्थित, वायनाड वन्यजीव अभयारण्य (WWS) नीलगिरि बायोस्फीयर रिजर्व का एक अभिन्न अंग है। इसकी स्थापना 1973 में हुई थी।
    • यह कर्नाटक के नागरहोल और बांदीपुर और तमिलनाडु के मुदुमलाई के बाघ अभयारण्यों से सटा हुआ है।
    • काबिनी नदी (कावेरी नदी की एक सहायक नदी) अभयारण्य से होकर बहती है।

अत: विकल्प 4 सही है।

इसलिए, विकल्प (सी) सही उत्तर है।

94

भारत की जैव विविधता के संदर्भ में, सीलोन फ्रॉगमाउथ, कॉपरस्मिथ बारबेट, ग्रे-चिन्ड मिनिवेट और व्हाइट-थ्रोटेड रेडस्टार्ट हैं

व्याख्या
  • ये सभी एवियन पारिस्थितिकी का हिस्सा हैं।
  • सीलोन फ्रॉगमाउथ:
    • यह एक भूरे-भूरे रंग की निशाचर पक्षी प्रजाति है जो पश्चिमी घाट और श्रीलंका के जंगलों में पाई जाती है।
    • यह अपने चौड़े, झुके हुए चोंच के साथ भट्ठा जैसे नथुने और एक बड़े सिर के साथ आगे की ओर आंखों के साथ प्रतिष्ठित है।
    • वन आवरण के विनाश के साथ, यह प्रजाति अब काजू के बागानों में शरण लेने के लिए मजबूर है।
  • कॉपरस्मिथ बारबेट:
    • इसे क्रिमसन-ब्रेस्टेड बारबेट और कॉपरस्मिथ भी कहा जाता है, यह क्रिमसन माथे और गले के साथ एक एशियाई बारबेट है, जो अपने मेट्रोनोमिक कॉल के लिए जाना जाता है जो एक तांबे के हथौड़ा के साथ हड़ताली धातु के समान लगता है।
    • यह भारतीय उपमहाद्वीप और दक्षिण पूर्व एशिया के कुछ हिस्सों में एक निवासी पक्षी है।
  • सफेद गले वाला रेडस्टार्ट:
    • यह Muscicapidae परिवार में पक्षी की एक प्रजाति है।
    • यह भूटान, चीन, भारत, म्यांमार और नेपाल में पाया जाता है।
  • ग्रे-चिन्ड मिनिवेट:
    • यह कैंपेफैगिडे परिवार में पक्षी की एक प्रजाति है।
    • यह बांग्लादेश, भूटान, कंबोडिया, चीन, भारत, इंडोनेशिया, लाओस, मलेशिया, म्यांमार, नेपाल, ताइवान, थाईलैंड और वियतनाम में पाया जाता है।
    • इसका प्राकृतिक आवास उपोष्णकटिबंधीय या उष्णकटिबंधीय नम तराई के जंगल हैं।

इसलिए, विकल्प (ए) सही उत्तर है।

95

निम्नलिखित में से कौन सा संरक्षित क्षेत्र भारतीय दलदली हिरण (बारासिंघा) की एक उप-प्रजाति के संरक्षण के लिए जाना जाता है जो कठोर जमीन पर अच्छी तरह से पनपता है और विशेष रूप से ग्रामीण है?

व्याख्या
  • मध्य प्रदेश का राज्य पशु, हार्ड ग्राउंड दलदल हिरण या बारासिंघा (Rucervus duvaucelii) , कान्हा राष्ट्रीय उद्यान और टाइगर रिजर्व (KNPTR) में पुनरुद्धार देख रहा है।
  • कान्हा राष्ट्रीय उद्यान में दलदली हिरण विलुप्त होने के करीब था। हालांकि, संरक्षण प्रयासों के साथ, वर्तमान में जनसंख्या लगभग 800 है।
  • हिरण सतपुड़ा पहाड़ियों की मैकल रेंज पर कान्हा राष्ट्रीय उद्यान और टाइगर रिजर्व के लिए स्थानिक है। बंदी प्रजनन और आवास सुधार जैसे उपायों का इस्तेमाल किया गया।

इसलिए, विकल्प (ए) सही उत्तर है।

96

स्टील स्लैग निम्नलिखित में से किसके लिए सामग्री हो सकता है?

  1. बेस रोड का निर्माण
  2. कृषि भूमि में सुधार
  3. सीमेंट का उत्पादन

नीचे दिए गए कूट का प्रयोग कर सही उत्तर चुनिए:

व्याख्या
  • स्टील स्लैग स्टील बनाने की प्रक्रिया का एक उप-उत्पाद है। यह पिघले हुए स्टील को स्टील बनाने वाली भट्टियों में अशुद्धियों से अलग करने के दौरान उत्पन्न होता है। स्लैग पिघले हुए तरल के रूप में होता है और सिलिकेट और ऑक्साइड का एक जटिल समाधान होता है जो ठंडा होने पर जम जाता है।
  • स्टील स्लैग का उपयोग बेस कोर्स सामग्री, डामर सड़क, ट्रैक या सतह की सतह परत के नीचे की सामग्री के रूप में किया जाता है। अत: कथन 1 सही है।
  • मिट्टी की अम्लता को ठीक करने की क्षमता के कारण कृषि क्षेत्र में स्टील स्लैग का उपयोग किया जा सकता है, क्योंकि इसमें पौधों के लिए कुछ पोषक तत्व होते हैं और सिलिकेट उर्वरक के रूप में भी होते हैं जो पौधों को सिलिकॉन प्रदान करने में सक्षम होते हैं। अत: कथन 2 सही है।
  • सीमेंट के उत्पादन के लिए स्टील-स्लैग का उपयोग किया जा सकता है। इसके अलावा, स्लैग सीमेंट का व्यापक रूप से कंक्रीट में उपयोग किया जाता है, या तो एक अलग सीमेंटिटियस घटक के रूप में या मिश्रित सीमेंट के हिस्से के रूप में। यह ताकत बढ़ाने, पारगम्यता को कम करने, रासायनिक हमले के प्रतिरोध में सुधार और रीबार जंग को रोकने के लिए पोर्टलैंड सीमेंट के साथ सहक्रियात्मक रूप से काम करता है। अत: कथन 3 सही है।

इसलिए, विकल्प (डी) सही उत्तर है।

97

कस्तूरी मृग को उसके प्राकृतिक आवास में खोजने के लिए निम्नलिखित में से सबसे संभावित स्थान कौन से हैं?

  1. अस्कोट वन्यजीव अभयारण्य
  2. गंगोत्री राष्ट्रीय उद्यान
  3. किशनपुर वन्यजीव अभयारण्य
  4. मानस राष्ट्रीय उद्यान

नीचे दिए गए कूट का प्रयोग कर सही उत्तर चुनिए:

व्याख्या
  • कस्तूरी मृग मुख्य रूप से दक्षिणी एशिया के पहाड़ों, विशेष रूप से हिमालय के जंगलों और अल्पाइन स्क्रब आवासों में रहते हैं।
  • अस्कोट वन्यजीव अभयारण्य:
    • यह भारत के उत्तराखंड राज्य में असकोट के पास पिथौरागढ़ से 54 किमी दूर स्थित है।
    • इस अभयारण्य की स्थापना मुख्य रूप से कस्तूरी मृग और उसके आवास के संरक्षण के उद्देश्य से की गई है। अत: 1 सही है।
  • गंगोत्री राष्ट्रीय उद्यान:
    • 1989 में स्थापित, यह उत्तराखंड में भागीरथी नदी के ऊपरी जलग्रहण क्षेत्र में स्थित है।
    • पार्क नीली भेड़, हिमालयी तहर, कस्तूरी मृग, हिम तेंदुए आदि का घर है। इसलिए, 2 सही है।
  • किशनपुर वन्यजीव अभयारण्य:
    • यह उत्तर प्रदेश में मैलानी के पास दुधवा टाइगर रिजर्व का एक हिस्सा है। इसकी स्थापना 1972 में हुई थी।
    • इस अभयारण्य में बाघ, चीतल, हॉग डियर, जंगली सूअर, ऊदबिलाव और कई अन्य जानवर अपने लिए घर पाते हैं।
    • खुली घास के मैदानों में बड़ी संख्या में निवासी और प्रवासी पक्षी जैसे फाल्कन्स, ड्रोंगोस, उल्लू, एग्रेट्स और मोर देखे जा सकते हैं, जो बारहमासी धाराओं से गुजरते हैं। अत: 3 सही नहीं है।
  • मानस राष्ट्रीय उद्यान:
    • मानस राष्ट्रीय उद्यान असम में हिमालय की तलहटी में स्थित है। यह भूटान में रॉयल मानस नेशनल पार्क से सटा हुआ है।
    • यह अपने दुर्लभ और लुप्तप्राय स्थानिक वन्यजीवों के लिए जाना जाता है जैसे कि असम की छत वाला कछुआ, हर्पिड हरे, गोल्डन लंगूर और पिग्मी हॉग।
    • मानस जंगली भैंसों की आबादी के लिए प्रसिद्ध है। अत: 4 सही नहीं है।

इसलिए, विकल्प (ए) सही उत्तर है।

98

ग्रामीण सड़क निर्माण में, पर्यावरणीय स्थिरता सुनिश्चित करने या कार्बन फुटप्रिंट को कम करने के लिए निम्नलिखित में से किसका उपयोग पसंद किया जाता है?

  1. कॉपर स्लैग
  2. कोल्ड मिक्स डामर तकनीक
  3. जियोटेक्सटाइल
  4. हॉट मिक्स डामर तकनीक
  5. पोर्टलैंड सीमेंट

नीचे दिए गए कूट का प्रयोग कर सही उत्तर चुनिए:

व्याख्या
  • कॉपर स्लैग तांबे के गलाने और शोधन के दौरान प्राप्त होने वाला उप-उत्पाद है। अपशिष्ट कॉपर स्लैग का उपयोग अपघर्षक उपकरण, सड़क निर्माण और गिट्टी के रूप में किया जा सकता है। सीमेंट और कंक्रीट में कॉपर स्लैग का उपयोग सभी संबंधित उद्योगों के लिए संभावित पर्यावरणीय और साथ ही आर्थिक लाभ प्रदान करता है, खासकर उन क्षेत्रों में जहां काफी मात्रा में कॉपर स्लैग का उत्पादन होता है। अत: 1 सही है।
  • कोल्ड मिक्स डामर का उत्पादन बिना गर्म किए गए खनिज समुच्चय को इमल्सीफाइड बिटुमेन या फोमेड बिटुमेन के साथ मिलाकर किया जाता है। कोल्ड डामर मिक्स आमतौर पर हल्के से मध्यम ट्रैफिक वाली सड़कों के लिए उपयुक्त होते हैं जब बेस और सतह के पाठ्यक्रमों में उपयोग किया जाता है। अत: 2 सही है।
  • भू टेक्सटाइल एक सिंथेटिक पारगम्य कपड़ा सामग्री है जिसका उपयोग मिट्टी की विशेषताओं में सुधार करने के लिए किया जाता है और इसमें मिट्टी के साथ उपयोग किए जाने पर अलग करने, फ़िल्टर करने, सुदृढ़ करने, संरक्षित करने और निकालने की क्षमता होती है। इसका उपयोग ज्यादातर सड़क निर्माण में निस्पंदन और पृथक्करण के लिए किया जाता है। अत: 3 सही है।
  • हॉट मिक्स डामर (एचएमए) लगभग 95% पत्थर, रेत या बजरी का एक संयोजन है जो कच्चे तेल के उत्पाद डामर सीमेंट द्वारा एक साथ बंधे हैं। हॉट मिक्स तकनीक के व्यापक उपयोग से पर्यावरण प्रदूषण होता है क्योंकि ये पौधे भारी मात्रा में ग्रीनहाउस गैसों का उत्सर्जन करते हैं। अत: 4 सही नहीं है।
  • पोर्टलैंड सीमेंट एक महीन पिसे हुए पाउडर के रूप में एक बाध्यकारी सामग्री है, जिसे चूना पत्थर और मिट्टी के मिश्रण को जलाने और पीसकर बनाया जाता है। इसके उत्पादन से ग्रीनहाउस गैसें निकलती हैं। अत: 5 सही नहीं है।

इसलिए, विकल्प (ए) सही उत्तर है।

99

निम्नलिखित कथनों पर विचार करें:

  1. कोयले की राख में आर्सेनिक, सीसा और पारा होता है।
  2. कोयले से चलने वाले बिजली संयंत्र पर्यावरण में सल्फर डाइऑक्साइड और नाइट्रोजन के ऑक्साइड छोड़ते हैं।
  3. भारतीय कोयले में राख की उच्च मात्रा पाई जाती है।

ऊपर दिए गए कथनों में से कौन-सा/से सही है/हैं?

व्याख्या
  • कोयले की राख, जिसे कोयला दहन अवशेष या सीसीआर भी कहा जाता है, मुख्य रूप से कोयले से चलने वाले बिजली संयंत्रों में कोयले के जलने से उत्पन्न होती है। इसमें पारा, कैडमियम और आर्सेनिक जैसे संदूषक होते हैं। उचित प्रबंधन के बिना, ये संदूषक जलमार्ग, भूजल, पेयजल और वायु को प्रदूषित कर सकते हैं। अत: कथन 1 सही है।
  • कोयला आधारित ताप विद्युत संयंत्र सल्फर डाइऑक्साइड और नाइट्रोजन ऑक्साइड उत्सर्जन के प्रमुख स्रोत हैं। अत: कथन 2 सही है।
  • कोल इंडिया लिमिटेड के अनुसार, देश में उत्पादित कोयले की राख सामग्री आम तौर पर 25 से 45% होती है जबकि आयातित कोयले की औसत राख सामग्री 10 से 20% तक भिन्न होती है। अत: कथन 3 सही है।

इसलिए, विकल्प (डी) सही उत्तर है।

100

खेती में बायोचार का क्या उपयोग है?

  1. बायोचार का उपयोग ऊर्ध्वाधर खेती में बढ़ते माध्यम के एक भाग के रूप में किया जा सकता है।
  2. जब बायोचार बढ़ते माध्यम का हिस्सा होता है, तो यह नाइट्रोजन-फिक्सिंग सूक्ष्मजीवों के विकास को बढ़ावा देता है।
  3. जब बायोचार बढ़ते माध्यम का हिस्सा होता है, तो यह बढ़ते माध्यम को लंबे समय तक पानी बनाए रखने में सक्षम बनाता है।

ऊपर दिए गए कथनों में से कौन-सा/से सही है/हैं?

व्याख्या
  • बायोचार एक झरझरा कार्बनयुक्त ठोस है जो ऑक्सीजन-सीमित वातावरण में उच्च तापमान के तहत विभिन्न बायोमास फीडस्टॉक्स को गर्म करके उत्पादित किया जाता है।
  • चूंकि बायोचार मिट्टी के प्रोफाइल के माध्यम से लंबवत रूप से माइग्रेट करता है, इसलिए इसका उपयोग ऊर्ध्वाधर खेती में बढ़ते माध्यम के हिस्से के रूप में किया जा सकता है। अत: कथन 1 सही है।
  • इसकी सोखने की क्षमता के कारण, कुछ बायोचार्स में भारी धातुओं, कीटनाशकों, जड़ी-बूटियों और हार्मोन को स्थिर करने की क्षमता होती है; जलमार्गों में नाइट्रेट लीचिंग और मल जीवाणुओं को रोकना; और मिट्टी से एन 2 ओ और सीएच 4 उत्सर्जन को कम करें। अत: कथन 2 सही है।
  • बायोचार पौधों को बढ़ने के लिए मिट्टी में पानी और पोषक तत्वों को बनाए रखने में मदद कर सकता है। अत: कथन 3 सही है।
    इसलिए, विकल्प (डी) सही उत्तर है।
 

Admin

My Name is Priyanshu Thakur and I am preparing for Civil Services! And I am from Bihar. My aim is to cooperate with the participants preparing for competitive exams in Hindi & English medium. It is my fervent desire to get the affection of all of you and to serve you by distributing my acquired experiences and knowledge.

Post a Comment

Previous Post Next Post

Contact Form